You are on page 1of 58

EMINENT DOMAIN Republic vs. La Orden De PP. Benedictinos De Filipinas, G.R. No.

L12792, February 28, 1961 The Power of Eminent Domain To ease and solve the daily traffic congestion on Legarda Street, the Government drew plans to extend Azcarraga street from its junction with Mendiola street, up to the Sta. Mesa Rotonda, Sampaloc, Manila. The petitioner in this case is the Republic of the Philippines through the Office of the Solicitor General; and the respondent is La Orden de PP. Benedictinos de Filipinas, a domestic religious corporation that owns the San Beda College. Facts: To ease and solve the daily traffic congestion on Legarda Street, the Government drew plans to extend Azcarraga St. (now Recto) from its junction with Mendiola St., up to the Sta. Mesa Rotonda, Sampaloc, Manila. To carry out this plan it offered to buy a portion of approximately 6,000 square meters of a bigger parcel belonging to La Orden situated on Mendiola St. Not having been able to reach an agreement on the matter with the owner, the Government instituted an expropriation proceeding. On May 27, 1957 the trial court valued the property in question at P270,000.00 and authorized appellant to take immediate possession upon depositing said amount. The deposit having been made with the City Treasurer of Manila, the trial court issued the corresponding order directing the Sheriff of Manila to place appellant in possession of the property aforesaid. In answer, the herein appellee filed a motion to dismiss the complaint based on the grounds that: (1) the property sought to be expropriated is already dedicated to public use and therefore is not subject to expropriation; (2) there is no necessity for the proposed expropriation; (3) the proposed Azcarraga Extension could pass through a different site which would entail less expense to the Government and which would not necessitate the expropriation of a property dedicated to education. The trial court granted the motion, holding that the expropriation was not of extreme necessity. Hence this present petition. Issue: Whether or not there is a genuine necessity for the exercise of the Power of Eminent Domain.

Held: It is the rule in this jurisdiction that private property may be expropriated for public use and upon payment of just compensation; that condemnation of private property is justified only if it is for the public good and there is a genuine necessity therefor of a public character. Consequently, the courts have the power to inquire into the legality of the exercise of the right of eminent domain and to determine whether or not there is a genuine necessity therefor. It does not need extended argument to show that whether or not the proposed opening of the Azcarraga extension is a necessity in order to relieve the daily congestion of traffic on Legarda St., is a question of fact dependent not only upon the facts of which the trial court very liberally took judicial notice but also up on other factors that do not appear of record and must, therefore, be established by means of evidence. The parties should have been given an opportunity to present their respective evidence upon these factors and others that might be of direct or indirect help in determining the vital question of fact involved, namely, the need to open the extension of Azcarraga street to ease and solve the traffic congestion on Legarda street. WHEREFORE, the appealed order of dismissal is set aside and the present case is remanded to the trial court for further proceedings in accordance with this decision. City of Manila v Chinese Community Facts: The City of Manila, plaintiff herein, prayed for the expropriation of a portion private cemetery for the conversion into an extension of Rizal Avenue. Plaintiff claims that it is necessary that suchpublic improvement be made in the said portion of the private cemetery and that the said lands are within their jurisdiction. Defendants herein answered that the said expropriation was not necessary because other routes were available. They further claimed that the expropriation of the cemetery would create irreparable loss and injury to them and to all those persons owing and interested in the graves and monuments that would have to be destroyed. The lower court ruled that the said public improvement was not necessary on the particular-strip of land in question. Plaintiff herein assailed that they have the right to

exercise the power of eminent domain and that the courts have no right to inquire and determine the necessity of the expropriation. Thus, the same filed an appeal. Issue: Whether or not the courts may inquire into, and hear proof of the necessity of the expropriation. Held: The courts have the power of restricting the exercise ofeminent domain to the actual reasonable necessities of the case and for the purposes designated by the law. The moment the municipalcorporation or entity attempts to exercise the authority conferred, it must comply with the conditions accompanying the authority. The necessity for conferring the authority upon a municipal corporation toexercise the right of eminent domain is admittedly within the power of the legislature. But whether or not the municipal corporation or entity is exercising the right in a particular case under the conditions imposed by the general authority, is a question that the courts have the right to inquire to. RP v PLDT Facts: The plaintiff Republic of the Philippines is a political entity exercising government powers through one of its branches, the Bureau of Telecommunication. Herein defendant, PLDT is a public service corporation holding a franchise to install operates and maintains atelephone system. After its creation, the BOT set up its owngovernment telephone system by utilizing its own appropriations and other equipment and by renting trunk lines of the PLDT to enable the govt offices to call privately. BOT entered into an agreement with the RCA communications for joint overseas telephone service whereby BOT would convey overseas calls received by RCA to local residents. PLDT complained to the BOT that it was a violation of the condition of theiragreement since the BOT had used trunk lines only for the use ofgovernment offices but even to serve private persons or the general public in competition with the business of PLDT. Subsequently, the plaintiff commenced suit against PLDT asking the court judgment be rendered ordering the PLDT to execute a contract with the plaintiff, through the BOT for the use of the facilities of PLDT's telephone systemthroughout the country under such conditions as the court may consider reasonable. The CFI rendered judgment stating that it could not compel

PLDT to enter into such agreement. Hence this petition. Issue: Whether or Not PLDT may be compelled to enter into suchagreement. Held: Yes, the state, may, in the interest of national welfare transfer utilities to public ownership upon payment of just compensation, there is no reason why the state ma not require a public utility to render services in the general interest provided just compensation is paid. Republic v Castellvi Facts: In 1947, the republic, through the Armed Forces of thePhilippines (AFP), entered into a lease agreement with Castelvi on a year-to-year basis. When Castelvi gave notice to terminate the lease in 1956, the AFP refused. She then instituted an ejectment proceeding against the AFP. In 1959, however, the republic commenced the expropriation proceedings for the land in question. Issue: Whether or Not the compensation should be determined as of 1947 or 1959. Held: The Supreme Court ruled that the taking should not be reckoned as of 1947, and that just compensation should not be determined on the basis of the value of the property as of that year. The requisites for taking are: 1) the expropriator must enter a private property, 2) the entry must be for more than a momentary period, 3) it must be under warrant or color of authorities, 4) the property must be devoted for public use or otherwise informally appropriated or injuriously affected, and 5) the utilization of the property for public use must be such a way as to oust the owner and deprive him of beneficial enjoyment of the property. Under Sec. 4 Rule 67 of the Rules of Court, just compensation is to be determined as of the date of the filing of the complaint. The Supreme Court has ruled that when the taking of the property sought to be expropriated coincides with the commencement of the expropriation proceedings, or takes place subsequent to the filing of the complaint for eminent domain, the just compensation should be determined as of the date of the filing of thecomplaint. In the instant case, it is undisputed that the Republic was placed in possession of the Castelvi property, by authority of court, on August 10, 1959. The

taking of the Castelvi property for the purposes of determining the just compensation to be paid must, therefore, be reckoned as of June 26, 1959 when the complaint for eminent domainwas filed. There is no basis to the contention of the Republic that a lease on a year-to-year basis can give rise to permanent right to occupy since by express provision a lease made for a determinate time, as was the lease of Castelvi land in the instant case, ceases upon the day fixed, without need of a demand (Art. 1669, New Civil Code). The Supreme Court, however, did not apply Art. 1250 of the New Civil Code for the adjustment of the peso rate in times of extraordinary inflation or deflation because in eminent domain cases the obligation to pay arises from law independent of contract. PP v Fajardo Facts: The municipal council of baao, camarines sur stating among others that construction of a building, which will destroy the view of the plaza, shall not be allowed and therefore be destroyed at the expense of the owner, enacted an ordinance. Herein appellant filed a written request with the incumbent municipal mayor for a permit to construct a building adjacent to their gasoline station on a parcel of land registered in Fajardo's name, located along the national highway and separated from the public plaza by a creek. The request was denied, for the reason among others that the proposed building would destroy the view or beauty of the public plaza. Defendants reiterated their request for abuilding permit, but again the mayor turned down the request. Whereupon, appellants proceeded with the construction of the building without a permit, because they needed a place of residence very badly, their former house having been destroyed by a typhoon and hitherto they had been living on leased property. Thereafter, defendants were charged in violation of the ordinance and subsequently convicted. Hence this appeal. Issue: Whether or Not the ordinance is a valid exercise of police power. Held: No. It is not a valid exercise of police power. The ordinance is unreasonable and oppressive, in that it operates to permanently deprive appellants of the right to use their own property; hence, it oversteps the bounds of police power, and amounts to a taking of appellants property

without just compensation. We do not overlook that the modern tendency is to regard the beautification of neighborhoods as conducive to the comfort and happiness of residents. As the case now stands, every structure that may be erected on appellants' land, regardless of its own beauty, stands condemned under the ordinance in question, because it would interfere with the view of the public plaza from the highway. The appellants would, in effect, be constrained to let their land remain idle and unused for the obvious purpose for which it is best suited, being urban in character. To legally achieve that result, the municipality must give appellants just compensation and an opportunity to be heard. Amigable v Cuenca Facts: Victoria Amigable is the registered owner of a particular lot. At the back of her Transfer Certificate of Title (1924), there was no annotation in favor of the government of any right or interest in the property. Without prior expropriation or negotiated sale, thegovernment used a portion of the lot for the construction of the Mango and Gorordo Avenues. On 1958, Amigables counsel wrote the Presidentof the Philippines, requesting payment of the portion of the said lot. It was disallowed by the Auditor General in his 9th Endorsement. Petitioner then filed in the court a quo a complaint against the Republic of the Philippines and Nicolas Cuenca, in his capacity as Commissioner of Public Highways for the recovery of ownership and possession of the lot. According to the defendants, the action was premature because it was not filed first at the Office of the Auditor General. According to them, the right of action for the recovery of any amount had already prescribed, that the Government had not given its consent to be sued, and that plaintiff had no cause of action against the defendants. Issue: Whether or Not, under the facts of the case, appellant may properly sue the government. Held: In the case of Ministerio v. Court of First Instance of Cebu, it was held that when the government takes away property from a private landowner for public use without going through the legal process of expropriation or negotiated sale, the aggrieved party may properly maintain a suit against the government without

violating the doctrine of governmental immunity from suit without its consent. In the case at bar, since no annotation in favor of the government appears at the back of the certificate of title and plaintiff has not executed any deed of conveyance of any portion of the lot to the government, then she remains the owner of the lot. She could then bring an action to recoverpossession of the land anytime, because possession is one of the attributes of ownership. However, since such action is not feasible at this time since the lot has been used for other purposes, the only relief left is for the government to make due compensationprice or value of the lot at the time of the taking. Reyes v Ntnl Housing Authority Facts: Respondent National Housing Authority (NHA) filed complaintsfor the expropriation of sugarcane lands belonging to the petitioners. The stated public purpose of the expropriation was the expansion of the Dasmarias Resettlement Project to accommodate the squatters who were relocated from the Metropolitan Manila area. The trial court rendered judgment ordering the expropriation of these lots and thepayment of just compensation. The Supreme Court affirmed the judgment of the lower court. A few years later, petitioners contended that respondent NHA violated the stated public purpose for the expansion of the Dasmarias Resettlement Project when it failed to relocate the squatters from the Metro Manila area, as borne out by the ocular inspection conducted by the trial court which showed that most of the expropriated properties remain unoccupied. Petitioners likewise question the public nature of the use by respondent NHA when it entered into a contract for the construction of low cost housing units, which is allegedly different from the stated public purpose in the expropriation proceedings. Hence, it is claimed that respondent NHA has forfeited its rights and interests by virtue of the expropriation judgment and the expropriated properties should now be returned to herein petitioners. Issue: Whether or not the judgment of expropriation was forfeited in the light of the failure of respondent NHA to use the expropriated property for the intended purpose but for a totally different purpose. Held: The Supreme Court held in favor of

the respondent NHA. Accordingly, petitioners cannot insist on a restrictive view of the eminent domain provision of the Constitution by contending that the contract forlow cost housing is a deviation from the stated public use. It is now settled doctrine that the concept of public use is no longer limited to traditional purposes. The term "public use" has now been held to be synonymous with "public interest," "public benefit," "public welfare," and "public convenience." Thus, whatever may be beneficially employed for the general welfare satisfies the requirement of public use." In addition, the expropriation of private land for slum clearance and urban development is for a public purpose even if the developed area is later sold to private homeowners, commercials firms, entertainment and service companies, and other private concerns. Moreover, the Constitution itself allows the State to undertake, for the common good and in cooperation with the private sector, a continuing program of urban land reform and housing which will make at affordable cost decent housing and basic services to underprivileged and homeless citizens in urban centers and resettlement areas. The expropriation of private property for the purpose of socialized housing for the marginalized sector is in furtherance of social justice. EPZA v Dulay Facts: The four parcels of land which are the subject of this case is where the Mactan Export Processing Zone Authority in Cebu (EPZA) is to be constructed. Private respondent San Antonio Development Corporation (San Antonio, for brevity), in which these lands are registered under, claimed that the lands were expropriated to thegovernment without them reaching the agreement as to the compensation. Respondent Judge Dulay then issued an order for the appointment of the commissioners to determine the just compensation. It was later found out that the payment of the government to San Antonio would be P15 per square meter, which was objected to by the latter contending that under PD 1533, the basis of just compensation shall be fair and according to the fair market value declared by the owner of the property sought to be expropriated, or by the assessor, whichever is lower. Such objection and the subsequent Motion for Reconsideration were denied and hearing was set for the reception of the

commissioners report. EPZA then filed this petition for certiorari and mandamus enjoining the respondent from further hearing the case. Issue: Whether or Not the exclusive and mandatory mode of determining just compensation in PD 1533 is unconstitutional. Held: The Supreme Court ruled that the mode of determination of just compensation in PD 1533 is unconstitutional. The method of ascertaining just compensation constitutes impermissible encroachment to judicial prerogatives. It tends to render the courts inutile in a matter in which under the Constitution is reserved to it for financial determination. The valuation in the decree may only serve as guiding principle or one of the factors in determining just compensation, but it may not substitute the courts own judgment as to what amount should be awarded and how to arrive at such amount. The determination of just compensation is a judicial function. The executivedepartment or the legislature may make the initial determination but when a party claims a violation of the guarantee in the Bill of Rightsthat the private party may not be taken for public use without just compensation, no statute, decree, or executive order can mandate that its own determination shall prevail over the courts findings. Much less can the courts be precluded from looking into the justness of the decreed compensation. Commission of Public Highways v Judge Burgos On 1924, the government took private respondent Victor Amigable's land for road-right-of-way purpose. On 1959, Amigable filed in the Court of First Instance a complaint to recover the ownership and possession of the land and for damages for the alleged illegal occupation of the land by the government (entitled Victor Amigable vs. Nicolas Cuenco, in his capacity as Commissioner of Public Highways and Republic of the Philippines). Amigable's complaint was dismissed on the grounds that the land was either donated or sold by its owners to enhance its value, and

that in any case, the right of the owner to recover the value of said property was already barred by estoppel and the statute of limitations. Also, the non-suability of the government was invoked. In the hearing, the government proved that the price of the property at the time of taking was P2.37 per square meter. Amigable, on the other hand, presented a newspaper showing that the price was P6.775. The public respondent Judge ruled in favor of Amigable and directed the Republic of the Philippines to pay Amigable the value of the property taken with interest at 6% and the attorney's fees.

The land sought to be expropriated were valued by the NHA at one peso (P1.00) per square meter adopting the market value fixed by the provincial assessor in accordance with presidential decrees prescribing the valuation of property in expropriation proceedings. Together with the complaint was a motion for immediate possession of the properties. The NHA deposited the amount of P158,980.00 with the Phil. Natl Bank, representing the total market value of the subject 25 ha. of land, pursuant to P.D. No. 1224 which defines the policy on the expropriation of private property for socialized housing upon payment of just compensation. On January 17, 1978, respondent Judge Buenaventura S. Guerrero issued a writ of possession pertaining to the subject parcels of land. Petitioners filed a motion for reconsideration on the ground that they had been deprived of the possession of their property without due process of law. This was however, denied. Hence, this petition challenging the orders of respondent Judge and assailing the constitutionality of P.D. No. 1224, as amended. Petitioners contend that the taking of their property subsumed under the topics of public use, just compensation, and due process. Issues: (1) Whether socialized housing as defined in P.D. 1224, as amended, for the purpose of condemnation proceedings is not public use since it will benefit only a handful of people, bereft of public character, hence it is not a valid exercise of the States power of eminent domain. (2) Whether NHA has the discretion to determine the size of the property/properties to be expropriated. (3) Whether P.D. 1224, as amended, allows unjust and unfairvaluations arbitrarily fixed by government assessors. (4) Whether petitioners were denied due process because their parcels of land were immediately possessed by the NHA by virtue of the writ of possession ordered by the respondent judge.

Held: (1) P.D. 1224 defines socialized housing as, the construction of dwelling units for the middle and lower class members of our society, including the construction of the supporting infrastructure and other facilities. The public use requirement for a valid exercise of the power of eminent domain is a flexible and evolving concept influenced by changing conditions. The taking to be valid must be for public use. As long as the purpose of the taking is public, then the power of eminent domain comes into play. It is accurate to state then that at present, whatever may be beneficially employed for the general welfare satisfies the requirement of public use. Ergo, socialized housing falls within the confines of public use. (2) The State acting through the NHA is vested with broad discretion to designate the particular property/properties to be taken for socialized housing purposes and how much thereof may be expropriated. Absent a clear showing of fraud, bad faith, or gross abuse of discretion, which petitioners failed to demonstrate, the Court will give due weight to and leave undisturbed the NHAs choice and the size of the site for the project. The right to use, enjoyment and disposal of private property is tempered by and has to yield to the demands of the common good. (3) Yes. The provisions on just compensation found in Presidential Decrees No. 1224, 1259, and 1313 are the same provisions found in P.D. No.s 76, 464, 794, and 1533 which were declared unconstitutional for being encroachments on judicial prerogative. Just compensation means the value of the property at the time of the taking. It means a fair and full equivalent for the loss sustained. Tax values can serve as guides but cannot be absolute substitute for just compensation. (4) Yes. The petitioners were denied of due process. P.D. 1224, as amended, violates procedural due process as it allows immediate taking of possession, control and disposition of property without giving the owner his day in court. Respondent Judge ordered the issuance of a writ of possession without notice and without hearing.

Issue: Whether or not the provision of Article 1250 of the New Civil Code is applicable in determining the amount of compensation to be paid to private respondent Amigable for the property taken. Held: Not applicable. Ratio: Article 1250 of the NCC provides that the value of currency at the time of the establishment of the obligation shall be the basis of payment which would be the value of peso at the time of taking of the property when the obligation of the government to pay arises. It is only when there is an agreement that the inflation will make the value of currency at the time of payment, not at the time of the establishment, the basis for payment. The correct amount of compensation would be P14,615.79 at P2.37 per square meter, not P49,459.34, and the interest in the sum of P145,410.44 at the rate of 6% from 1924 up to the time respondent court rendered its decision as was awarded by the said court should accordingly be reduced. Sumulong v Guerrero Facts: On December 5, 1997 the National Housing Authority (NHA) filed a complaint for expropriation of parcels of land for the expansion of Bagong Nayon Hosing Project to provide housing facilities to low-salaried government employees, covering approximately twenty five (25) hectares in Antipolo, Rizal. This included the lots of petitioners Lorenzo Sumulong (6,667 sq.m.) and Emilia Vidanes-Balaoing (3,333 sq.m.).

Familiara v JM Tuason and Co Prior to 1953, the Deudors, with their vendees,1 and the Tuasons were disputing in five cases the ownership and possession of about 50 quiones of land located in Tatalon, Quezon City. On March 16, 1953, the disputing parties entered into a COMPROMISE AGREEMENT2 wherein the Deudors and their vendees recognized the ownership of the Tuasons over the lands in question and for a consideration of P1,201,063.00 to be paid in installments, the former agreed to vacate and deliver unto the Tuasons the said 50 quiones of land. The Court of First Instance of Rizal, Branch IV, sitting at Quezon City, which was trying the five cases, approved on April 10, 1953 the COMPROMISE AGREEMENT, after explaining fully the terms thereof to all the parties. In the same decision it rendered, the Tuasons were declared as absolute owners of the disputed lands. This decision based on said agreement which recognized the absolute ownership of the Tuasons, became final and executory. Somehow, later, not all of the 50 quiones of land were delivered to the Tuasons. At their instance, the trial court on February 28, 1957 gave the Deudors a period of four months within which to clear and deliver peaceful possession of the remaining 30 quiones of land to the Tuasons. The incident was elevated by the Deudors to the Supreme Court3 and We sustained the lower court. Bent on impugning the COMPROMISE AGREEMENT, the Deudors and their vendees on February 16, 1962, filed Civil Case No. 49657 in the Court of First Instance of Manila to annul and rescind the said agreement, alleging fraud on the part of the Tuasons, and to recover from them the 20 quiones of land already delivered. This was dismissed by the lower court, acting on motion of the Tuasons, for improper venue and lack of cause of action, which dismissal We sustained in the ensuing appeal.4 Still undaunted, the Deudors only, filed on March 13, 1964 Civil Case No. 8080 in the Court of First Instance of Pasig5 against the Tuasons to recover the 20 quiones of land, annul the certificates of title of the Tuasons, and to nullify and enjoin the execution of the decision of Branch IV of the Court of First Instance of Quezon City,6 rendered on April 10, 1953.

On January 18, 1965, Branch II of the Court of First Instance of Quezon City presided by Judge Mencias, acting in Civil Cases 36213623, held7 that the land registration proceedings which gave rise to the Tuasons certificates of title were null and void and, accordingly, declared the Tuasons title over the 50 quiones of land null and void also. This is now pending appeal before Us. In less than a month after the promulgation of the decision immediately above referred to, herein respondents Fulgencio and the other Deudor vendees instituted on February 11, 1965, Civil Case No. 8559 in the Court of First Instance of Pasig, to be declared owners of the 50 quiones of land at Tatalon, Quezon City; to annul and enjoin the execution of the decision of Branch IV of the Court of First Instance of Quezon City, rendered on April 10, 1953; to compel the Tuasons to vacate the said 50 quiones and to render an accounting; and to declare the Tuasons titles as null and void. The Tuasons moved for dismissal, alleging,inter alia, lack of jurisdiction of the Court of First Instance of Pasig to annul and enjoin the execution of a final decision rendered by a branch of the Court of First Instance of Quezon City. The lower court denied the motion. Preliminary injunctions were issued8 in favor of several parties who sought to intervene in the proceedings as parties plaintiffs also. On July 8, 1965, the Tuasons came to Us on a petition for certiorari and prohibition with preliminary and mandatory injunction to prevent the Court of First Instance of Pasig from acting in Civil Case No. 8559. We issued the preliminary injunction prayed for. Subsequently, herein intervenors were allowed to join in the proceedings. Petitioners submission that only Branch IV of the Court of First Instance of Quezon City can annul its own decision is well taken. It is settled that the jurisdiction to annul a judgment of a branch of the Court of First Instance belongs solely to the very same branch which rendered the judgment. Any other branch, even if it be the same judicial district like those of the Courts of First Instance of Rizal, sitting at Pasig and at Quezon City, which belong to the 7th Judicial District that attempts to do so either exceeds its jurisdiction, as We held in Cabigao v. Del Rosario, 44 Phil. 182, or acts with grave abuse of discretion amounting to lack of jurisdiction, as We ruled in P.N.B. v. Javellana, 92 Phil. 525. In either case, certiorariand prohibition would be

proper to prevent the attempting branch of the court from proceeding to nullify a final decision rendered by a co-equal and coordinate branch. The two cases cited have only recently been reaffirmed by Us in Mas v. Dumaraog, L-16252, Sept. 29, 1964. Withal, We see no need for Civil Case No. 8559 to pend further before the lower court. Respondents bases for seeking to nullify the decision of the Court of First Instance of Quezon City are: (1) The compromise agreement on which it was based had already been declared totally rescinded, and (2) the titles of the Tuasons, on which the compromise agreement was based, are null and void. We have already definitely settled, however, in the two cases involving the Tuasons and the Deudors9 that only the obligation of the Tuasons to make further payments as originally contemplated in the COMPROMISE AGREEMENT had been rescinded; the rest of the said agreement is still valid and binding between the Tuasons, on the one hand and the Deudors and their vendees herein respondents on the other.10 As for the alleged nullity of the Tuasons titles the validity of which should be resolved in the proper court in appropriate proceedings, suffice it to say that the basis of the decision of April 10, 1953 is not really the titles of the Tuasons but the COMPROMISE AGREEMENT wherein the Deudors and herein respondents acknowledged the complete, absolute and indefeasible titles of the Tuasons. WHEREFORE, finding it meritorious, the petition for certiorari and prohibition is hereby, granted. The Court of First Instance of Rizal, sitting at Pasig, is ordered to permanently desist from taking cognizance of, and acting in, its Civil Case No. 8559. The preliminary injunction heretofore issued is made permanent. No costs. So ordered. Manotok v Ntnl Housing Authority Manotok v. NHA 150 SCRA 89 (1987) F: Petitioners are the owners of two large estates known as the Tambunting Estate and Sunog-Apog in Tondo, Manila, both of which were declared expropriated in two decrees issued by President Marcos, PD 1669 and PD 1670. The petitioners contend that the decrees violate their constitutional right to due process and equal protection since by their mere passage their properties were automatically expropriated and they were immediately deprived of the ownership and possession thereof without being given the chance to oppose such expropriation.

The government on the other hand contends that the power of eminent domain is inherent in the State and when the legislature or the President through his lawmaking powers exercises this power, the public use and public necessity of the expropriation and the fixing of the just compensation become political in nature and the courts must respect the decision. HELD: The challenged decrees are unfair in the procedures adopted and the powers given to the NHA. The Tambunting subdivision is summarily proclaimed a blighted area and directly expropriated by decree without the slightest semblance of a hearing or any proceeding whatsoever. The expropriation is instant and automatic to take effect immediately upon the signing of the decree. No deposit before the taking is required. There is not provision for any interest to be paid upon unpaid installments. Not only are the owners given absolutely no opportunity to contest the expropriation, or question the amount of payments fixed by the decree, but the decision of the NHA are expressly declared beyond judicial review. PD 1669 and 1670 are declared unconstitutional. Teehankee, CJ, concurring: The judgment at bar now learly overturns the majority ruling in JM Tuason v. LTA that the power of Congress to designate the particular property to be taken adn how much may be condemned thereof must be duly recognized, leaving only as a judicial question whether in the exercise of such competence, the party adversely affected is the victim of partiality and prejudice. The SC now rules that such singling out of properties does not foreclose judicial scrutiny as to whether such expropriation by legislative act transgresses the due process and equal protection and just compensation guarantees of the Constitution. Pulido v CA Bienvenido Agapito owned a 230-square meter parcel of unregistered residential land in Indang, Cavite, which he mortgaged with the Rural Bank of Cavite City (RBCC) to secure a loan. 1 On 23 May 1973, upon his failure on maturity to pay the loan, the mortgage was foreclosed and sold at public auction to RBCC as the highest bidder. He failed to redeem his property within the one-year reglementary period despite repeated extensions granted him. On 26 July 1980 RBCC through its PresidentManager Leonarda G. Alegre sold the

property together with its improvements to spouses Juan and Filomena Pulido 2 who on 1 September 1980 sold the same in turn to spouses Alberto and Norma Gloria. 3 Contending that he was not notified of the foreclosure sale Bienvenido Agapito filed a complaint in the Regional Trial Court of Cavite against RBCC, the spouses Juan and Filomena Pulido and the spouses Alberto and Norma Gloria 4 seeking the annulment of the extrajudicial foreclosure and sale with damages, and praying that he be allowed to redeem the property by paying with interest his mortgage debt. He also claimed that the public auction sale conducted by the Provincial Sheriff was null and void for noncompliance with the posting of notice requirement under P.D. No. 122. To support his claim Agapito presented Valeriano Fajardo, then Postmaster of Indang, Cavite, who testified that he was not aware of any notice of foreclosure sale posted in the municipal building. On 5 August 1982 the court on motion of plaintiff allowed the filing of a second amended complaint to include the value of certain personal properties estimated at P75,000.00 which were allegedly taken by the Pulido spouses and then by the Gloria spouses when they respectively took possession of the foreclosed property. The trial court sustained the validity of the auction sale and the subsequent transfer to the Pulidos and then to the Glorias 5 holding that the requirements for the validity of the questioned foreclosure proceedings had been complied with by the Provincial Sheriff and RBCC. His motion for reconsideration having been denied, 6 Agapito appealed to the Court of Appeals which reversed the court a quo and holding that no evidence existed to show that the notice of foreclosure sale was posted as required by law. The appellate court observed that the "Notice of Extrajudicial Sale of Mortgaged Properties" presented by RBCC did not prove that said notice, and at least two other copies thereof, had been actually posted, and that the "Minutes of Sheriff Public Auction Sale" 7 could not be considered as evidence since it was not formally offered in court. Thus finding no clear and preponderant evidence to show compliance with the requisite posting of notice, the Court of

Appeals declared the foreclosure sale as null and void, labeling the non-compliance as a jurisdictional defect which invalidated the sale. Consequently, it declared the deeds of sale of 26 July 1980 and 1 September 1980 8 void and without force and effect, ordered the Pulido spouses and the Gloria spouses to pay their respective vendors the amount corresponding to the purchase price with legal interest from the date of finality of the decision. The appellate court also granted Agapito a non-extendible period of sixty (60) days from the finality of the decision within which to exercise the right of repurchase by paying RBCC the amount of the mortgage indebtedness with 12% interest. 9 Hence this recourse by the spouses Juan and Filomena Pulido and the spouses Alberto and Norma Gloria. RBCC did not appeal. We deny the petition and affirm the Court of Appeals. Petitioners contend that respondent Court of Appeals erred when it concluded that the requirement of posting had not been complied with merely on the basis that the evidence presented by them, as defendants in the trial court, to show compliance was not sufficient. They insist that Agapito, as the plaintiff alleging non-compliance, had the burden of proof and that it was on the sufficiency of the evidence presented by him that the merits ought to have been decided: While it may be true that the party alleging non-compliance with the requisite publication has the burden of proof, still negative allegations need not be proved even if essential to one's cause of action or defense if they constitute a denial of the existence of a document the custody of which belongs to the other party. 10 Section 5, R.A. No. 720, as amended by Sec. 2 of P.D. No. 122, specifically requires that proof of the required posting shall consist of the affidavit of the sheriff or officer conducting the foreclosure sale which shall be attached with the records of the case. 11 Hence, it is a simple matter for RBCC, petitioners' predecessor-in-interest, to rebut the allegation of non-compliance by producing the required proof of posting, i.e., the affidavit of the sheriff who conducted the sale. But no such affidavit was

presented. Neither was there any equally competent and convincing proof offered to show compliance. As respondent Court of Appeals correctly concluded, the "Notice of Extrajudicial Sale of Mortgaged Properties" does not prove that said notice was actually posted as required by law, while the "Minutes of Sheriff Public Auction Sale" cannot be appreciated in favor of petitioners because it was not presented, much less formally offered as evidence. It is axiomatic that a decision of a lower court cannot be reversed for its failure to consider evidence which was not even presented by the parties. 12 Neither was the sheriff who conducted the sale presented as a witness, no reason having been given for such non-presentation. Hence, since the required proof of posting was not presented and no other competent and convincing proof was offered in its stead, petitioners' reliance on the presumption of regularity in the performance of official duties falls in the face of a serious imputation on noncompliance. The presumption of compliance with official duty is rebutted by failure to present proof of posting. 13 While the trial court concluded that the requirements for the validity of the foreclosure proceedings had been complied with, still we cannot apply the rule that factual findings of the trial court are conclusive on us since a reading alone of the decision of the trial court shows that its finding is not factual but merely a conclusion of law with nothing to lean on. It is further argued by petitioners that even assuming that the requirements for a valid extrajudicial foreclosure of mortgage have not been met the title that passed on to petitioners was cured of any and all defects since they merely relied on the registered deeds of sale of their respective transferors (RBCC in the case of spouses Pulido and the latter as regards the spouses Gloria) which appeared in order and was not in any way suspicious. Plainly, petitioners claim they are entitled to protection under the law as buyers in good faith and for value. This defense is unavailing. It must be remembered that the property involved is an unregistered land. When RBCC acquired it at foreclosure it registered its title under Act No. 3344 14 which specifically provides that "[a]ny registration made . . . shall be

understood to be without prejudice to a third party with a better right." 15 In other words, registration under said Act, unlike under Act No. 496, 16 does not afford full protection as it must yield to aprior and valid title, even if unregistered. Therefore, even if petitioners relied on the registered title of their predecessor-in-interest (RBCC) they cannot capitalize on their being innocent purchasers for value and in good faith over and above Agapito's valid and prior right to have his property foreclosed in accordance with law. The rule that the purchaser is not required to explore further than what the record in the Registry indicates on its face in quest of any hidden defect or inchoate right which may subsequently defeat his right thereto refers only to properties registered under the Torrens system, not to those under Act No. 3344. 17 Therefore, we agree with respondent court when it declared as null and void the deeds of sale of 26 July 1980 and 1 September 1980 evidencing the transfer of the controverted property to the Pulido spouses and then to the Gloria spouses. However, we cannot logically agree with the portion of the appealed decision directing petitioners to pay theirvendors the amount corresponding to the purchase price with legal interest from the date of finality of the decision; 18 rather, it should be the other way around. The Pulidos should return to the Glorias P25,900.00 representing the amount which the latter paid by virtue of the deed of sale dated 1 September 1980. The Pulido spouses, in turn, can demand from RBCC the P25,900.00 which they paid to the latter when they purchased the foreclosed property on 26 July 1980. Considering that private respondent Bienvenido Agapito had been given by RBCC several opportunities to buy back his property before foreclosure but which he merely ignored, we delete the portion of the decision appealed from awarding him an inextendible period of sixty (60) days within which to repurchase the controverted property. WHEREFORE, the decision of the Court of Appeals of 24 February 1993 declaring the extrajudicial foreclosure of mortgage and auction sale held on 23 May 1973 void ab initio for failure to comply with Sec. 5, R.A. No. 720, as amended by Sec. 2, P.D. No. 122, is AFFIRMED without prejudice to another foreclosure sale conducted faithfully in compliance with the law.

Accordingly, the spouses Alberto and Norma Gloria, the current possessors of the disputed property, are directed to return possession thereof to the mortgage creditor, the Rural Bank of Cavite City, and the latter to return to the spouses Juan and Filomena Pulido the amount of P25,900.00 representing the purchase price they paid by virtue of the deed of sale of 26 July 1980 which is declared null and void, with legal interest from the date of finality of this decision. The spouses Juan and Filomena Pulido, in turn, are also directed to return to petitioner spouses Alberto and Norma Gloria the amount of P25,900.00 which the latter paid to them by virtue of the Deed of Sale of 1 September 1980 which is likewise declared void ab initio, with legal interest from the finality of this decision. To this extent the judgment appealed from is modified. Heirs of Juancho Ardona v Reyes Heirs of Juancho Ardona v. Reyes 123 SCRA 220 F: The Philippine Tourism Authority sought the expropriation of 282 Ha of land in Barangay Malubog and Babag in Cebu City. upon deposit of an amount equivalent to 10% of the value of the property, the CFI authorized the PTA to take immediate possession of the property. The charter of the PTA authorizes it to acquire through condemnation proceedings lands for tourist zone development of a sports complex. The petitioners who are occupants of the lands, filed a petition for certiorari in the SC. They contended that (1) the taking was not for public use; (2) the land was covered by the land reform program; and (3) expropriation would impair the obligation of contracts. HELD: The concept of public use is not limited to traditional purposes for the construction of roads, bridges, and the like. The idea that "public use" means "use by the public" has been discarded. As long as the purpose of the taking is public, then the power of eminent domain comes into play. It is accurate to state then that at present whatever may be beneficially employed for the general welfare satisfies the requirement of public use. The petititioners have not shown that the area being developed is land reform area and that the affected persons have been given emancipation patents and certificates of land transfer. The contract clause has never been regarded as a barrier to the exercise of the police power and likewise eminent domain.

Assoc. of Small Landowners in the Phils v Sec. of Agrarian Reform These are 3 cases consolidated questioning the constitutionality of the Agrarian Reform Act. Article XIII on Social Justice and Human Rights includes a call for the adoption by the State of an agrarian reform program. The State shall, by law, undertake an agrarian reform program founded on the right of farmers and regular farmworkers, who are landless, to own directly or collectively the lands they till or, in the case of other farmworkers, to receive a just share of the fruits thereof. RA 3844, Agricultural Land Reform Code, had already been enacted by Congress on August 8, 1963. This was substantially superseded almost a decade later by PD 27, which was promulgated on Oct 21, 1972, along with martial law, to provide for the compulsory acquisition of private lands for distribution among tenantfarmers and to specify maximum retention limits for landowners. On July 17, 1987, Cory issued EO 228, declaring full land ownership in favor of the beneficiaries of PD 27 and providing for the valuation of still unvalued lands covered by the decree as well as the manner of their payment. This was followed on July 22, 1987 by PP 131, instituting a comprehensive agrarian reform program (CARP), and EO 229, providing the mechanics for its implementation. Afterwhich is the enactment of RA 6657, Comprehensive Agrarian Reform Law of 1988, which Cory signed on June 10. This law, while considerably changing the earlier mentioned enactments, nevertheless gives them suppletory effect insofar as they are not inconsistent with its provisions. In considering the rentals as advance payment on the land, the executive order also deprives the petitioners of their property rights as protected by due process. The equal protection clause is also violated because the order places the burden of solving the agrarian problems on the owners only of agricultural lands. No similar obligation is imposed on the owners of other properties. The petitioners maintain that in declaring the beneficiaries under PD 27 to be the owners of the lands occupied by them, EO 228 ignored judicial prerogatives and so violated due process. Worse, the measure would not solve the agrarian problem because even the small farmers are deprived of their lands and the retention rights guaranteed by the Constitution.

In his comment the Sol-Gen asserted that the alleged violation of the equal protection clause, the sugar planters have failed to show that they belong to a different class and should be differently treated. The Comment also suggests the possibility of Congress first distributing public agricultural lands and scheduling the expropriation of private agricultural lands later. From this viewpoint, the petition for prohibition would be premature. ISSUE: Whether or not there was a violation of the equal protection clause. HELD: The SC ruled affirming the Sol-Gen. The argument of the small farmers that they have been denied equal protection because of the absence of retention limits has also become academic under Sec 6 of RA 6657. Significantly, they too have not questioned the area of such limits. There is also the complaint that they should not be made to share the burden of agrarian reform, an objection also made by the sugar planters on the ground that they belong to a particular class with particular interests of their own. However, no evidence has been submitted to the Court that the requisites of a valid classification have been violated. Classification has been defined as the grouping of persons or things similar to each other in certain particulars and different from each other in these same particulars. To be valid, it must conform to the following requirements: (1) it must be based on substantial distinctions; (2) it must be germane to the purposes of the law; (3) it must not be limited to existing conditions only; and (4) it must apply equally to all the members of the class. The Court finds that all these requisites have been met by the measures here challenged as arbitrary and discriminatory. Equal protection simply means that all persons or things similarly situated must be treated alike both as to the rights conferred and the liabilities imposed. The petitioners have not shown that they belong to a different class and entitled to a different treatment. The argument that not only landowners but also owners of other properties must be made to share the burden of implementing land reform must be rejected. There is a substantial distinction between these two classes of owners that is clearly visible except to those

who will not see. There is no need to elaborate on this matter. In any event, the Congress is allowed a wide leeway in providing for a valid classification. Its decision is accorded recognition and respect by the courts of justice except only where its discretion is abused to the detriment of the Bill of Rights. Telecommunications & Broadcast Attys of the Phils Inc v COMELEC Facts: Petitioner Telecommunications and Broadcast Attorneys of thePhilippines, Inc. (TELEBAP) is an organization of lawyers of radio and television broadcasting companies. It was declared to be without legal standing to sue in this case as, among other reasons, it was not able to show that it was to suffer from actual or threatened injury as a result of the subject law. Petitioner GMA Network, on the other hand, had the requisite standing to bring the constitutional challenge. Petitioner operates radio and television broadcast stations in the Philippinesaffected by the enforcement of Section 92, B.P. No. 881. Petitioners challenge the validity of Section 92, B.P. No. 881 which provides: Comelec Time- The Commission shall procure radio and television time to be known as the Comelec Time which shall be allocated equally and impartially among the candidates within the area of coverage of all radio and television stations. For this purpose, the franchise of all radio broadcasting and television stations are hereby amended so as to provide radio or television time, free of charge, during the period of campaign. Petitioner contends that while Section 90 of the same law requires COMELEC to procure print space in newspapers and magazines withpayment, Section 92 provides that air time shall be procured by COMELEC free of charge. Thus it contends that Section 92 singles out radio and television stations to provide free air time. Petitioner claims that it suffered losses running to several million pesos in providing COMELEC Time in connection with the 1992 presidential election and 1995 senatorial election and that it stands to suffer even more should it be required to do so again this year. Petitioners claim that the primary source of revenue of the radio and television stations is the sale of air time to advertisers

and to require these stations to provide free air time is to authorize unjust taking of private property. According to petitioners, in 1992 it lost P22,498,560.00 in providing free air time for one hour each day and, in this years elections, it stands to lost P58,980,850.00 in view of COMELECs requirement that it provide at least 30 minutes of prime time daily for such. Issues: (1) Whether of not Section 92 of B.P. No. 881 denies radio andtelevision broadcast companies the equal protection of the laws. (2) Whether or not Section 92 of B.P. No. 881 constitutes taking of property without due process of law and without just compensation. Held: Petitioners argument is without merit. All broadcasting, whether radio or by television stations, is licensed by the government. Airwave frequencies have to be allocated as there are more individuals who want to broadcast that there are frequencies to assign. Radio and television broadcasting companies, which are given franchises, do not own the airwaves and frequencies through which they transmitbroadcast signals and images. They are merely given the temporary privilege to use them. Thus, such exercise of the privilege may reasonably be burdened with the performance by the grantee of some form of public service. In granting the privilege to operate broadcaststations and supervising radio and television stations, the state spends considerable public funds in licensing and supervising them. The argument that the subject law singles out radio and television stations to provide free air time as against newspapers and magazines which require payment of just compensation for the print space they may provide is likewise without merit. Regulation of the broadcastindustry requires spending of public funds which it does not do in the case of print media. To require the broadcast industry to provide free air time for COMELEC is a fair exchange for what the industry gets. As radio and television broadcast stations do not own the airwaves, no private property is

taken by the requirement that they provide air time to the COMELEC. De Knecht v Bautista De Knecht v. Bautista 100 SCRA 660 (1980) F: The plan to extend EDSA to Roxas Boulevard to be ultimately linked to the Cavite Coastal Road Project, originally called for the expropriation of properties along Cuneta Avenue in Pasay City. Later on, however, the Ministry of Public Highways decided to make the proposed extension pass through Fernando Rein and Del Pan Streets. Because of the protests of residents of the latter, the Commission on Human Settlements recommended the reversion to the original plan, but the Ministry argued the new route withh save the government P2 million. The government filed expropriation proceedings against the owners of Fernando Rein and Del Pan streets, among whom was petitioner. HELD: The choice of Fernando Rein and Del Pan streets is arbitrayr and should not receive judicial aprpoval. The Human Settlements Commission concluded that the cost factor is so minimal that it can be disregarded in making a choice between the two lines. The factor of functionality strongly militates against the choice of Fernando Rein and Del Pan streets, while the factor of social and economic impact bears grievously on the residents of Cuneta Avenue. While the issue would seem to boil down to a choice between people, on one hand, and progress and development, on the other, it is to be remembered that progress and development are carried out for the benefit of the people. Republic v De Knecht Republic v. De Knecht, 182 SCRA 142 (1990) F: De Knecht was one of the owners of several properties along the Fernando Rein-Del Pan streets which the Government sought to expropriate to give way to the extension of EDSA and the construction of drainage facilities. De Knecht filed a case to restrain the Government from proceeding with the expropriation. Her prayer was denied by the lower court but upon certiorari, the SC reversed the lower court decision and granted the relief asked for by De Knecht ruling that the expropriation was arbitrary. The case was remanded to the lower court. No further action was taken despite the SC decision until two years later, in 1983, when the Government moved for the dismissal of the case on the ground that the Legislature has since enacted BP 340 expropriating the same properties for the

same purpose. The lower court denied tthe motion. Appeal. RULING: While it is true that said final judgment of this Curt on the subject becomes the law of the case between the parties, it is equally true that the right of petitioner to take private properties for public use upon payment of just compensation is so provided in the Constitution and the laws. Such expropriation proceeding may be undertaken by the petitioner not only by voluntary negotiation with the land owners but also by taking appropriate court action or by legislation. When BP 340 was passed, it appears that it was based on supervening events that occured after the 1980 decision of the SC on the De Knecht case was rendered. The social impact factor which persuaded the Court to consider this extension to be arbitrary had disappeared. Moreover, the said decision is no obstacle to the legislative arm of the Government in thereafter making its own independent assessment of the circumstances then pravailing as to the propriety of undertaking the expropriation of properties in question and thereafter by enacting the corresponding legislation as it did in this case. The Court agrees in the wisdom and necessity of enacting BP 340. Thus the anterior decision of the Court must yield to the subsequent legislative fiat

premises, and thereafter, prepare their appraisals as to the fair and just compensation to be paid to the owners of the lots. The lower court rendered judgement ordered Napocor to pay defendant spouses the sum of P10.00 per square meter as the fair and reasonable compensation for the right-of-way easement of the affected area and P800.00 as attorney's fees'. Napocor filed a motion for reconsideration contending that the Court of Appeals committed gross error by adjudging the petitioner liable for the payment of the full market value of the land traversed by its transmission lines, and that it overlooks the undeniable fact that a simple right-of-way easemen transmits no rights, except that of the easement. ISSUE: Whether or not petitioner should be made to pay simple easement fee or full compensation for the land traversed by its transmission lines. RULING: In RP v. PLDT, the SC ruled that "Normally, the power of eminent domain results in the taking or appropriation of the title to, and possession of, the expropriated property, but no cogent reason appears why said power may not be availed of to impose only a burden upon the owner of the condemned property, without loss of title or possession. It is unquestionable that real property may, through expropriation, be subjected to an easement of right of way." In this case, the easement is definitely a taking under the power of eminent domain. Considering the nature and effect of the installation of the transmission lines, the limitations imposed by the NPC against the use of the land (that no plant higher than 3 meters is allowed below the lines) for an indefinite period deprives private respondents of its ordinary use. For these reasons, the owner of the property expropriated is entitled to a just compensation which should neither be more nor less, whenever it is possible to make the assessment, than the money equivalent of said property. Just equivalent has always been understood to be the just and complete equivalent of the loss which the owner of the thing expropriated has to suffer by reason of the expropriation. The price or value of the land and its character at the time of taking by the Govt. are the criteria for determining just compensation.

NAPOCOR v. Gutierrez, 193 SCRA 1 (1991) Plaintiff National Power Corporation (Napocor), for the construction of its 230 KV Mexico-Limay transmission lines, its lines have to pass the lands belonging to respondents Matias Cruz, heirs of Natalie Paule and spouses Misericordia Gutierrez and Recardo Malit. Unsuccessful with its negotiations for the acquisition of the right of way easements, Napocor was constrained to file eminent domain proceedings. Trial courts ordered that the defendant spouses were authorized to withdraw the fixed provisional value of their land in the sum of P973.00 deposited by the plaintiff to cover the provisional value of the land to proceed their construction and for the purpose of determining the fair and just compensation due the defendants, the court appointed three commissioners, comprised of one representative of the plaintiff, one for the defendants and the other from the court, who then were empowered to receive evidence, conduct ocular inspection of the

*************TAXATION Punsalan v Municipal Board of Mla Facts: Petitioners, who are professionals in the city, assail Ordinance No. 3398 together with the law authorizing it (Section 18 of the Revised Charter of the City of Manila). The ordinance imposes a municipal occupation tax on persons exercising various professions in the city and penalizes nonpayment of the same. The law authorizing said ordinance empowers the Municipal Board of the city to impose a municipal occupation tax on persons engaged in various professions. Petitioners, having already paid their occupation tax under section 201 of theNational Internal Revenue Code, paid the tax under protest as imposed by Ordinance No. 3398. The lower court declared the ordinance invalid and affirmed the validity of the law authorizing it. Issue: Whether or Not the ordinance and law authorizing it constitute class legislation, and authorize what amounts to double taxation. Held: The Legislature may, in its discretion, select what occupations shall be taxed, and in its discretion may tax all, or select classes of occupation for taxation, and leave others untaxed. It is not for the courts to judge which cities or municipalities should be empowered to impose occupation taxes aside from that imposed by the NationalGovernment. That matter is within the domain of political departments. The argument against double taxation may not be invoked if one tax is imposed by the state and the other is imposed by the city. It is widely recognized that there is nothing inherently terrible in the requirement that taxes be exacted with respect to the same occupation by both the state and the political subdivisions thereof. Judgment of the lower court is reversed with regards to the ordinance and affirmed as to the law authorizing it.

Lladoc v CIR Facts: Sometime in 1957, M.B. Estate Inc., of Bacolod City, donated 10,000.00 pesos in cash to Fr. Crispin Ruiz, the parish priest of Victorias, Negros Occidental, and predecessor of Fr. Lladoc, for the construction of a new Catholic church in the locality. The donated amount was spent for such purpose. On March 3, 1958, the donor M.B. Estate filed the donor's gift tax return. Under date of April 29, 1960. Commissioner of Internal Revenueissued an assessment for the donee's gift tax against the Catholic Parishof Victorias of which petitioner was the parish priest. Issue: Whether or not the imposition of gift tax despite the fact the Fr. Lladoc was not the Parish priest at the time of donation, CatholicParish priest of Victorias did not have juridical personality as the constitutional exemption for religious purpose is valid. Held: Yes, imposition of the gift tax was valid, under Section 22(3) Article VI of the Constitution contemplates exemption only from payment of taxes assessed on such properties as Property taxes contra distinguished from Excise taxes The imposition of the gift tax on the property used for religious purpose is not a violation of the Constitution. A gift tax is not a property by way of gift inter vivos. The head of the Diocese and not the parish priest is the real party in interest in the imposition of the donee's tax on the property donated to the church for religious purpose.

*******DUE PROCESS Villegas v Hiu Chong Tsai Pao Ho Villegas v. Hiu Chiung Tsai Pao Ho 86 SCRA 270 (1978) F: An ordinance of the City of Manila prohibited the employment of aliens in any occupation or business unless they first secured a permit from the Mayor of Manila and paid a fee of P500. Respondent, an alien, employed in Manila, brought suit and obtained judgment from the CFI declaring the ordinance null and void. HELD: The ordinance is a tax measure. In imposing a flat rate of P500, it failed to consider substantial differences in situations among aliens and for that reason violates the rule on uniformity of taxation. It also lays down no guide for granting/denying the permit and therefore permits the arbitrary exercise of discretion by the Mayor. Finally, the ordinance denies aliens due process and the equal protection of the laws Kwong Sing v City of Mla Facts: Kwong Sing, in his own behalf and of other Chineselaundrymen who has general and the same interest, filed a complaintfor a preliminary injunction. The Plaintiffs also questioned the validity of enforcing Ordinance No. 532 by the city of Manila. Ordinance No. 532 requires that the receipt be in duplicate in English and Spanish duly signed showing the kind and number of articles delivered by laundries and dyeing and cleaning establishments. The permanent injunction was denied by the trial court. The appellants claim is that Ordinance No. 532 savors of class legislation; putting in mind that they are Chinesenationals. It unjustly discriminates between persons in similar circumstances; and that it constitutes an arbitrary infringement of property rights. They also contest that the enforcement of the legislation is an act beyond the scope of their police power. In view of the foregoing, this is an appeal with the Supreme Court. Issues: (1) Whether or Not the enforcement of Ordinance no, 532 is an act beyond the scope of police power (2) Whether or Not the enforcement of the same is a class legislation that infringes property rights. Held: Reasonable restraints of a lawful business for such purposes are permissible under the police power. The police power of the City of Manila to enact Ordinance No.

532 is based on Section 2444, paragraphs (l) and (ee) of the Administrative Code, as amended by Act No. 2744, authorizes the municipal board of the city of Manila, with the approval of the mayor of the city: (l) To regulate and fix the amount of the license fees for the following: xxxx xxxxxlaundries xxxx. (ee) To enact all ordinances it may deem necessary and proper for thesanitation and safety, the furtherance of the prosperity, and the promotion of the morality, peace, good order, comfort, convenience, and general welfare of the city and its inhabitants. The court held that the obvious purpose of Ordinance No. 532 was to avoid disputes between laundrymen and their patrons and to protect customers of laundries who are not able to decipher Chinese charactersfrom being defrauded. (Considering that in the year 1920s, people of Manila are more familiar with Spanish and maybe English.) In whether the ordinance is class legislation, the court held that the ordinance invades no fundamental right, and impairs no personal privilege. Under the guise of police regulation, an attempt is not made to violate personal property rights. The ordinance is neither discriminatory nor unreasonable in its operation. It applies to all public laundries without distinction, whether they belong to Americans, Filipinos, Chinese, or any other nationality. All, without exception, and each every one of them without distinction, must comply with the ordinance. The obvious objection for the implementation of the ordinance is based in sec2444 (ee) of
the Administrative Code. Although, an additional burden will be imposed on the business and occupation affected by the ordinance such as that of the appellant bylearning even a few words in Spanish or English, but mostly Arabicnumbers in order to properly issue a receipt, it seems that the same burdens are cast upon the them. Yet, even if private rights of person or property are subjected to restraint, and even if loss will result to individuals from the enforcement of the ordinance, this is not sufficient ground for failing to uphold the power of the legislative body. The very foundation of the police power is the control of private interests for the public welfare. Finding that the ordinance is valid, judgment is affirmed, and the petition for a preliminary injunction is denied, with costs against the appellants.

Yu Cong Eng v Trinidad Facts: The petitioner, Yu Cong Eng, was charged by information in the court of first instance of Manila, with a violation of Act 2972, which provides that (Section 1) it shall be unlawful for any person, company, or partnership or corporation engaged in commerce, industry or any other activity for the purpose of profit in the Philippine Islands, in accordance with existing law, to keep its account books in any language other than English, Spanish or any local dialect. He was arrested, his books were seized, and the trial was about to proceed, when he and the other petitioner, Co Liam, on their own behalf, and on behalf of all the other Chinese merchants in the Philippines, filed the petition against the fiscal, or prosecuting attorney of Manila, and the collector of internal revenue engaged in the prosecution, and against the judge presiding. Issue: Whether or Not Act 2972 is unconstitutional. Held: Yes. The Philippine government may make every reasonable requirement of its taxpayers to keep proper records of their business transactions in English or Spanish or Filipino dialect by which an adequate measure of what is due from them in meeting the cost ofgovernment can be had. But we are clearly of opinion that it is not within the police power of the Philippine Legislature, because it would be oppressive and arbitrary, to prohibit all Chinese merchants from maintaining a set of books in the Chinese language, and in the Chinese characters, and thus prevent them from keeping advised of the status of their business and directing its conduct. Ichong v Hernandez Facts: Republic Act 1180 or commonly known as An Act to Regulate the Retail Business was passed. The said law provides for a prohibitionagainst foreigners as well as corporations owned by foreigners from engaging from retail trade in our country. This was protested by the petitioner in this case. According to him, the said law violates theinternational and treaty of the Philippines therefore it is unconstitutional. Specifically, the Treaty of Amity between the Philippines and China was violated according to him. Issue: Whether or Not Republic Act 1180 is a valid exercise of police power.

Held: According to the Court, RA 1180 is a valid exercise of police power. It was also then provided that police power can not be bargained away through the medium of a treaty or a contract. The Court also provided that RA 1180 was enacted to remedy a real and actual danger to national economy posed by alien dominance and control. If ever the law infringes upon the said treaty, the latter is always subject to qualification or amendment by a subsequent law and the same may never curtain or restrict the scope of the police power of the state. Banco Espaol v Palanca Engracio Palanca was indebted to El Banco and he had his parcel of land as security to his debt. His debt amounted to P218,294.10. His property is worth 75k more than what he owe. Due to the failure of Engracio to make his payments, El Banco executed an instrument to mortgage Engracios property. Engracio however left for China and he never returned til he died. Since Engracio is a non resident El Banco has to notify Engracio about their intent to sue him by means of publication using anewspaper. The lower court further orderdd the clerk of court to furnish Engracio a copy and that itd be sent to Amoy, China. The court eventually granted El Banco petition to execute Engracios property. 7 years thereafter, Vicente surfaced on behalf of Engracio as his administrator to petition for the annulment of the ruling. Vicente averred that there had been no due process as Engracio never received the summons. ISSUE: Whether or not due process was not observed. HELD: The SC ruled against Palanca. The SC ruled that the requisites for judicial due process had been met. The requisites are; 1. There must be an impartial court or tribunal clothed with judicial power to hear and decide the matter before it. 2. Jurisdiction must be lawfully acquired over the person of the defendant or over the property subject of the proceedings. 3. The defendant must be given the opportunity to be heard. 4. Judgment must be rendered only after lawful hearing.

PP v Beriales Facts: A case of three men who were charged for the murder of Saturnina on Sept. 13, 1974. During the hearing on Nov. 26, 1974, upon motion of the defense the Court ordered the re-investigation of the case pending submission of the Fiscal of its reports. Couple of postponements was made until Dec. 13, 1974 hearing when the Court proceeded with the arraignment and trial in the absence of the Fiscal and its report on re-investigation, and over the disagreement of the defense. The CFI of Leyte relied on the private prosecutor being authorized by the Fiscal to present evidence and the defense presumed to have waived its right over its disagreement. Trial then proceeded and the 3 found guilty of he offense. Thus, this appeal on the constitutional requirement of due process. Issue: Whether or not due process of law had been observed. Held: Constitutional due process was violated, thus, case remanded to CFI for arraignment and trial. Court should have held in abeyance the trial while the report on e-investigation was still pending. Consistent disregard of the defense objection on the arraignment, trial, presentation of private prosecutors evidence, and rendition of judgment violates due process. Prosecutor or Fiscal entrusted with the investigation is duty bound to take charge until final termination. They shall have direction and control of the criminal prosecution over private prosecutors. Ang Tibay v Court of Industrial Relations TeodoroToribio owns and operates Ang Tibay a leather company which supplies the Philippine Army. Due to alleged shortage of leather, Toribio caused the lay off of members of National Labor Union Inc. NLU averred that Toribios act is not valid as it is not within the CBA. That there are two labor unions in Ang Tibay; NLU and National Workers Brotherhood. That NWB is dominated by Toribio hence he favors it over NLU. That NLU wishes for a new trial as they were able to come up with new evidence/documents that they were not able to obtain before as they were inaccessible and they were not able to present it before in the CIR. ISSUE: Whether or not there has been a due process of law.

HELD: The SC ruled that there should be a new trial in favor of NLU. The SC ruled that all administrative bodies cannot ignore or disregard the fundamental and essential requirements of due process. They are; (1) The right to a hearing which includes the right of the party interested or affected to present his own case and submit evidence in support thereof. (2) Not only must the party be given an opportunity to present his case and to adduce evidence tending to establish the rights which he asserts but the tribunal must consider the evidence presented. (3) While the duty to deliberate does not impose the obligation to decide right, it does imply a necessity which cannot be disregarded, namely, that of having something to support its decision. A decision with absolutely nothing to support it is a nullity, a place when directly attached. (4) Not only must there be some evidence to support a finding or conclusion but the evidence must be substantial. Substantial evidence is more than a mere scintilla It means such relevant evidence as a reasonable mind might accept as adequate to support a conclusion. (5) The decision must be rendered on the evidence presented at the hearing, or at least contained in the record and disclosed to the parties affected. (6) The Court of Industrial Relations or any of its judges, therefore, must act on its or his own independent consideration of the law and facts of the controversy, and not simply accept the views of a subordinate in arriving at a decision. (7) The Court of Industrial Relations should, in all controversial questions, render its decision in such a manner that the parties to the proceeding can know the vario issues involved, and the reasons for the decisions rendered. The performance of this duty is inseparable from the authority conferred upon it.

Taada v Tuvera Facts: Invoking the peoples right to be informed on matters of public concern, a right recognized in Section 6, Article IV of the 1973 constitution, petitioners seek a writ of mandamus to compel respondent public officials to publish, and/or cause the publication in the Official Gazette, of various presidential decrees, letters of instructions, general orders, proclamations, executive orders, letter of implementation and administrative orders. The respondents would have this case dismissed on the ground that petitioners have no legal personality to bring this petition. Petitioners maintain that since the subject of the petition concerns a public right and its object is to compel public duty, they need not show any specific interest. Respondents further contend that publication in the OG is not a sine qua non requirement for the effectivity of laws where the laws themselves provide for their own effectivity dates. Issue: WON publication in the Official Gazatte is an indispensable requirement for the effectivity of the PDs, LOIs, general orders, EOs, etc. where laws themselves provide for their own effectivity dates. Held: Yes. It is the peoples right to be informed on matters of public concern & corollarily access to official records, & to documents & papers pertaining to official acts, transactions, or decisions, shall be afforded the citizens subject to such limitation as may be provided by law (6 AIV, 1973 Constitution). Laws, to be valid & enforceable, must be published in the OG or otherwise effectively promulgated. The fact that a PD or LOI states its date of effectivity does not preclude their publication in the OG as they constitute important legislative acts. The publication of presidential issuances of public nature or of general applicability is a requirement of due process. Before a person may be bound by law, he must first be officially informed of its contents. Judgment: Respondents ordered to publish in Official Gazette all unpublished presidential issuances of general application, and unless so published shall have no binding force and effect. Impt Point: It illustrates how decrees & issuances issued by one manMarcosare in fact laws of genl application & provide for penalties. The constitution afforded Marcos both executive & legislative powers.

The generality of law (CC A14) will never work w/o constructive notice. The ruling of this case provides the publication constitutes the necessary constructive notice & is thus the cure for ignorance as an excuse. Ignorance will not even mitigate the crime. Perez v Subido In this appeal from a decision finding for the petitioner in a mandamus suit, the appellants pose the legal issue: Has the respondent Commissioner of Civil Service the power and authority to invalidate exparte the examination papers of petitioner, cancel his eligibility (as patrolman) derived therefrom, and eventually terminate his services as patrolman in the Manila Police Department? Our answer is: No. In his application for the Patrolman Examinations held by the Civil Service Commission on 24 November 1962, the petitioner-appellee, Tomas M. Perez, answered yes to question No. 6, which reads as follows: 6. Have you ever been accused of, indicted for or tried for the violation of any law, ordinance, or regulation, before any court, or have you ever been charged with or tried for any breach or infraction of military, naval, or constabulary discipline before any military, naval, or constabulary tribunal or other authority? Petitioner-appellee mentioned only two of the criminal cases of which he had been charged but failed to mention two other cases, namely, attempted corruption of a public official and gambling. These cases were dismissed before he took the examinations. He was thus cleared by the pertinent government offices and lawenforcing agencies. When he passed the examinations, a character investigation and a six-month training course at the Manila Police Academy, petitioner Perez was appointed by Mayor Antonio Villegas of Manila as a probational patrolman of the Manila Police Department on 27 May 1963, with a compensation of P2,400 per annum. The respondent-appellant Commissioner of Civil Service approved the appointment. On 1 January 1964, appellee was extended a permanent appointment with a salary adjusted to P2,940 per annum. This appointment and a subsequent promotion in salary to P3,300 per annum were also approved by the respondent Commissioner.

On 20 October 1965, however, the Commissioner of Civil Service addressed a letter to the City Mayor informing him of the invalidation of the examination papers of appellee because of his failure to state the two other criminal cases of which he had been charged in his application to take the examinations, the cancellation of his civil service eligibility and the termination of his employment as patrolman in the Manila Police Department. Perez requested a reconsideration but was denied. The City Mayor found no ground upon which to proceed against Perez and did not dismiss him but respondent Commissioner transmitted copies of his letter to the mayor to the City Treasurer and the City Auditor who, thereafter, refused to pay the salary of Perez. On 12 April 1966, Tomas M. Perez filed a petition for mandamus with the Court of First Instance of Manila praying for the nullification of the order of the respondent Commissioner of Civil Service and to compel the other respondents City Treasurer and City Auditor to authorize and pay the petitioners salary. After trial bused on a stipulation of facts, the lower court granted the petition by declaring as null and void the Commissioners order terminating the petitioners employment and ordering the payment of his salary. Respondent Commissioner appealed to this Court on the point of law stated at the beginning of this decision. The situation of petitioner-appellee Tomas M. Perez is not unlike, and, even better than, the petitioner in the case of Abaya vs. Villegas, et al., L-25641, 17 December 1966 (18 Sup. Ct. Rep. Anno. 1034). Speaking through Justice Conrado Sanchez, this Court stated in said case: Two acts adverse to petitioner were done: Appellant Commissioners cancellation of his eligibility, and respondent Mayors order of dismissal. Concededly, however, there was no investigation prior to the cancellation of petitioners civil service eligibility and before he was eased out of the service. Consequently, petitioner was denied due process. On the first adverse act, this Court said: We go to the controlling principle. Rule II, Article 4, of the Civil Service Rules, in its proviso, reads: That when an applicant for examination intentionally makes a false statement of any

material fact in his application, or practices or attempts to practice any deception or fraud in connection with his examination, the Commissioner shall invalidate his examination and such offense shall be ground for his removal from the service.
The accent is on the word intentionally. The fact that petitioner made an answer which is at war with the truth does not connote the idea that it was intentionally made. Conceivably, situations may exist which could bring up that answer to the level of one given in good faith. Without evidence, we are loathe to tar and feather petitioner as a perjurer. Short of an inquiry which proves that the false answer was intentionally made, we cannot attribute thereto a meaning which would make a felon out of petitioner. It should then go without saying that the vitality of the constitutional principle of due process cannot be allowed to weaken by sanctioning cancellation of an employees civil service eligibility and/or his dismissal from service without hearing upon a doubtful assumption that he has admitted his guilt for an offense against civil service rules. It is pertinent here to recall that good faith is always presumed. In the context just presented, we are constrained to state that petitioner received less than a fair treatment. Due process, in the end, merely is the embodiment of the sporting idea of fair play. And on the second adverse act, this Court said: 1. The constitutionally protected security of tenure: [N]o officer or employee in the civil service shall be removed or suspended except for cause as provided by law. Section 32 of the Civil Service Law of 1959 echoes this precept with the provision that [N]o officer or employee in the civil service shall be removed or suspended except for cause as provided by law and after due process. Said Section 32 adds that the officer or employee complained of shall be entitled to a formal investigation if he so desires, in which case, he shall have the right to appear and defend himself at said investigation in person or by counsel, to confront and cross-examine the witnesses against him, and to have the attendance of witnesses and production of documents in his favor by compulsory process or subpoena or subpoena duces tecum. A civil service employee should be heard before he is condemned. Jurisprudence has clung to this rule with such unrelentless grasp that by now it would appear trite to make citation thereof. It is, therefore, plain that the Commissioner of Civil Service is without power or authority to cancel ex parteor without hearing the civil service eligibility of a patrolman nor to terminate his employment without a formal investigation.1 FOR THE FOREGOING REASONS, the appealed decision is hereby affirmed. No costs.

Trocio v Labayo Sometime in 1961, petitioner-appellant Arturo Trocio, as Municipal Treasurer of Mambajao, Camiguin, Misamis Oriental, was administratively charged for violation of Sec. 14 of the 1954 Revised Manual of Instructions to Treasurers and Sec. 614 of the Revised Administrative Code. Upon finding that Trocio had cash items consisting of National and Provincial payrolls amounting to P8,993.99 paid by him without the express approval of the Provincial Treasurer in violations of the provisions abovestated, the Civil Service Commissioner, on June 21, 1961, found him guilty as charged in a decision the dispositive portion of which read: x x x Wherefore, he is hereby fined his fifteen (15) days pay, reprimanded and warned that commission of another offense will be dealt with more severely. In the interest of the service, he should likewise be transferred without demotion in rank or salary such action not being considered disciplinary in character. xxx xxx xxx

Trocio refused to obey this directive, insisting that the decision of the Commissioner was illegal. On June 15, 1964, he appealed to the Civil Service Board of Appeals claiming that his conviction was without due process of law and that, given the opportunity, he can justify all his acts. Trocio also questioned the immediate execution of the decision on the ground that it has not yet become final and executory since under Sec. 36 of the Civil Service Act of 1959 (Republic Act 2260) he still had 30 days within which to appeal to the Civil Service Board of Appeals and on the further ground that his transfer was unlawful since he did not consent thereto. On June 20, 1964, Trocio went to the Court of First Instance of Misamis Oriental with a petition for prohibition with preliminary injunction to prevent the immediate execution of the questioned decision. The court issued the preliminary writ of injunction prayed for. But subsequently, the same court, acting upon an urgent motion to dismiss filed by respondents-appellees, dismissed the petition and dissolved the preliminary injunction, upholding the Commissioner's right to execute his decision pending appeal to the Civil Service Board of Appeals and Trocio's recall to the Provincial Treasurer's office. Upon denial of his motion for reconsideration, Trocio appealed to this Court raising questions purely of law. On August 6, 1964, petitioner-appellant filed here a motion for issuance of preliminary injunction to put him in possession of the office. This We denied. At issue is the authority of the Commissioner of Civil Service to enforce his decision pending its appeal to the Civil Service Board of Appeals. The Revised Civil Service Rules provides1 as follows: The Commissioner shall transmit to the head of Department concerned his decision on the case within six months from date of receipt of the complete record of the case and the head of Department shall, within ten (10) days from receipt thereof, serve the respondent a copy of the decision. The Commissioner, if public interest so warrants, may order his decision executed

pending appeal to the Civil Service Board of Appeals. (Emphasis supplied) Petitioner-appellant does not seriously question the power, in general, of the Commissioner to execute his decision pending appeal. What he assails, however, is the correctness or propriety of the latter's action, submitting that public interest meaning the interest of the Republic of the Philippines, and not that of Mambajao, Camiguin subprovince does not call for execution of the decision pending appeal in his particular case. The submission is untenable. Petitionerappellant perhaps would be correct if the Rules used the words "national interest." But even applying petitioner's concept of "public interest," the result will be the same. Since as municipal treasurer, he undoubtedly handles national as well as provincial and municipal funds, the interest of the Republic, or the "Community at large" is affected. And it is primarily for the Commissioner to determine when public interest warrants the execution, pending appeal, of his decisions since that is not ministerial but discretionary on his part. Petitioner-appellant then has the burden to show clearly that the action undertaken by the Commissioner amounted to grave abuse of discretion. None having been shown, the dismissal of the petition by the court a quocannot be validly assailed as erroneous. Petitioner-appellant relies upon Cabigao v. Del Rosario, L-18370, Oct. 31, 1962, for the proposition that the Commissioner may execute his decision pending appeal only if special reasons require it. This is untenable. Sec. 28 of Rule, XVIII of the Revised Civil Service Rules does not require the presence of special reasons. Moreover,Cabigao v. Del Rosario, is not authority for the proposition maintained by petitioner. The ratio decindendi there was that although the decision of the Commissioner adverse to the government employee under investigation is appealable to the Civil Service Board of Appeals, the former has discretion to enforce it and make it effective pending appeal, to protect public interest. The presence or absence of special reasons therefor was not at issue and hence anything said on the point must be regarded as mere obiter.

Petitioner-appellant would also advance as an issue here the correctness and legality of the questioned decision of the Civil Service Commissioner. But such question which goes to the merits of the case is not a matter that should be passed upon here. It should be resolved in the Civil Service Board of Appeals where the entire administrative decision is being questioned. The only question here is whether the decision of the Commissioner can be legally executed pending appeal. And under the facts and circumstances above described, We hold that it can. Wherefore, the order of dismissal appealed from is hereby affirmed with costs against petitioner-appellant. So ordered. Mendoza v NHA DECISION ABAD, J.: This case is about (a) the right of the National Housing Authority to seek annulment of sales made by housing beneficiaries of lands they bought from it within the prohibited period and (b) the distinction between actions for rescission instituted under Article 1191 of the Civil Code and those instituted under Article 1381 of the same code. The Facts and the Case On November 25, 1980 the National Housing Authority (NHA) executed a Deed of Sale with Mortgage over a Quezon City lot[1] in favor of the spouses Isidro and Flaviana Alfaro (the Alfaros). In due time, the Quezon City Registry of Deeds issued Transfer Certificate of Title (TCT) 277321 in the name of the Alfaros. The deed of sale provided, among others, that the Alfaros could sell the land within five years from the date of its release from mortgage without NHAs prior written consent. Thus: x x x. 5. Except by hereditary succession, the lot herein sold and conveyed, or any part thereof, cannot be alienated, transferred or encumbered within five (5) years from the date of release of herein mortgage without the prior written consent and authority from the VENDOR-MORTGAGEE

Trocio moved for reconsideration. This was denied by resolution of the Commissioner, dated May 27, 1964, which stated further that "the aforementioned decision be executed immediately, it appearing from the records that the respondent-petitioner has not yet paid the fine imposed upon him and that he has not also been transferred." Upon indorsement to him of the foregoing resolution by the Undersecretary of Finance for compliance, the Provincial Treasurer of Misamis Oriental issued a directive on June 3, 1964 reprimanding and warning Trocio, ordering him to effect immediate payment of the fine imposed and detailing him for assignment in the office of the Provincial Treasurer in Cagayan de Oro City, without demotion in rank or reduction in salary. On the same date, the Provincial Treasurer designated the assistant Municipal Treasurer, Gregorio Balanza, as officer in charge of the Municipal Treasurer's office of Mambajao in place of petitionerappellant.1wph1.t

(NHA). x x x.[2] (Emphasis supplied) The mortgage and the restriction on sale were annotated on the Alfaros title on April 14, 1981. About nine years later or on November 30, 1990, while the mortgage on the land subsisted, the Alfaros sold the same to their son, Victor Alfaro, who had taken in a common-law wife, Cecilia, with whom he had two daughters, petitioners Vicelet and Vicelen Lalicon (the Lalicons). Cecilia, who had the means, had a house built on the property and paid for the amortizations. After full payment of the loan or on March 21, 1991 the NHA released the mortgage. Six days later or on March 27 Victor transferred ownership of the land to his illegitimate daughters. About four and a half years after the release of the mortgage or on October 4, 1995, Victor registered the November 30, 1990 sale of the land in his favor, resulting in the cancellation of his parents title. The register of deeds issued TCT 140646 in Victors name. On December 14, 1995 Victor mortgaged the land to Marcela Lao Chua, Rosa Sy, Amparo Ong, and Ida See. Subsequently, on February 14, 1997 Victor sold the property to Chua, one of the mortgagees, resulting in the cancellation of his TCT 140646 and the issuance of TCT N172342 in Chuas name. A year later or on April 10, 1998 the NHA instituted a case before the Quezon City Regional Trial Court (RTC) for the annulment of the NHAs 1980 sale of the land to the Alfaros, the latters 1990 sale of the land to their son Victor, and the subsequent sale of the same to Chua, made in violation of NHA rules and regulations. On February 12, 2004 the RTC rendered a decision in the case. It ruled that, although the Alfaros clearly violated the five-year prohibition, the NHA could no longer rescind its sale to them since its right to do so had already prescribed, applying Article 1389 of the New Civil Code. The NHA and the Lalicons, who intervened, filed their respective appeals to the Court of Appeals (CA). On August 1, 2008 the CA reversed the RTC decision and found the NHA entitled to rescission. The CA declared TCT 277321 in the name of the Alfaros and all

subsequent titles and deeds of sale null and void. It ordered Chua to reconvey the subject land to the NHA but the latter must pay the Lalicons the full amount of their amortization, plus interest, and the value of the improvements they constructed on the property. The Issues Presented The issues in this case are: 1. Whether or not the CA erred in holding that the Alfaros violated their contract with the NHA; 2. Whether or not the NHAs right to rescind has prescribed; and 3. Whether or not the subsequent buyers of the land acted in good faith and their rights, therefore, cannot be affected by the rescission. The Rulings of the Court First. The contract between the NHA and the Alfaros forbade the latter from selling the land within five years from the date of the release of the mortgage in their favor.[3] But the Alfaros sold the property to Victor on November 30, 1990 even before the NHA could release the mortgage in their favor on March 21, 1991. Clearly, the Alfaros violated the five-year restriction, thus entitling the NHA to rescind the contract. The Lalicons contend, however, that the Alfaros did not violate the five-year restriction against resale since what the contract between the parties barred was a transfer of the property within five years from the release of the mortgage, not a transfer of the same prior to such release. But the Lalicons are trying to be clever. The restriction clause is more of a condition on the sale of the property to the Alfaros rather than a condition on the mortgage constituted on it. Indeed, the prohibition against resale remained even after the land had been released from the mortgage. The five-year restriction against resale, counted from the release of the property from the NHA mortgage, measures out the desired hold that the government felt it needed to ensure that its objective of providing cheap housing for the homeless is not defeated by wily entrepreneurs.

The Lalicons claim that the NHA unreasonably ignored their letters that asked for consent to the resale of the subject property. They also claim that their failure to get NHAs prior written consent was not such a substantial breach that warranted rescission. But the NHA had no obligation to grant the Lalicons request for exemption from the five-year restriction as to warrant their proceeding with the sale when such consent was not immediately forthcoming. And the resale without the NHAs consent is a substantial breach. The essence of the governments socialized housing program is to preserve the beneficiarys ownerships for a reasonable length of time, here at least within five years from the time he acquired it free from any encumbrance. Second. Invoking the RTC ruling, the Lalicons claim that under Article 1389 of the Civil Code the action to claim rescission must be commenced within four years from the time of the commission of the cause for it. But an action for rescission can proceed from either Article 1191 or Article 1381. It has been held that Article 1191 speaks of rescission in reciprocal obligations within the context of Article 1124 of the Old Civil Code which uses the term resolution. Resolution applies only to reciprocal obligations such that a breach on the part of one party constitutes an implied resolutory condition which entitles the other party to rescission. Resolution grants the injured party the option to pursue, as principal actions, either a rescission or specific performance of the obligation, with payment of damages in either case. Rescission under Article 1381, on the other hand, was taken from Article 1291 of the Old Civil Code, which is a subsidiary action, not based on a partys breach of obligation.[4] The four-year prescriptive period provided in Article 1389 applies to rescissions under Article 1381. . Here, the NHA sought annulment of the Alfaros sale to Victor because they violated the five-year restriction against such sale provided in their contract. Thus, the CA correctly ruled that such violation comes under Article 1191 where the applicable prescriptive period is that provided in Article 1144 which is 10 years

from the time the right of action accrues. The NHAs right of action accrued on February 18, 1992 when it learned of the Alfaros forbidden sale of the property to Victor. Since the NHA filed its action for annulment of sale on April 10, 1998, it did so well within the 10-year prescriptive period. Third. The Court also agrees with the CA that the Lalicons and Chua were not buyers in good faith. Since the five-year prohibition against alienation without the NHAs written consent was annotated on the propertys title, the Lalicons very well knew that the Alfaros sale of the property to their father, Victor, even before the release of the mortgage violated that prohibition. As regards Chua, she and a few others with her took the property by way of mortgage from Victor in 1995, well within the prohibited period. Chua knew, therefore, based on the annotated restriction on the property, that Victor had no right to mortgage the property to her group considering that the Alfaros could not yet sell the same to him without the NHAs consent. Consequently, although Victor later sold the property to Chua after the five-year restriction had lapsed, Chua cannot claim lack of awareness of the illegality of Victors acquisition of the property from the Alfaros. Lastly, since mutual restitution is required in cases involving rescission under Article 1191,[5] the NHA must return the full amount of the amortizations it received for the property, plus the value of the improvements introduced on the same, with 6% interest per annum from the time of the finality of this judgment. The Court will no longer dwell on the matter as to who has a better right to receive the amount from the NHA: the Lalicons, who paid the amortizations and occupied the property, or Chua, who bought the subject lot from Victor and obtained for herself a title to the same, as this matter was not raised as one of the issues in this case. Chuas appeal to the Court in a separate case[6] having been denied due course and NHA failing to file its own petition for review, the CA decision ordering the restitution in favor of the Lalicons has now become final and binding against them. WHEREFORE, the Court AFFIRMS the Decision of the Court of

In Re: Wenceslao Laureta In the case of In Re: Wenceslao Laureta, 148 SCRA 382 (1987), a lawyer was suspended indefinitely. A letter individually addressed to some justices of the Supreme Court is not covered by the constitutional right to privacy of communication when the same pertain to their exercise of judicial functions. To subject the threat and ordeal of investigation and prosecution, a judge, more so a member of the Supreme Court for official acts done by him in good faith and in the regular exercise of official duty and judicial functions is to subvert and undermine that very independence of the judiciary, and subordinate the judiciary to the executive. For it is a general principle of the highest importance to the proper administration of justice that a judicial officer exercising the authority vested in him, shall be free to act upon his own convictions, without apprehension of personal consequences to himself. Liability to answer to everyone who might feel himself aggrieved by the action of the judge would be inconsistent with the possession of this freedom, and would destroy that independence without which no judiciary can be either respectable or useful. (Bradley vs. Fisher, 80 U.S. 335). To allow litigants to go beyond the Courts resolution and claim that the members acted with deliberate bad faith and rendered an unjust resolution in disregard or violation of the duty of their high office to act upon their own independent consideration and judgment of the matter at hand would be destroy the authenticity, integrity and conclusiveness of such collegiate acts and resolutions and to disregard utterly the presumption of regular performance of official duty. To allow such collateral attack would destroy the separation of powers and undermine the role of the Supreme Court as the final arbiter of all justiciable disputes.

Nuez v Sandiganbayan Nuez assails the validity of the PD 1486 creating the Sandiganbayan as amended by PD 1606. He was accused before the Sandiganbayan of estafa through falsification of public and commercial documents committed in connivance with his other co-accused, all public officials, in several cases. It is the claim of Nuez that PD1486, as amended, is violative of the due process, equal protection, and ex post facto clauses of the Constitution. He claims that the Sandiganbayan proceedings violates Nuezs right to equal protection, because appeal as a matter of right became minimized into a mere matter of discretion; appeal likewise was shrunk and limited only to questions of law, excluding a review of the facts and trial evidence; and there is only one chance to appeal conviction, by certiorari to the SC, instead of the traditional two chances; while all other estafa indictees are entitled to appeal as a matter of right covering both law and facts and to two appellate courts, i.e., first to the CA and thereafter to the SC. ISSUE: Whether or not the creation of Sandiganbayan violates equal protection insofar as appeals would be concerned. HELD: The SC ruled against Nuez. The 1973 Constitution had provided for the creation of a special court that shall have originaljurisdiction over cases involving public officials charged with graft and corruption. The constitution specifically makes mention of the creation of a special court, the Sandiganbayan, precisely in response to a problem, the urgency of which cannot be denied, namely, dishonesty in the public service. It follows that those who may thereafter be tried by such court ought to have been aware as far back as January 17, 1973, when the present Constitution came into force, that a different procedure for the accused therein, whether a private citizen as
petitioner is or a public official, is not necessarily offensive to the equal protection clause of the Constitution. Further, the classification therein set forth met the standard requiring that it must be based on substantial distinctions which make real differences; it must be germane to the purposes of the law; it must not be limited to existing conditions only, and must apply equally to each member of the class. Further still, decisions in the Sandiganbayan are reached by a unanimous decision from 3 justices - a showing that decisions therein are more conceivably carefully reached than other trial courts.

Justice Makasiar (concurring & dissenting) Persons who are charged with estafa or malversation of funds not belonging to the government or any of its instrumentalities or agencies are guaranteed the right to appeal to two appellate courts first, to the CA, and thereafter to the SC. Estafa and malversation of private funds are on the same category as graft and corruption committed by public officers, who, under the decree creating the Sandiganbayan, are only allowed one appeal to the SC (par. 3, Sec. 7, P.D. No. 1606). The fact that the Sandiganbayan is a collegiate trial court does not generate any substantial distinction to validate this invidious discrimination. Three judges sitting on the same case does not ensure a quality of justice better than that meted out by a trial court presided by one judge. The ultimate decisive factors are the intellectual competence, industry and integrity of the trial judge. But a review by two appellate tribunals of the same case certainly ensures better justice to the accused and to the people. Then again, par 3 of Sec 7 of PD 1606, by providing that the decisions of the Sandiganbayan can only be reviewed by the SC through certiorari, likewise limits the reviewing power of the SC only to question of jurisdiction or grave abuse of discretion, and not questions of fact nor findings or conclusions of the trial court. In other criminal cases involving offenses not as serious as graft and corruption, all questions of fact and of law are reviewed, first by the CA, and then by the SC. To repeat, there is greater guarantee of justice in criminal cases when the trial courts judgment is subject to review by two appellate tribunals, which can appraise the evidence and the law with greater objectivity, detachment and impartiality unaffected as they are by views and prejudices that may be engendered during the trial. Limiting the power of review by the SC of convictions by the Sandiganbayan only to issues of jurisdiction or grave abuse of discretion, likewise violates the constitutional presumption of innocence of the accused, which presumption can only be overcome by proof beyond reasonable doubt (Sec. 19, Art. IV, 1973 Constitution).

Montemayor v Araneta University Montemayor v. Araneta University, 77 SCRA 321 (1977) Lack of Due Process in Termination of Employment Remedied by Hearing in the NLRC. F: Petitioner was a professor at the Araneta University Foundation. On 7/8/74, he was found guilty of making homosexual advances on one Leonardo De Lara by a faculty investating committee. On 11/8/74, another committee was appointed to investigate another charge of a similar nature against petitioner. Petitioner, through cousel, asked for the postponement of the hearing set for 11/18 and 19, 1974, but the w/c motion was denied. The committe then proceeded to hear the testimony of the complainants and on 12/5/74, submitted its report recommending the separation of petitioner from the University. On 12/12/74, the University applied w/ the NLRC for clearance to terminate petitioner''''s employment. Meanwhile, petitioner filed a complaint w/ the NLRC for reinstatement and backwages. Judgement was rendered in petitioner''''s favor, but on appeal to the Sec. of Labor, the latter found petitioner''''s dismissal to be justified. Hence, this petition for certiorari. HELD: The Consti. assures to workers security of tenure. In the case of petitioner, this guarantee is reinforced by the provision on academic freedom. In denying petitioner''''s motion for postponement of the hearing, the committee did not accord procedural due process to the petitioner. This was, however, remedied at the mediation conference called at the Dept. of Labor during w/c petitioner was heard on his evidence. There he was given the fullest opportunity to present his case. Petition dismissed. Petitioner filed a MFR contending that the hearing in the NLRC did not conform to ther requirements of due process as the witnesses against petitioner were not called so that petitioner could cross-examine them. HELD: Pettioner did not object to the presentation of the testimony of the complainant and the witnesses at the school investigation and did not assert his right to cross-examine them. Petitioner waived his right to confront the witnesses, relying solely on the strength of his evidence. Nor was it incumbent on resp. to present the witnesses in the NLRC. Petitioner''''s only right is to be heard. VV. Tanada v. Phil. Atomic Energy Commission, 141 SCRA 307 (1986) Bias as Disqualification in Administrative Investigations F: Petition for

prohibition brought by taxpayers, questioning the competence of members of the PAEC to pass judgment on the safety of the Bataan Power Plant and charging them w/ bias and prejudgment, based on their publications stating that the plant was safe.
HELD: (1) Where the validity of an appointment is not challenged in an appropriate proceeding, the question of competence is not w/in the filed of judicial inquiry. The question of competence is a matter addressed to the appointing power. (2) In these publications, PAEC clearly indicated its prejudgement that the nuclear plant is safe. The first was published in 1985. The other 2 were issued earlier, but as the majority of the PAEC commissioners even then were already occupying responsible positions in the PAEC, they cannot escape responsibility for these publications. Petition granted and PAEC restrained from acting in the proceedings for issuance of license. In re Letter to Mrs. Maria Coronel In re Letter to Mrs. Maria Coronel, 238 SCRA 619 F: Benito Mapilisan was dismissed by the Supreme Court for dishonestry amounting to gross misconduct occasioned by the loss, while in his custody, of a registered mail letter, the contents of which were pilfered, as the P25,000 bank draft contained therein, eventually found its way for encashment with payee bank, PNB. The contentions of this MFR hinge on the alleged violation of procedural due process by claiming that there was no formal administrative complaint filed against him by either complainant Maria Coronel or the CA nor was there a formal investigation conducted. Accroding to him, the letter complaint of Maria Coronel failed to properly and formally charge the former since the letter complaint was neither suscribed nor sworn to by the complainant. ISSUE: W/N there was a violation of procedural due process. HELD: NO Under Sec. 6, Art. VIII of the 1987 Constitution, the Supreme Court shall have administrative supervision over all courts and the personnel thereof. By this power, the Supreme Court can act on a subject complaint, particularly where , as in this case, an investigation thereon had already been conducted and concluded by the CA prior to its referral to SC, with the recommendation that movant be charged with gross negligence. The complaint, though defective as to form, has been fully substantiated on the basis of the accompanying independent investigation Report and Recommendation submitted to SC. As a matter of fact, the Court even entertains anonymous complaints where the charge can be fully borne out by the evidence offered, or continues proceeding with the administrative case despite withdrawal by complainant of his complaint where an investigation thereto has

been made by the Acting Court Administrator prior to the filing of the complaint. Thus, what is given paramount importance by this Court in instances where its power of administrative supervision over court employees is invoked is the substantiation of the complaint rather than its conformity with the formal requirements.

Agustin v EDU Volkswagen Beetle Car. He is assailing the validity of Letter of Instruction No 229 which requires all motor vehicles to have early warning devices particularly to equip them with a pair of reflectorized triangular early warning devices. Agustin is arguing that this order is unconstitutional, harsh, cruel and unconscionable to the motoring public. Cars are already equipped with blinking lights which is already enough to provide warning to other motorists. And that the mandate to compel motorists to buy a set of reflectorized early warning devices is redundant and would only make manufacturers and dealers instant millionaires. ISSUE: Whether or not the said is EO is valid. HELD: Such early warning device requirement is not an expensive redundancy, nor oppressive, for car owners whose cars are already equipped with 1) blinking-lights in the fore and aft of said motor vehicles, 2) battery-powered blinking lights inside motor vehicles, 3) built-in reflectorized tapes on front and rear
bumpers of motor vehicles, or 4) well-lighted two (2) petroleum lamps (the Kinke) . . . because: Being universal among the signatory countries to the said 1968 Vienna Conventions, and visible even under adverse conditions at a distance of at least 400 meters, any motorist from this country or from any part of the world, who sees a reflectorized rectangular early warning device installed on the roads, highways or expressways, will conclude, without thinking, that somewhere along the travelled portion of that road, highway, or expressway, there is a

Vera v Cuevas Vera v. Cuevas 90 SCRA 379 (1979) F: Respondents are engaged in the manufacture and sale of filled milk products. They brought an action in the CFI for a declaration of their rights in respect of section 169 of the Tax Code. This provision required that "all condensed skimmed milk in whatever form, from which the fatty part has been removed totally or in part or put on sale in the Philippines shall be clearly and legibly marked on its immediate containers with the words: This milk is not suitable for nourishment for infants less than one year of age. HELD: Sec. 169 of the Tax Code has been repealed by RA 344. At any rate, Sec. 169 applied only to skimmed milk and not to filled milk. Sec. 169 is being enforced only against respondent manufacturers of filled milk but not against manufacturers of skimmed milk, thus denying them the equal protection of the laws. Sison v Araneta Facts: A suit for declaratory relief or prohibition was filed by petitioner Sison, challenging the constitutionality of Section I of Batas Pambansa Blg. 135, amending Section 21 of the National Internal Revenue Code of 1977, which provides for rates of tax on citizens or residents on (a) taxable compensation income, (b) taxable net income, (c) royalties, prizes, and other winnings, (d) interest from bank deposits and yield or any other monetary benefit from deposit substitutes and from trust fund and similar arrangements, (e) dividends and share of individual partner in the net profits of taxable partnership, (f) adjusted gross income. Petitioner as taxpayer alleges that by virtue of such provision, "he would be unduly discriminated against by the imposition of higher rates of tax upon his income arising from the exercise of his profession vis-a-vis those which are imposed upon fixed income or salaried individual taxpayers. He further contends that such law is arbitrary, amounting to class legislation, oppressive and capricious in character and that it transgresses both the equal and due process clauses of the Constitution as well as the rule requiring uniformity in taxation.

Issue: Whether or not Section 1 of Batas Pambansa Blg. 135 is constitutional. Held: A mere allegation of arbitrariness, as here, does not suffice. There must be a factual foundation of such unconstitutional taint This is merely to adhere to the authoritative doctrine that where the due process and equal protection clauses are invoked, considering that they are not fixed rules but rather broad standards, there is a need for of such persuasive character as would lead to such a conclusion. Absent such a showing, the presumption of validity must prevail. It is undoubted that the due process clause may be invoked where a taxing statute is so arbitrary that it finds no support in the Constitution. It has also been held that where the assailed tax measure is beyond the jurisdiction of the state, or is not for a public purpose, or, in case of a retroactive statute is so harsh and unreasonable, it is subject to attack on due process grounds. Now for equal protection. The applicable standard to avoid the charge that there is a denial of this constitutional mandate, whether the assailed act is in the exercise of the police power or the power of eminent domain, is to demonstrated that the governmental act assailed, far from being inspired by the attainment of the common weal was prompted by the spirit of hostility, or at the very least, discrimination that finds no support in reason. It suffices then that the laws operate equally and uniformly on all persons under similar circumstances or that all persons must be treated in the same manner, the conditions not being different, both in the privileges conferred and the liabilities imposed. That same formulation applies as well to taxation measures. The Constitution does not require things which are different in fact or opinion to be treated in law as though they were the same." Hence, the constant reiteration of the view that classification if rational in character is allowable. As a matter of fact, in a leading case of Lutz V. Araneta, this Court, through Justice J.B.L. Reyes, went so far as to hold "at any rate, it is inherent in the power to tax that a state be free to select the subjects of taxation, and it has been repeatedly held that inequalities which result from a singling out of one particular

motor vehicle which is stationary, stalled or disabled which obstructs or endangers passing traffic. On the other hand, a motorist who sees any of the aforementioned
other built-in warning devices or the petroleum lamps will not immediately get adequate advance warning because he will still think what that blinking light is all about. Is it an emergency vehicle? Is it a law enforcement car? Is it an ambulance? Such confusion or uncertainty in the mind of the motorist will thus increase, rather than decrease, the danger of collision.

class for taxation, or exemption infringe no constitutional limitation.'" Petitioner likewise invoked the kindred concept of uniformity. According to the Constitution: "The rule of taxation shall be uniform and equitable." This requirement is met according to Justice Laurel in Philippine Trust Company v. Yatco, decided in 1940, when the tax "operates with the same force and effect in every place where the subject may be found. " He likewise added: "The rule of uniformity does not call for perfect uniformity or perfect equality, because this is hardly attainable." The taxing power has the authority to make reasonable and natural classifications for purposes of taxation. Apparently, what misled petitioner is his failure to take into consideration the distinction between a tax rate and a tax base. There is no legal objection to a broader tax base or taxable income by eliminating all deductible items and at the same time reducing the applicable tax rate. Taxpayers may be classified into different categories. To repeat, it is enough that the classification must rest upon substantial distinctions that make real differences. In the case of the gross income taxation embodied in Batas Pambansa Blg. 135, the discernible basis of classification is the susceptibility of the income to the application of generalized rules removing all deductible items for all taxpayers within the class and fixing a set of reduced tax rates to be applied to all of them. Taxpayers who are recipients of compensation income are set apart as a class. As there is practically no overhead expense, these taxpayers are not entitled to make deductions for income tax purposes because they are in the same situation more or less. On the other hand, in the case of professionals in the practice of their calling and businessmen, there is no uniformity in the costs or expenses necessary to produce their income. WHEREFORE, the petition is dismissed. Costs against petitioner.

*****EQUAL PROTECTION OF LAWS Pp v Cayat Facts: Law prohibits any member of a nonChristian tribe to buy, receive, have in his possession, or drink, any intoxicating liquors of any kind. The law, Act No. 1639, exempts only the so-called native wines or liquors which the members of such tribes have been accustomed to take. Issue: Whether or Not the law denies equal protection to one prosecuted and sentenced for violation of said law. Held: No. It satisfies the requirements of a valid classification, one of which is that the classification under the law must rest on real or substantial distinctions. The distinction is reasonable. The classification between the members of the non- Christian and the members of the Christian tribes is not based upon accident of birth or parentage but upon the degree of civilization and culture. The term nonChristian tribes refers to a geographical area and more directly to natives of the Philippines of a low grade civilization usually living in tribal relationship apart from settled communities. The distinction is reasonable for the Act was intended to meet the peculiar conditions existing in the nonChristian tribes The prohibition is germane to the purposes of the law. It is designed to insure peace and order in and among the non- Christian tribes has often resulted in lawlessness and crime thereby hampering the efforts of the government to raise their standards of life and civilization. This law is not limited in its application to conditions existing at the time of the enactment. It is intended to apply for all times as long as those conditions exists. The Act applies equally to all members of the class. That it may be unfair in its operation against a certain number of nonChristians by reason of their degree of culture is not an argument against the equality of its operation nor affect the reasonableness of the classification thus established.

Tan Ty v Land Tenure Administration The decision of the Court of First Instance of Manila (in Civil Case No. 68028), is appealed to this Court by Tan Ty, a Chinese citizen, raising the issue of the qualification of a non-Filipino citizen to lease land previously expropriated by the Government for resale to its bona fide tenants and occupants and to other qualified persons, under the provisions of Republic Act 1162, as amended by Republic Act 1599. The case arose from the denial by the Land Tenure Administration (now replaced by Land Authority) of the application of Tan Ty, a Chinese, to lease four lots (Nos. 19, 20, 21, and 24, Block No. 3) of the Fable Estate in Paco, Manila, which was purchased by the government in 1955 to be subdivided into small lots for resale to its bona fide tenants and occupants and to other persons. The denial of the aforementioned application was based on the fact that, although applicant Tan Ty was able to establish that she was the tenant of the lots applied for even prior to the governments acquisition of the Estate, her application to lease the same was filed with the Land Tenure Administration only on 21 October 1957, or after the said lots had been allocated to other parties.1 Tan Tys claim to priority in the lease of the lots was also rejected, for the reason that under the law, the privilege to lease lands expropriated by the government for resale may be availed of only by tenants or occupants qualified to purchase, but who are not financially capable at the start of purchasing at cost the lot or lots occupied by them. And applicant Tan Ty is not such tenant or occupant qualified to buy the land. On appeal by the applicant, the President, through the Assistant Executive Secretary, affirmed the ruling of the Land Tenure Administration. It was held that although the applicant may not be considered disqualified by the fact of her Chinese citizenship, the legal prohibition against alien acquisition of real property being directed against the purchase thereof and not to a mere leasing of the same, Tan Tys application had to be denied because it is filed after the lots applied for had been allocated to other applicants. Besides, the decision stated, investigation disclosed that applicant Tan Ty was not an occupant of the area applied for, in fact, no construction existed thereon. The applicant was staying in a building adjacent to the lots in question, and originally owned by her but which she later sold to Roxas Chua de Ayala. When her

motion for reconsideration of the Presidents decision was denied, Tan Ty instituted certiorari proceeding in the Court of First Instance of Manila (Civil Case No. 68028), charging the Land Tenure Administration (whose functions were taken over by the Land Authority) of having committed grave abuse of discretion in allocating Lots Nos. 19, 20, 21, and 24 of Block 3 of the Fable Estate to other persons, thus depriving petitioner of her property rights thereto without due process of law. After the respondents Land Authority and Executive Secretary had filed their answers, and the parties their respective memoranda, the case was submitted for judgment on the pleadings. In its decision of 5 July 1967, the court ruled that petitioner, being a Chinese and therefore disqualified to purchase the disputed lots, is equally without qualification to lease the same, the right to lease, under the law, arising merely from the applicants right to purchase. Her claim that she was deprived of the right to equal protection of the law was, likewise, overruled. The court declared that the equal protection clause is not without limitations; that there are rights that the law reserves only to Filipino citizens, such as the right of suffrage, the right to hold public office, to operate public services and others. The right to own lands is one of them. The applicant moved to reconsider this decision, and when it was denied, she interposed the present appeal. The lower court could not have ruled otherwise. Admittedly, appellant is not qualified to purchase the land involved in the case on account of her Chinese citizenship. She insists, however, that as a bona fide tenant thereof, she is entitled to lease the property, citing in support of her allegation Section 3 of Republic Act 1162, 2 as amended by Republic Act 1599, which provides -. SEC. 3. The landed estates or haciendas expropriated by virtue of this Act shall be subdivided into small lots, none of which shall exceed one hundred and fifty square meters in area, to be sold at cost to the tenants, or occupants, of said lots, and to other individuals, in the order mentioned; Provided, That if the tenant of any given lot is not able to purchase said lot, he shall be given a lease from month to month of said lot until such time that he is able to purchase the same; Provided, further, That in the event of lease, the rentals that may be charged by the Government shall not

exceed eight per cent per annum of the assessed valuation of the property leased. The legal provisions abovequoted is too clear to cause any confusion or diverging interpretations. The pervading aim of the law, which properly should guide the construction to be placed on any of its provisions, is for the State to provide home lots to its citizens, to enable them to own at least the land on which their houses are built. Hence, the mandate to the Administration to acquire landed estates, break them up into small lots not exceeding 150 square meters in area, for resale at cost to the tenants or occupants thereof, not to other persons. And if a tenant is in no position to buy the lot he is occupying at the time of its acquisition by the government, the law even grants him the privilege to lease the same from the government on monthly basis, until such time that he is able to purchase the same. No other conclusion can be derived from the provision than that the lease mentioned therein can be availed of only by one who is qualified, but is in no financial position for the present to buy the land. It certainly would not justify the lease of the lots to non-Philippine citizens, notwithstanding the fact that they may have really been the tenants thereon even prior to the acquisition of the expropriated property by the government. To extend to these aliens the benefit of the first proviso of Section 3 of Republic Act No. 1162 above-quoted, not withstanding their permanent disability to acquire title to land under the Constitution, would result in their acquiring possession in perpetuity. This is not only incompatible with the essential temporary nature in law of a contract of lease, but, in effect, would be a violation of the Constitutional inhibition. For a perpetual lease of land would be practically an alienation thereof. Appellant asks whether the provision as thus construed would not constitute a deprivation of appellants right to property without due process of law, or amount to a denial to appellant, a resident of the Philippines, of the equal protection guarantee of the Constitution. It must be remembered in this connection that the fact alone that the law withdraws from non-Filipinos the enjoyment of certain rights exclusively reserved to Philippine citizens does not expose such law to charges of constitutional infirmity. For classification will constitute no violation of the individuals right to equal protection as

long as it is not unreasonable, arbitrary or capricious. The established and recognized principle is that classification is not unreasonable where it is based on substantial distinctions that make real differences, is germane to the aim and purpose of the law, is not limited to existing conditions, and it applies equally to all members of the same class, under similar conditions.3 A legislation that affects a particular class would not infringe the constitutional guarantee of equal protection of the laws, provided said statute applies uniformly and without discrimination to everyone of that class.4 In this case, a
classification based on citizenship is more than justified. The requirement that lots in an expropriated estate shall be leased only to those who are qualified in law to buy the same can hardly be considered unreasonable. Republic Act 1162 was not enacted to allow the government to go into the real estate business, by leasing lots to those who desire and can afford to rent. It is rather aimed at giving realization and meaning to the policy of the State to provide land for the landless citizens and enabling them to acquire home-lots at minimum cost. Clearly, the measure of reasonableness is reached. Neither can it be seriously contended that by the governments denial of appellants application to lease the lots, and their allocation in favor of other persons, appellant was deprived of property rights without due process. It was said, not inappropriately, that due process, as a constitutional mandate, is based on reasons.5 No irrationality can be said to characterize the denial of appellants application. Her occupancy of the lots as a tenant of the previous owner on a month-tomonth basis did not confer upon her any vested right on the property leased that the new owner was bound to respect. And nothing appears on the record that the government, in exercising the right of eminent domain, had agreed to respect any existing contract in favor of tenants like the appellant. While appellant may have introduced improvements on the lot leased, by so doing despite absence of a fixed term, the lessee took the risk of not being able to recoup the value of said improvements before being compelled to vacate. For this mischance, the lessee is solely responsible. The law is clear that at the expiration of the lease the lessor is not compellable to acquire the improvements made by the lessee (Civil Case * of the Philippines, Article 1678). WHEREFORE, finding no error in the decision of the court below, the same is hereby affirmed, with costs against the appellant.

Gomez v Palomar FACTS: Petitioner Benjamin Gomez mailed a letter at the post office in San Fernando, Pampanga. It did not bear the special antiTB stamp required by the RA 1635. It was returned to the petitioner. Petitioner now assails the constitutionality of the statute claiming that RA 1635 otherwise known as the Anti-TB Stamp law is violative of the equal protection clause because it constitutes mail users into a class for the purpose of the tax while leaving untaxed the rest of the population and that even among postal patrons the statute discriminatorily grants exemptions. The law in question requires an additional 5 centavo stamp for every mail being posted, and no mail shall be delivered unless bearing the said stamp. ISSUE: Is the Anti-TB Stamp Law unconstitutional, for being allegedly violative of the equal protection clause? HELD: No. It is settled that the legislature has the inherent power to select the subjects of taxation and to grant exemptions. This power has aptly been described as "of wide range and flexibility." Indeed, it is said that in the field of taxation, more than in other areas, the legislature possesses the greatest freedom in classification. The reason for this is that traditionally, classification has been a device for fitting tax programs to local needs and usages in order to achieve an equitable distribution of the tax burden. The classification of mail users is based on the ability to pay, the enjoyment of a privilege and on administrative convenience. Tax exemptions have never been thought of as raising revenues under the equal protection clause. Assoc of Small Landowners v Sec of Agrarian Reform "Land for the Landless" is a slogan that underscores the acute imbalance in the distribution of this precious resource among our people. But it is more than a slogan. Through the brooding centuries, it has become a battle-cry dramatizing the increasingly urgent demand of the dispossessed among us for a plot of earth as their place in the sun. Recognizing this need, the Constitution in 1935 mandated the policy of social justice to "insure the well-being and economic security of all the people," especially the less privileged. In 1973, the new Constitution affirmed this goal adding specifically that "the State shall regulate the

acquisition, ownership, use, enjoyment and disposition of private property and equitably diffuse property ownership and profits." Significantly, there was also the specific injunction to "formulate and implement an agrarian reform program aimed at emancipating the tenant from the bondage of the soil." Facts: The petitioners in this case invoke the right of retention granted by P.D. No. 27 to owners of rice and corn lands not exceeding seven hectares as long as they are cultivating or intend to cultivate the same. Their respective lands do not exceed the statutory limit but are occupied by tenants who are actually cultivating such lands. According to P.D. No. 316, which was promulgated in implementation of P.D. No. 27: No tenant-farmer in agricultural lands primarily devoted to rice and corn shall be ejected or removed from his farmholding until such time as the respective rights of the tenant- farmers and the landowner shall have been determined in accordance with the rules and regulations implementing P.D. No. 27. The petitioners claim they cannot eject their tenants and so are unable to enjoy their right of retention because the Department of Agrarian Reform has so far not issued the implementing rules required under the above-quoted decree. They therefore ask the Court for a writ of mandamus to compel the respondent to issue the said rules. The public respondent argues that P.D. No. 27 has been amended by LOI 474 removing any right of retention from persons who own other agricultural lands of more than 7 hectares in aggregate area or lands used for residential, commercial, industrial or other purposes from which they derive adequate income for their family. And even assuming that the petitioners do not fall under its terms, the regulations implementing P.D. No. 27 have already been issued, to wit, the Memorandum dated July 10, 1975 (Interim Guidelines on Retention by Small Landowners, with an accompanying Retention Guide Table), Memorandum Circular No. 11 dated April 21, 1978, (Implementation Guidelines of LOI No. 474), Memorandum Circular No. 18-81 dated December 29,1981 (Clarificatory Guidelines

on Coverage of P.D. No. 27 and Retention by Small Landowners), and DAR Administrative Order No. 1, series of 1985 (Providing for a Cut-off Date for Landowners to Apply for Retention and/or to Protest the Coverage of their Landholdings under Operation Land Transfer pursuant to P.D. No. 27). For failure to file the corresponding applications for retention under these measures, the petitioners are now barred from invoking this right. The petitioners insist that the above-cited measures are not applicable to them because they do not own more than seven hectares of agricultural land. The Constitution of 1987 was not to be outdone. Besides echoing these sentiments, it also adopted one whole and separate Article XIII on Social Justice and Human Rights, containing grandiose but undoubtedly sincere provisions for the uplift of the common people. These include a call in the following words for the adoption by the State of an agrarian reform program: SEC. 4. The State shall, by law, undertake an agrarian reform program founded on the right of farmers and regular farmworkers, who are landless, to own directly or collectively the lands they till or, in the case of other farmworkers, to receive a just share of the fruits thereof. To this end, the State shall encourage and undertake the just distribution of all agricultural lands, subject to such priorities and reasonable retention limits as the Congress may prescribe, taking into account ecological, developmental, or equity considerations and subject to the payment of just compensation. In determining retention limits, the State shall respect the right of small landowners. The State shall further provide incentives for voluntary landsharing. Issue: Whether or not all rights acquired by the tenant-farmer under P.D. No. 27, as recognized under E.O. No. 228, are retained by him even under R.A. No. 6657. Held: P.D. No. 27 expressly ordered the emancipation of tenant-farmer as October 21, 1972 and declared that he shall "be deemed the owner" of a portion of land consisting of a family-sized farm except that "no title to the land owned by him was to be actually issued to him unless and until he had become a full-fledged member of a duly recognized farmers' cooperative." It was

understood, however, that full payment of the just compensation also had to be made first, conformably to the constitutional requirement. When E.O. No. 228, categorically stated in its Section 1 that: All qualified farmer-beneficiaries are now deemed full owners as of October 21, 1972 of the land they acquired by virtue of Presidential Decree No. 27. The CARP Law, for its part, conditions the transfer of possession and ownership of the land to the government on receipt by the landowner of the corresponding payment or the deposit by the DAR of the compensation in cash or LBP bonds with an accessible bank. Until then, title also remains with the landowner. No outright change of ownership is contemplated either. This should counter-balance the express provision in Section 6 of the said law that "the landowners whose lands have been covered by Presidential Decree No. 27 shall be allowed to keep the area originally retained by them thereunder, further, that original homestead grantees or direct compulsory heirs who still own the original homestead at the time of the approval of this Act shall retain the same areas as long as they continue to cultivate said homestead." R.A. No. 6657 does provide for such limits now in Section 6 of the law, which in fact is one of its most controversial provisions. Retention Limits. Except as otherwise provided in this Act, no person may own or retain, directly or indirectly, any public or private agricultural land, the size of which shall vary according to factors governing a viable family-sized farm, such as commodity produced, terrain, infrastructure, and soil fertility as determined by the Presidential Agrarian Reform Council (PARC) created hereunder, but in no case shall retention by the landowner exceed five (5) hectares. Three (3) hectares may be awarded to each child of the landowner, subject to the following qualifications: (1) that he is at least fifteen (15) years of age; and (2) that he is actually tilling the land or directly managing the farm; Provided, That landowners whose lands have been covered by Presidential Decree No. 27 shall be allowed to keep the area originally retained by them thereunder, further, That original

homestead grantees or direct compulsory heirs who still own the original homestead at the time of the approval of this Act shall retain the same areas as long as they continue to cultivate said homestead. All rights previously acquired by the tenantfarmers under P.D. No. 27 are retained and recognized. Landowners who were unable to exercise their rights of retention under P.D. No. 27 shall enjoy the retention rights granted by R.A. No. 6657 under the conditions therein prescribed. Subject to the above-mentioned rulings all the petitions are DISMISSED, without pronouncement as to costs. Pp v Vera Cu Unjieng was convicted by the trial court in Manila. He filed for reconsideration which was elevated to the SC and the SC remanded the appeal to the lower court for a new trial. While awaiting new trial, he appealed for probation alleging that the he is innocent of the crime he was convicted of. Judge Tuason of the Manila CFI directed the appeal to the Insular Probation Office. The IPO denied the application. However, Judge Vera upon another request by Cu Unjieng allowed the petition to be set for hearing. The City Prosecutor countered alleging that Vera has no power to place Cu Unjieng under probation because it is in violation of Sec. 11 Act No. 4221 which provides that the act of Legislature granting provincial boards the power to provide a system of probation to convicted person. Nowhere in the law is stated that the law is applicable to a city like Manila because it is only indicated therein that only provinces are covered. And even if Manila is covered by the law it is unconstitutional because Sec 1 Art 3 of the Constitution provides equal protection of laws. The said law provides absolute discretion to provincial boards and this also constitutes undue delegation of power. Further, the said probation law may be an encroachment of the power of the executive to provide pardon because providing probation, in effect, is granting freedom, as in pardon. ISSUE: Whether or not equal protection is violated when the Probation Law provides that ony in those provinces in which the respective provincial boards have provided for the salary of a probation officer may the probation system be applied. HELD: The act of granting probation is not the same as pardon. In fact it is limited and

is in a way an imposition of penalty. There is undue delegation of power because there is no set standard provided by Congress on how provincial boards must act in carrying out a system of probation. The provincial boards are given absolute discretion which is violative of the constitution and the doctrine of the non delegability of power. Further, it is a violation of equity so protected by the constitution. The challenged section of Act No. 4221 in section 11 which reads as follows: This Act shall apply only in those provinces in which the respective provincial boards have provided for the salary of a probation officer at rates not lower than those now provided for provincial fiscals. Said probation officer shall be appointed by the Secretary of Justice and shall be subject to the direction of the Probation Office. This only means that only provinces that can provide appropriation for a probation officer may have a system of probation within their locality. This would mean to say that convicts in provinces where no probation officer is instituted may not avail of their right to probation. The SC declared the old probation law as unconstitutional. Manila Electric Co v Public Utilities Employers Assoc This is an appeal by certiorari under Rule 44 of the Rules of Court interposed by the petitioner Manila Electric Company against the decision of July 15, 1946 of the Court of Industrial Relations, which reads as follows: Although the practice of the company, according to the manifestations of counsel for said company, has been to grant one day vacationwith pay to every workingman who had worked for seven consecutivedays including Sundays, the Court considers justified the oppositionpresented by the workingmen to the effect that they need Sundays andholidays for the observance of their religion and for rest. The Court,therefore, orders the respondent company to pay 50 per cent increasefor overtime work done on ordinary days and 50 per cent increase for work done during Sundays and legal holidays irrespective of the

numberof days they work during the week. The appellant contends that the said decision of the Court of IndustrialRelations is against the provision of section 4, Commonwealth Act No. 444, which reads as follows: No person, firm, or corporation, business establishment or place or center of labor shall compel an employee or laborer to work during Sundays and legal holidays, unless he is paid an additionalsum of at least twenty-five per centum of his regular remuneration: Provided, however, Thast this prohibition shall not apply to publicutilities performing some public service such as supplying gas,electricity, power, water, or providing means of transportationor communication. After a careful consideration of the issue involved in this appeal, we are of the opinion and so hold that the decision of the Court of Industrial Relations is erroneous od contrary to the clear and express provision of the above quoted provisions. The power of theCourt to settle industrial disputes between capital and labor, which include the fixing of wages of employees or laborers, granted by the general provisions of section 1 of Commonwealth Act No. 103, has beenrestricted by the above quoted special provisions of Commonwealth ActNo. 444, in the sense that public utilities supplying electricity,gas, power, water, or providing means of transportation or communication may compel their employees or laborers to work duringSundays and legal holidays without paying them an additional compensation of not less than 25 per cent of their regular remuneration on said days. Since the provisions of the above quoted section 4, are plain and unambiguous and convey a clear and definite meaning, there is no need of resorting to the rules of statutory interpretation orconstruction in order to determine the intention of the Legislature.Said section 1 consists of two parts: the first, which is the enactmentclause, prohibits a person, firm or corporation, business establishment,or place

or center of labor from compelling an employee or laborer towork during Sundays and legal holidays, unless the former pays thelatter an additional sum of at least twenty five per centum of his regular remuneration; and the second part, which is an exception,exempts public utilities performing some public service, such assupplying gas, electricity, power, water or providing means oftransportation or communication, from the prohibition establishedin the enactment clause. As the appellant is a public utility that supplies the electricity and provides means of transportation to the public, it is evident that the appellant is exempt from the qualifiedprohibition established in the enactment clause, and may compel its employees or laborers to work during Sundays and legal holidays without paying them said extra compensation. To hold that the exception or second part of section 4, CommonwealthAct No. 444, only exempts public utilities mentioned therein from the prohibition to compel employees or laborers to work during Sundaysand legal holidays, but not from the obligation to pay them an extraor additional compensation for compelling them to work during thosedays, is to make the exception meaningless or a superfluity, thatis, an exception to a general rule that does not exist, because theprohibition in the enactment clause is not an absolute prohibitionto compel a laborer or employee to work during Sundays and legal holidays. The prohibition to compel a laborer or employee to workduring those days is qualified by the clause "unless he is paid anadditional sum of at least twenty five per centum of his regular remuneration," which is inseparable from the prohibition whichthey qualify and of which they are a part and parcel. The secondportion of section 1 is in reality an exception and not a provisoalthough it is introduced by the word "provided"; and it is elementalthat an exception takes out of an enactment something which wouldotherwise be part of the subject matter of it. To construe section 4, Commonwealth Act No. 444, as exempting public utilities, like the appellant, from the obligation to pay the additional remuneration required by said section 4 should they compel their employees or laborers to work on Sundays and legalholidays, would not make such

exception a class legislation, violative of the constitutional guaranty of equal protectionof the laws (section 1 [1] Art. III of our Constitution). For itis a well-settled rule in constitutional law that a legislation which affects with equal force all persons of the same class and notthose of another, is not a class legislation and does not infringesaid constitutional guaranty of equal protection of the laws, if thedivision into classes is not arbitrary and is based on differenceswhich are apparent and reasonable. (Magonn vs. Illinois Trust Savings Bank, 170 N. S., 283, 294; State vs. Garbroski, 111 Iowa, 496; 56 L. R. A., 570.) And it is evident that the division made by section 4, of Commonwealth Act No. 444, of persons, firms, and corporations into two classes: one composed of public utilities performing somepublic service such as supplying gas, electricity, power, water orproviding means of transportation; and another composed of persons,firms, and corporations which are not public utilities and do notperform said public service , is not arbitrary and is based ondifferences which are apparent and reasonable. The division is not arbitrary, and the basis thereof is reasonable. Public utilities exempted from the prohibition set forth in the enactment clause of section 4, Commonwealth Act No. 444, are required to perform a continuous service including Sundays andlegal holidays to the public, since the public good so demands,and are not allowed to collect an extra charge for services performed on those days; while the others are not required to do so and are free to operate or not their shops, business, or industries on Sundays and legal holidays. If they operate andcompel their laborers to work on those days it is but just andnatural that they should pay an extra compensation to them, because it is to be presumed that they can make money or business by operating on those days even if they have to pay such extra remuneration. It would be unfair for the law to compel publicutilities like the appellant to pay an additional or extra compensation to laborers whom they have to compel to work duringSundays and legal holidays, in order to perform a continuous service to the public. To require public utilities performingservice to do so, would be tantamount to penalize them forperforming public service during said days in compliance withthe requirement of the law and public interest.

The conclusion on which the dissenting opinion is based, which is alsosubstantially the basis of the resolution of the lower court, is that "As to them [referring to public utilities like the petitioner] section4 of Commonwealth Act No. 444 may be considered as not having been enacted at all. . . . Therefore, when there is a labor dispute as in the present case, and the dispute is submitted to the Court of Industrial Relations for decision or settlement, the court is free to providewhat it may deem just and more beneficial to the interested parties,and that freedom to settle and decide the case certainly includesthe power to grant additional compensation to workers who work onSundays and holidays. The general power granted by section 1, 4, and13 of Commonwealth Act No. 103, are not affected in any way or senseby section 4 of Commonwealth Act No. 444." This conclusion finds no support in law, reason or logic. It is a well settled rule of statutory construction adopted by courts of last resort in the States that if one statute enacts some thing in general terms,and afterwards another statute is passed on the same subject, whichalthough expressed in affirmative language introduces special conditions or restrictions, the subsequent statute will usually be considered asrepealing by implication the former regarding the matter covered by the subsequent act; and more specially so when the latter act is expressed in negative terms , as where for example it prohibits a certain thing for being done, or where it declares that a given act shall be performed in a certain manner and not otherwise. (See Black on Interpretation of Laws, 2d ed., p. 354, and Sutherland, Statutory Construction, 3d ed., Vol. 1, section 1922, and cases therein cited.) In accordance with this rule, the provision of Commonwealth Act No. 103 which confers upon the Court of Industrial Relations power to settle dispute between employers and employees in general, including those relating to compulsion of laborers to work on Sundays and legalholidays and additional compensation for those working on those days,should be considered as impliedly repealed by section 4 of Act No. 444,which limits or restricts the minimum of the additional compensationand specifies the persons, firms or corporation who may be requered to pay said compensation. That is, that the Court of

Industrial Relations may, under the provision of said section 4, order a person, firm orcorporation or business establishment or place or center of labor whocompel an employee or laborer to work on Sundays and legal holidays,to pay him an additional compensation of at least 25 per centum of his regular remuneration; but said court can not require public utilities performing public service mentioned therein to pay saidextra compensation to laborers and employees required by them towork on Sundays and legal holidays, because the necessity of publicservice so requires.lawphil.net It is evident that the principal purpose of the Legislature in enactingsaid section 4, is not only to restrict the general power of the Court of Industrial Relations granted by Act No. 103, to fix the minimumadditional compensation which an employer may be required to pay a laborer compelled to work on those days, but principally to exemptpublic utilities affected with public interest, from the payment ofsuch additional compensation. If it were the intention of the lawmakersin enacting section 4 of the Act No. 444 to fix the limit of the minimum of additional compensation of laborers working on those days, withoutexempting the public utilities, that is, leaving intact the general power of the court to require the public utilities to pay said additional compensation, the law would have only provided, in substance, that allemployers are prohibited from compelling their laborers to work onSundays and legal holidays without paying them an additional compensationof not less than 50 per cent of their regular remuneration. That the intention of the Legislature is to exempt the public utilitiesunder consideration from the prohibition set forth in the enactmentclause of section 4, Act No. 444, is supported by the provision ofsection 19 of Act No. 103. As amended this section provides "that with exception of employers engaged in the operation of public services orin the business coupled with a public interest, employers will notbe allowed to engage the services of the strike breakers within fifteendays after the declaration of the strike; which shows a contrario sensu that public utilities performing public services are permitted to engage the services of strike breakers within fifteen days, that is,immediately upon the declaration of the strike. The same public interest, the reason of the exception in the

above quoted provision, underlies the exception provided in section 4, of Act No. 444. Therefore, the ruling of the Court of Industrial Relations quoted in the first part of this decision appealed from, being contrary to law, is set aside. So ordered. National Development Company and New Agrix vs. Philippine Veterans Bank (192 SCRA 257) Facts: Agrix Marketing executed in favor of respondent a real estate mortgage over three parcels of land. Agrix later on went bankrupt. In order to rehabilitate the company, then President Marcos issued Presidential Decree 1717 which mandated, among others, the extinguishing of all the mortgages and liens attaching to the property of Agrix, and creating a Claims Committee to process claims against the company to be administered mainly by NDC. Respondent thereon filed a claim against the company before the Committee. Petitioners however filed a petition with the RTC of Calamba, Laguna invoking the provision of the law which cancels all mortgage liens against it. Respondent took measures to extrajudicially foreclose which the petitioners opposed by filing another case in the same court. These cases were consolidated. The RTC held in favor of the respondent on the ground of unconstitutionality of the decree; mainly violation of the separation of powers, impairment of obligation of contracts, and violation of the equal protection clause. Hence this petition. Issue: Is the respondent estopped from questioning the constitutionality of the law since they first abided by it by filing a claim with the Committee? Is PD 1717 unconstitutional?

government institutions when he was the President. The creation of New Agrix as mandated by the decree was also ruled as unconstitutional since it violated the prohibition that the Batasang Pambansa (Congress) shall not provide for the formation, organization, or regulation of private corporations unless such corporations are owned or controlled by the government. PD 1717 was held as unconstitutional on the other grounds that it was an invalid exercise of police power, It had no lawful subject and no lawful method. It violated due process by extinguishing all mortgages and liens and interests which are property rights unjustly taken. It also violated the equal protection clause by lumping together all secured and unsecured creditors. It also impaired the obligation of contracts, even though it only involved purely private interests. Gutierrez v Cantadda The obstacle to the reversal of a decision of respondent Judge Santiago O. Taada dismissing an ejectment suit against private respondents 1 in this petition for certiorari by way of review comes from police power legislation, the first Republic Act No. 6539 and thereafter Presidential Decree No. 20. They had a common objective to remedy the plight of the lessees, Presidential Decree No. 20, moreover, having a constitutional sanction in that it is specifically referred to in the fundamental law as part of "the law of the land." 2Under the former statute, actions for ejectment were "suspended from two years from the effectivity" thereof. 3It took effect on July 14, 1971. The complaint for ejectment in this litigation was filed on April 22, 1972. Such a period was made indefinite by Presidential Decree No. 20 thus: "Except when the lease is for a definite period, the provisions of paragraph (1) of Article 1673 of the Civil Code of the Philippines insofar as they refer to dwelling unit or land on which another's dwelling is located shall be suspended until otherwise provided; but other provisions of the Civil Code and the Rules of Court of the Philippines on lease contracts, insofar as they are not in conflict with the provisions of this Act, shall apply ." 4Under paragraph (1) of Article 1673 of the Civil Code, one of the grounds for judicially ejecting the lessee is the expiration of the period fixed for the

duration of the lease. law library

chanrobles virtual

Petitioners in this proceeding were the plaintiffs in Civil Case No. 8805 for ejectment filed with the City Court of Caloocan City on April 27, 1972, against a certain Benjamin Leyva. With leave of court, and within the reglementary period, respondent spouses filed on May 2, 1972 an answer in intervention to the complaint. In such pleading, they admitted that plaintiffs, now petitioners, are the owners of the land on which their house is built, including as an affirmative defense the fact that their life savings were spent for the purchase of that house, a transaction entered into only because they were given the assurance by petitioners as lessors that they would be allowed to remain for a substantial number of years. After trial duly had in the City Court of Caloocan City, a decision in said ejectment action, in favor of the petitioners and against respondent spouses was promulgated on September 21, 1972. Respondent spouses on the same day appealed to the Court of First Instance of Rizal, Caloocan City Branch XXXIII, presided over by respondent Judge. On February 14, 1973, respondent Judge rendered a decision on the case, reversing the City Court of Caloocan City, Branch II, dismissing the complaint for ejectment.chanroblesvirtualawlibrary chanro bles virtual law library In such decision, respondent Judge stated the following. "From the evidence thus adduced, it is unmistakably clear that this case comes under the coverage of the rental law of 1971, Republic Act No. 6359. Here, the rental stipulated is only for P30.00 or P40.00 a month for the occupancy of defendant-intervenor on plaintiffs' property. The residential house of said intervenor is involved. There is an agreement as to use of the premises between the plaintiffs and the defendants-intervenors. It cannot be claimed that the plaintiffs were not informed nor that did they know about the occupancy of the intervenors on their property or intervenors' having purchased the house of their (plaintiffs') own collector. It is hard to believe that they did not know until after almost one and a half years from the purchase of the house that it is intervenor Armando Cantada, and not anymore Benjamin Leyva nor Jose Chaingan who is actually paying rentals therefor. After all a house with a value of P8,000.00 could not

Ruling:

On the issue of estoppel, the Court held that it could not apply in the present case since when the respondent filed his claim, President Marcos was the supreme ruler of the country and they could not question his acts even before the courts because of his absolute power over all

just be ignored, and more, no person would just part with P8,000.00 for a house standing on another's property without the assurance or at least the knowledge of the latter." 6 chanrobles virtual law library Respondent Judge also pointed out: "'There is nothing in the complaint insinuating, or in the evidence which shows that the grounds for ejectment as enumerated in Article 1673 are present. In fact, the evidence shows that intervenors are up-to-date in their payment of their monthly rentals, at least before the filing of the complaint. There is no question, either, that the property leased from plaintiffs is used for the intervenors for residential purposes, as was tacitly agreed upon by the parties, or at least permitted by the plaintiffs. The only ground, therefore, apparently available to the plaintiffs to justify their action is the fact that the duration of the lease as fixed by Article 1687 has already expired, or expires each month and may be terminated at the end of said month. But this is excepted as ground for ejectment by Section 4 of Republic Act No. 6359.chanroblesvirtualawlibrary chanrobles virtual law library Under said section, the provision of the first paragraph of Article 1673 is suspended when the period of the lease has not been agreed upon but has been only fixed in accordance with Article 1687 and said period has expired. The period of the suspension of said provision is for 2 years from July 14, 1971, the date of the effectivity of the Act. This action was filed on April 27, 1972, well within the period of the suspension of the provision authorizing ejectment by judicial action. 7 After a motion for reconsideration failed, this petition for certiorari was filed. It should be noted that in the opposition to such motion, Presidential Decree No. 20 which suspended court actions for ejectment for leases covered by the same was relied upon by private respondents. 8 chanrobles virtual law library As noted at the outset, the reliance of the decision sought to be reviewed was on the previous statute and subsequently Presidential Decree No. 20, both police power legislation intended to remedy the situation of the lessees. Hence the weakness of this petition. We find for private respondents.chanroblesvirtualawlibrary chan robles virtual law library

1. On the facts as found by the lower court to which no objection could be raised by petitioners as they brought the certiorari proceeding directly to this Tribunal, the decision arrived at is free from the taint of any infirmity. When the ejectment suit was filed on April 22, 1972, Republic Act No. 6359 was in full force and effect. As noted earlier, for a period of two years from July 14, 1971, the right of the lessees to remain could not be disputed for as found by the lower court the lease was not for a definite period. Hence the reversal of the decision of the City Court of Caloocan City by respondent Judge in his decision of February 14, 1973. As a matter of fact, as of that date, Presidential Decree No. 20 was in full force and effect. The suspension of actions for ejectment was for an indefinite period. Inasmuch as it is a police power legislation, it was applicable to leases entered into prior to that date. The applicability thereof to existing contracts cannot be denied. From Pangasinan transportation Co. v. Public Service Commission, 9 such a doctrine has been repeatedly adhered to by this Court. As was held in Ongsiako v. Gamboa, 10 decided in 1950, a police power measure being remedial in character covers existing situations; otherwise, it would be self-defeating. In Abe v. Foster Wheeler Corp., 11Justice Barrera speaking for the Court, took note of the contention "that as the contracts of employment were entered into at a time when there was no law granting the workers said right, the application as to them of the subsequent enactment restoring the same right constitutes an impairment of their contractual obligations." Then he made clear why the Court was of a contrary view as "the constitutional guaranty of nonimpairment ... is limited by the exercise of the police power of the State, in the interest of public health, safety, morals and general welfare " 12So it must be in this case.chanroblesvirtualawlibrary chanrobles virtual law library 2. The futility of this petition is thus apparent. A belated attempt by counsel for petitioners by raising a constitutional question is equally unavailing. He would have this Court declare that Republic Act No. 6539 is violative of the equal protection clause. The imputation that a Police power measure of that character intended to remedy the deplorable situation of lessees 13 suffers from such infirmity, is far from persuasive, It cannot be stigmatized as class legislation. There was a clear need for

such a statute. It was enacted to promote the public interest and the general welfare. The State is not compelled to stand Idly by while a considerable segment of its citizens suffers from economic distress. Only recently, in Agustin v. Edu, 14 reference was made to the "broad and expansive scope of police power" citing Chief Justice Taney of the American Supreme Court in an 1847 decision as "nothing more or less than the powers of government inherent in every sovereignty " 15 Correctly was it characterized by Justice Malcolm as "that most essential, insistent, and illiminate of powers." 16Moreover, there is a procedural objection to this last-ditch effort, the wellsettled rule being that the constitutional question should be raised at the earliest opportunity. 17Nor does this case falls within one of the exceptions mentioned in People v. Vera. 18 Even if success could have attended this maneuver, still there is the insuperable barrier posed by Presidential Decree No. 20 which was relied upon in the objection of private respondents to the motion for reconsideration. It was therefore within the cognizance of respondent Judge, as it ought to have been all the while. Petitioners apparently had nothing to say about this Presidential Decree. They ought not to have been surprised at its being applied being a part "of the law of the land." To repeat, petitioner had failed to make out a case for reversal.chanroblesvirtualawlibrary chanrobl es virtual law library 3. At any rate, as of April 10, 1979, a new Rent Control Law, Batas Pambansa Blg. 25 was signed by the President and took effect immediately. Petitioners, if falling within the terms thereof, may avail themselves of the benefits of this new legislation chanrobles virtual law library WHEREFORE, the petition for certiorari is dismissed. Costs against petitioners.

Victoriano v Elizalde Rope Workers Union Facts: Plaintiff is a member of the Elizalde Rope Workers Union who later resigned from his affiliation to the said union by reason of the prohibition of his religion for its members to become affiliated with any labor organization. The union has subsisting closed shop agreementin their collective bargaining agreement with their employer that all permanent employees of the company must be a member of the union and later was amended by Republic Act No. 3350 with the provision stating "but such agreement shall not cover members of any religious sects which prohibit affiliation of their members in any such labor organization".. By his resignation, the union wrote a letter to the company to separate the plaintiff from the service after which he was informed by the company that unless he makes a satisfactory arrangement with the union he will be dismissed from the service. The union contends that RA 3350 impairs obligation of contract stipulated in their CBA and discriminatorily favors religious sects in providing exemption to be affiliated with any labor unions. Issue: WON RA 3350 impairs the right to form association. Held: The court held that what the Constitution and the Industrial Peace Act recognize and guarantee is the "right" to form or join associations which involves two broad notions, namely: first, liberty or freedom, i.e., the absence of legal restraint, whereby an employee may act for himself without being prevented by law; and second, power, whereby an employee may join or refrain from joining an association. Therefore the right to join a union includes the right to abstain from joining any union. The exceptions provided by the assailed Republic Act is that members of said religious sects cannot be compelled or coerced to join labor unions even when said unions have closed shop agreements with the employers; that in spite of any closed shopagreement, members of said religious sects cannot be refused employment or dismissed from their jobs on the sole ground that they are not members of the collective bargaining union. Thus this exception does not infringe upon the constitutional provision on freedom of association but instead reinforces it.

Phil. Assoc. of Service Exporters inc v Drilon This case involved the constitutionality of Department Order No. 1 of the DOLE suspending the deployment of Filipino Domestic Workers. It was questioned on the ground that it discriminated against women. In this case, the Supreme Court upheld the validity of the order. Facts: The Philippine Association of Service Exporters, Inc. (PASEI) challenged the decision of the Secretary of Labor and Employment to enforce Department Order No. 1 (D.O. No. 1), which Order provided for an indefinite ban on the deployment of Filipina workers abroad. PASEI claimed that D.O. No. 1 offended equal protection guarantees by discriminating against domestic helpers and Filipinas with similar skills, while imposing no such similar ban on male workers or female workers engaged in other trades. Held: The concept of police power is wellestablished in this jurisdiction. It has been defined as the state authority to enact legislation that may interfere with personal liberty or property in order to promote the general welfare. As defined, it consists of (1) an imposition of restraint upon liberty or property, (2) in order to foster the common good. It is not capable of an exact definition but has been, purposely, veiled in general terms to underscore its all-comprehensive embrace. The petitioner has shown no satisfactory reason why the contested measure should be nullified. There is no question that Department Order No. 1 applies only to female contract workers, but it does not, thereby, make an undue discrimination between the sexes. It is well-settled that equality before the law under the Constitution does not import a perfect identity of rights among all men and women. It admits of classifications, provided that (1) such classifications rest on substantial distinctions; (2) they are germane to the purposes of the law; (3) they are not confined to existing conditions; and (4) they apply equally to all members of the same class. As a matter of judicial notice, the Court is well-aware of the unhappy plight that has befallen our female labor force abroad, especially domestic servants, amid exploitative working conditions marked by, in not a few cases, physical and personal

abuse. The sordid tales of maltreatment suffered by migrant Filipina workers, even rape and various forms of torture, confirmed by testimonies of returning workers, are compelling motives for urgent Government action. The same, however, cannot be said of our male workers. In the first place, there is no evidence that, except perhaps for isolated instances, our men abroad have been afflicted with an identical predicament. The petitioner has proffered no argument that the Government should act similarly with respect to male workers. The Court, of course, is not impressing some male chauvinistic notion that men are superior to women. What the Court is saying is that it was largely a matter of evidence (that women domestic workers are being illtreated abroad in massive instances) and not upon some fanciful or arbitrary yardstick that the Government acted in this case. It is evidence capable indeed of unquestionable demonstration and evidence. The Court cannot, however, say the same thing as far as men are concerned. There is simply no evidence to justify such an inference. Suffice it to state, then, that insofar as classifications are concerned, this Court is content that distinctions are borne by the evidence. Discrimination in this case is justified.

Tablarin v Gutierrez Facts: The petitioners sought to enjoin the Secretary of Education, Culture and Sports, the Board of Medical Education and the Center for Educational Measurement from enforcing Section 5 (a) and (f) of Republic Act No. 2382, as amended, and MECS Order No. 52, series of 1985, dated 23 August 1985 and from requiring the taking and passing of the NMAT as a condition for securing certificates of eligibility for admission, from proceeding with accepting applications for taking the NMAT and from administering the NMAT as scheduled on 26 April 1987 and in the future. The trial court denied said petition on 20 April 1987. The NMAT was conducted and administered as previously scheduled. Republic Act 2382, as amended by Republic Acts Nos. 4224 and 5946, known as the "Medical Act of 1959" defines its basic objectives in the following manner: "SECTION 1. Objectives. This Act provides for and shall govern (a) the standardization and regulation of medical education; (b) theexamination for registration of physicians; and (c) the supervision, control and regulation of the practice of medicine in the Philippines." The statute, among other things, created a Board of Medical Education. Its functions as specified in Section 5 of the statute include the following: "(a) To determine and prescribe requirements for admission into a recognized college of medicine; xxx (f) To accept applications for certification for admission to a medical school and keep a register of those issued said certificate; and to collect from said applicants the amount of twenty-five pesos each which shall accrue to the operating fund of the Board of Medical Education; Section 7 prescribes certain minimum requirements for applicants tomedical schools: "Admission requirements. The medical college may admit any student who has not been convicted by any court of competent jurisdiction of any offense involving moral turpitude and who presents (a) a record of completion of a bachelor's degree in science

or arts; (b) a certificate of eligibility for entrance to a medical school from the Board of Medical Education; (c) a certificate of good moral character issued by two former professors in the college of liberal arts; and (d) birth certificate. Nothing in this act shall be construed to inhibit any college of medicine from establishing, in addition to the preceding, other entrance requirements that may be deemed admissible. MECS Order No. 52, s. 1985, issued by the then Minister of Education, Culture and Sports and dated 23 August 1985, established a uniform admission test called the National Medical Admission Test (NMAT) as an additional requirement for issuance of a certificate of eligibility for admission into medical schools of the Philippines, beginning with the school year 1986-1987. This Order goes on to state that: "2. The NMAT, an aptitude test, is considered as an instrument toward upgrading the selection of applicants for admission into the medical schools and its calculated to improve the quality of medical education in the country. The cutoff score for the successful applicants, based on the scores on the NMAT, shall be determined every year by the Board of Medical Education after consultation with the Association of Philippine Medical Colleges. The NMAT rating of each applicant, together with the other admission requirements as presently called for under existing rules, shall serve as a basis for the issuance of the prescribed certificate of eligibility for admission into the medical colleges. Issue: Whether or not Section 5 (a) and (f) of Republic Act No. 2382, as amended, and MECS Order No. 52, s. 1985 are constitutional. Held: Yes. We conclude that prescribing the NMAT and requiring certain minimum scores therein as a condition for admission to medical schools in the Philippines, do not constitute an unconstitutional imposition. The police power, it is commonplace learning, is the pervasive and non-waivable power and authority of the sovereign to secure and promote all the important interests and needs in a word, the public order of the general community. An important component of that public order is the health and physical safety and well being of the population, the securing of

which no one can deny is a legitimate objective of governmental effort and regulation. Perhaps the only issue that needs some consideration is whether there is some reasonable relation between the prescribing of passing the NMAT as a condition for admission to medical school on the one hand, and the securing of the health and safety of the general community, on the other hand. This question is perhaps most usefully approached by recalling that the regulation of the practice of medicine in all its branches has long been recognized as a reasonable method of protecting the health and safety of the public. MECS Order No. 52, s. 1985 articulates the rationale of regulation of this type: the improvement of the professional and technical quality of the graduates of medical schools, by upgrading the quality of those admitted to the student body of the medical schools. That upgrading is sought by selectivity in the process of admission, selectivity consisting, among other things, of limiting admission to those who exhibit in the required degree the aptitude for medical studies and eventually for medical practice. The need to maintain, and the difficulties of maintaining, high standards in our professional schools in general, andmedical schools in particular, in the current stage of our social and economic development, are widely known. We believe that the government is entitled to prescribe an admission test like the NMAT as a means for achieving its stated objective of "upgrading the selection of applicants into [our] medical schools" and of "improv[ing] the quality ofmedical education in the country. We are entitled to hold that the NMAT is reasonably related to the securing of the ultimate end of legislation and regulation in this area. That end, it is useful to recall, is the protection of the public from the potentially deadly effects of incompetence and ignorance in those who would undertake to treat our bodies and minds for disease or trauma. WHEREFORE, the Petition for Certiorari is DISMISSED and the Order of the respondent trial court denying the petition for a writ of preliminary injunction is AFFIRMED. Costs against petitioners.

Ormoc Sugar Co Inc v Treasurer of Ormoc City Facts: The Municipal Board of Ormoc City passed Ordinance No. 4 imposing on any and all productions of centrifugal sugar milled at the Ormoc Sugar Company, Inc., in Ormoc City a municipal tax equivalent to one per centum (1%) per export sale to USA and other foreign countries. Payments for said tax were made, under protest, by Ormoc Sugar Company, Inc. Ormoc Sugar Company, Inc. filed before the Court of First Instance of Leyte a complaint against the City of Ormoc as well as its Treasurer, Municipal Board and Mayor alleging that the ordinance is unconstitutional for being violative of the equal protection clause and the rule of uniformity of taxation. The court rendered a decision that upheld the constitutionality of the ordinance. Hence, this appeal. Issue: Whether or not constitutional limits on the power of taxation, specifically the equal protection clause and rule of uniformity of taxation, were infringed? Held: Yes. Equal protection clause applies only to persons or things identically situated and does not bar a reasonable classification of the subject of legislation, and a classification is reasonable where 1) it is based upon substantial distinctions; 2) these are germane to the purpose of the law; 3) the classification applies not only to present conditions, but also to future conditions substantially identical to those present; and 4) the classification applies only to those who belong to the same class. A perusal of the requisites shows that the questioned ordinance does not meet them, for it taxes only centrifugal sugar produced and exported by the Ormoc Sugar Company, Inc. and none other. The taxing ordinance should not be singular and exclusive as to exclude any subsequently established sugar central for the coverage of the tax.

*********SEARCHES AND SEIZURES Placer v Villanueva Facts: Petitioners filed informations in the city court and they certified that Preliminary Investigation and Examination had been conducted and that prima facie cases have been found. Upon receipt of said informations, respondent judge set the hearing of the criminal cases to determine propriety of issuance of warrants of arrest. After the hearing, respondent issued an order requiring petitioners to submit to the court affidavits of prosecution witnesses and other documentaryevidence in support of the informations to aid him in the exercise of his power of judicial review of the findings of probable cause by petitioners. Petitioners petitioned for certiorari and mandamus to compel respondent to issue warrants of arrest. They contended that the fiscals certification in the informations of the existence of probable cause constitutes sufficient justification for the judge to issue warrants of arrest. Issue: Whether or Not respondent city judge may, for the purpose of issuing warrants of arrest, compel the fiscal to submit to the court the supporting affidavits and other documentary evidence presented during the preliminary investigation. Held: Judge may rely upon the fiscals certification for the existence of probable cause and on the basis thereof, issue a warrant of arrest. But, such certification does not bind the judge to come out with the warrant. The issuance of a warrant is not a mere ministerial function; it calls for the exercise of judicial discretion on the part of issuing magistrate. Under Section 6 Rule 112 of the Rules of Court, the judge must satisfy himself of the existence of probable cause before issuing awarrant of arrest. If on the face of the information, the judge finds no probable cause, he may disregard the fiscals certification and require submission of the affidavits of witnesses to aid him in arriving at the conclusion as to existence of probable cause. Petition dismissed. Soliven v Makasiar Facts: In these consolidated cases, three principal issues were raised: (1) whether or not petitioners were denied

due process wheninformations for libel were filed against them although the finding of the existence of a prima facie case was still under review by the Secretaryof Justice and, subsequently, by the President; and (2) whether or not the constitutional rights of Beltran were violated when respondent RTC judge issued a warrant for his arrest without personally examining the complainant and the witnesses, if any, to determine probable cause. Subsequent events have rendered the first issue moot and academic. On March 30, 1988, the Secretary of Justice denied petitioners' motion for reconsideration and upheld the resolution of the Undersecretary of Justice sustaining the City Fiscal's finding of a prima facie case against petitioners. A second motion for reconsideration filed by petitioner Beltran was denied by the Secretary of Justice on April 7, 1988. On appeal, the President, through the Executive Secretary, affirmed the resolution of the Secretary of Justice on May 2, 1988. The motion for reconsideration was denied by the Executive Secretary on May 16, 1988. With these developments, petitioners' contention that they have been denied the administrative remedies available under the law has lost factual support. Issues: (1) Whether or Not petitioners were denied due process wheninformations for libel were filed against them although the finding of the existence of a prima facie case was still under review by the Secretaryof Justice and, subsequently, by the President. (2) Whether or Not the constitutional rights of Beltran were violated when respondent RTC judge issued a warrant for his arrest without personally examining the complainant and the witnesses, if any, to determine probable cause Held: With respect to petitioner Beltran, the allegation of denial of due process of law in the preliminary investigation is negated by the fact that instead of submitting his counter- affidavits, he filed a "Motion to Declare Proceedings Closed," in effect waiving his right to refute the complaint by filing counter-affidavits. Due process of law does not require that the respondent in a criminal case actually file his counteraffidavits before the preliminary investigation is deemed completed. All that is required is that the respondent be given

the opportunity to submit counter-affidavits if he is so minded. The second issue, raised by petitioner Beltran, calls for an interpretation of the constitutional provision on the issuance of warrants of arrest. The pertinent provision reads: Art. III, Sec. 2. The right of the people to be secure in their persons, houses, papers and effects against unreasonable searches and seizures of whatever nature and for any purpose shall be inviolable, and no search warrant or warrant of arrest shall issue except upon probable cause to be determined personally by the judge after examination nder oath or affirmation of the complainant and the witnesses he may produce, and particularly describing the place to be searched and the persons or things to be seized. The addition of the word "personally" after the word "determined" and the deletion of the grant of authority by the 1973 Constitution to issue warrants to "other responsible officers as may be authorized by law," has apparently convinced petitioner Beltran that the Constitution now requires the judge to personally examine the complainant and his witnesses in his determination of probable cause for the issuance of warrants of arrest. This is not an accurate interpretation. What the Constitution underscores is the exclusive and personal responsibility of the issuing judge to satisfy himself of the existence of probable cause. In satisfying himself of the existence of probable cause for the issuance of a warrant of arrest, the judge is not required to personally examine the complainant and his witnesses. Following established doctrine and procedure, he shall: (1) personally evaluate the report and the supporting documents submitted by the fiscal regarding the existence of probable cause and, on the basis thereof, issue a warrant of arrest; or (2) if on the basis thereof he finds no probable cause, he may disregard the fiscal's report and require the submission of supporting affidavits of witnesses to aid him in arriving at a conclusion as to the existence of probable cause. Sound policy dictates this procedure, otherwise judges would be unduly laden with the preliminary examination and investigation of criminal complaints instead

of concentrating on hearing and deciding cases filed before their courts. It has not been shown that respondent judge has deviated from the prescribed procedure. Thus, with regard to the issuance of the warrants of arrest, a finding of grave abuse of discretion amounting to lack or excess of jurisdiction cannot be sustained. The petitions fail to establish that public respondents, through their separate acts, gravely abused their discretion as to amount to lack of jurisdiction. Hence, the writs of certiorari and prohibition prayed for cannot issue. WHEREFORE, finding no grave abuse of discretion amounting to excess or lack of jurisdiction on the part of the public respondents, the Court Resolved to DISMISS the petitions in G. R. Nos. 82585, 82827 and 83979. The Order to maintain the status quo contained in the Resolution of the Court en banc dated April 7, 1988 and reiterated in the Resolution dated April 26, 1988 is LIFTED.

HELD: Yes, mere affidavits of the complainant and his witnesses are thus not sufficient. The examining Judge has to take depositions in writing of the complainant and the witnesses he may produce and attach them to the record. Such written deposition is necessary in order that the Judge may be able to properly determine the existence or non-existence of the probable cause, to hold liable for perjury the person giving it if it will be found later that his declarations are false. We, therefore, hold that the search warrant is tainted with illegality by the failure of the Judge to conform with the essential requisites of taking the depositions in writing and attaching them to the record, rendering the search warrant invalid. (See Rule 126, Sec 4) The respondent judge also declared that he "saw no need to have applicant Quillosa's deposition taken considering that he was applying for a search warrant on the basis of the information provided by the witnesses whose depositions had already been taken by the undersigned. In other words, the applicant was asking for the issuance of the search warrant on the basis of mere hearsay and not of information personally known to him, as required by settled jurisprudence.

February1988 after close surveillance for 3 month of the CID in Pagsanjan, Laguna. 17 of the arrested aliens opted for selfdeportation. One released for lack of evidence, another charged not for pedophile but working with NO VISA, the 3 petitioners chose to face deportation proceedings. On 4 March1988, deportation proceedings were instituted against aliens for being undesirable aliens under Sec.69 of Revised Administrative Code. Warrants of Arrest were issued 7March1988 against petitioners for violation of Sec37, 45 and 46 of Immigration Act and sec69 of Revised Administrative Code. Trial by the Board of Special Inquiry III commenced the same date. Petition for bail was filed 11March 1988 but was not granted by the Commissioner of Immigration. 4 April1988 Petitioners filed a petition for Writ of Habeas Corpus. The court heard the case on oral argument on 20 April 1988. Issues: (1) Whether or Not the Commissioner has the power to arrest and detain petitioners pending determination of existence of probable cause. (2) Whether or Not there was unreasonable searches and seizures by CID agents. (3) Whether or Not the writ of Habeas Corpus may be granted to petitioners. Held: While pedophilia is not a crime under the Revised Penal Code, it violates the declared policy of the state to promote and protect the physical, moral, spiritual and social well being of the youth. The arrest of petitioners was based on the probable cause determined after close surveillance of 3 months. The existence of probable cause justified the arrest and seizure of articles linked to the offense. The articles were seized as an incident to a lawful arrest; therefore the articles are admissible evidences (Rule 126, Section12 of Rules on Criminal Procedure). The rule that search and seizures must be supported by a valid warrant of arrest is not an absolute rule. There are at least three exceptions to this rule. 1.) Search is incidental to the arrest. 2.) Search in a moving vehicle. 3.) Seizure of evidence in plain view. In view of the foregoing, the

Roan v Gonzales FACTS: The challenged search warrant was issued by the respondent judge on May 10, 1984. The petitioner's house was searched two days later but none of the articles listed in the warrant was discovered. However, the officers conducting the search found in the premises one Colt Magnum revolver and eighteen live bullets which they confiscated. They are now the bases of the charge against the petitioner. Respondent Judge said that when PC Capt. Mauro P. Quinosa personally filed his application for a search warrant on May 10, 1984, he appeared before him in the company of his two (2) witnesses, Esmael Morada and Jesus Tohilida, both of whom likewise presented to him their respective affidavits taken by Pat. Josue V. Lining, a police investigator. As the application was not yet subscribed and sworn to, he proceeded to examine Captain Quillosa on the contents thereof to ascertain, among others, if he knew and understood the same. Afterwards, he subscribed and swore to the same before him. ISSUE: Whether the Respondent Judge failed to comply with the proper procedure in issuing the Search Warrant.

Harvey v Santiago Facts: This is a petition for Habeas Corpus. Petitioners are the following: American nationals Andrew Harvey, 52 and Jonh Sherman 72. Dutch Citizen Adriaan Van Den Elshout, 58. All reside at Pagsanjan Laguna respondent Commissioner Miriam Defensor Santiago issued Mission Orders to the Commission of Immigration and Deportation (CID) to apprehended petitioners at their residences. The Operation Report read that Andrew Harvey was found together with two young boys. Richard Sherman was found with two naked boys inside his room. While Van Den Elshout in the after Mission Report read that two children of ages 14 and 16 has been under his care and subjects confirmed being live-in for sometime now. Seized during the petitioners apprehension were rolls of photo negatives and photos of suspected child prostitutes shown in scandalous poses as well as boys and girls engaged in sex. Posters and other literature advertising the child prostitutes were also found. Petitioners were among the 22 suspected alien pedophiles. They were apprehended 17

search done was incidental to the arrest. The filing of the petitioners for bail is considered as a waiver of any irregularity attending their arrest and estops them from questioning its validity. Furthermore, the deportation charges and the hearing presently conducted by the Board of Special Inquiry made their detention legal. It is a fundamental rule that habeas corpus will not be granted when confinement is or has become legal, although such confinement was illegal at the beginning. The deportation charges instituted by the
Commissioner of Immigration are in accordance with Sec37 (a) of the Philippine Immigration Act of 1940 in relation to sec69 of the Revised Administrative code. Section 37 (a) provides that aliens shall be arrested and deported upon warrant of the Commissioner of Immigration and Deportation after a determination by the Board of Commissioners of the existence of a ground for deportation against them. Deportation proceedings are administrative in character and never construed as a punishment but a preventive measure. Therefore, it need not be conducted strictly in accordance with ordinary Court proceedings. What is essential is that there should be a specific charge against the alien intended to be arrested and deported. A fair hearing must also be conducted with assistance of a counsel if desired. Lastly, the power to deport aliens is an act of the State and done under the authority of the sovereign power. It a police measure against the undesirable aliens whose continued presence in the country is found to be injurious to the public good and tranquility of the people.

her stenographic notes; and thereafter, J Ruiz asked respondent Logronio to take the oath and warned him that if his deposition was found to be false and without legal basis, he could be charged for perjury. J Ruiz signed de Leons application for search warrant and Logronios deposition. The search was subsequently conducted. ISSUE: Whether or not there had been a valid search warrant. HELD: The SC ruled in favor of Bache on three grounds.
1. J Ruiz failed to personally examine the complainant and his witness. Personal examination by the judge of the complainant and his witnesses is necessary to enable him to determine the existence or nonexistence of a probable cause. 2. The search warrant was issued for more than one specific offense. The search warrant in question was issued for at least four distinct offenses under the Tax Code. As ruled in Stonehill Such is the seriousness of the irregularities committed in connection with the disputed search warrants, that this Court deemed it fit to amend Section 3 of Rule 122 of the former Rules of Court that a search warrant shall not issue but upon probable cause in connection with one specific offense. Not satisfied with this qualification, the Court added thereto a paragraph, directing that no search warrant shall issue for more than one specific offense. 3. The search warrant does not particularly describe the things to be seized. The documents, papers and effects sought to be seized are described in the Search Warrant Unregistered and private books of accounts (ledgers, journals, columnars, receipts and disbursements books, customers ledgers); receipts for payments received; certificates of stocks and securities; contracts, promissory notes and deeds of sale; telex and coded messages; business communications, accounting and business records; checks and check stubs; records of bank deposits and withdrawals; and records of foreign remittances, covering the years 1966 to 1970. The description does not meet the requirement in Art III, Sec. 1, of the Constitution, and of Sec. 3, Rule 126 of the Revised Rules of Court, that the warrant should particularly describe the things to be seized. A search warrant may be said to particularly describe the things to be seized when the description therein is as specific as the circumstances will ordinarily allow or when the description expresses a conclusion of fact not of law by which the warrant officer may be guided in making the search and seizure or when the things described are limited to those which bear direct relation to the offense for which the warrant is being issued.

Prudentev Dayrit FACTS: The Chief of the Intelligence Special Action Division (ISAD) filed with the Regional Trial Court (RTC) Manila, Judge Abelardo Dayrit, for the issuance of Search Warrant for violation of PD No. 1866 (Illegal Possession of Firearm, etc). In the deposition of witness (P/Lt. Florencio C. Angeles), it was made mentioned of result of our continuous surveillance conducted for several days. We gathered information from verified sources that the holders of said firearms and explosives as well as ammunitions arent licensed to possess said firearms and ammunition. Further, the premises is a school and the holders of these firearms are not student who were not supposed to possess firearms, explosives and ammunitions. Person to be searched in Nemesio Prudente at the Polytechnic University of the Philippines, Sta. Mesa, Sampaloc, Manila, has in his control or possession firearms, explosives hand grenades and ammunitions which are illegally possesses at the office of Department of Military Science and Tactics and at the office of the President. Petitioner moved to quash the Search Warrant. He claimed that: 1. Petitioners, had no personal knowledge of the facts 2. The examination of the said witness was not in form of searching questions and answers 3. Search warrant was a general warrant 4. Violation of Circular No. 19 of the Supreme Court in that the complainant failed to allege under oath that the issuance of the search warrant on a Saturday, urgent. ISSUE: Whether or not the search and seizure was valid? HELD: Search Warrant annulled and set aside. RATIONALE: Valid search warrant to issue, there must be probable cause, which is to be determined personally by the Judge, after examination under oath and affirmation of the complainant, and that witnesses he may produce and particularly describing the place to be searched and the persons and things to be seized. The probable cause must be in connection with one specific

offense and the Judge must, before issuing Search Warrant, personally examine in the form of searching questions and answers, In writing and under oath, the complainant and any witnesses he may produce, on facts personally known to them and attach to the record their sworn statements together with any affidavits submitted. Probable Cause for a valid search warrant, has been defined as such facts and circumstances which would lead a reasonably discreet and prudent man to believe that an offense has been committed, and that objects sought in connection which the offense are in the place sought to be searched. This probable case must be shown to be personal knowledge and of the complainant and witnesses he may produce and not based on mere hearsay. PARTICULARITY For violation of PD 1866 (Illegal Possession of Firearms, etc.) while the said decree punishes several offenses, the alleged violation in this case was, qualified by the phrase illegal possession of firearms etc. - Reformed to ammunitions and explosives. In other words, the search warrant was issued for the specific offense of illegal possession of firearms and explosives. Hence, the failure of the Search Warrant to mention the particular provision of PD1-866 that was violated is not of such gravity as to call for the invalidation of this case. Guazon v De Villa Facts: The 41 petitioners alleged that the "saturation drive" or "aerial target zoning" that were conducted in their place (Tondo Manila) were unconstitutional. They alleged that there is no specific target house to be search and that there is no search warrant or warrant of arrestserved. Most of the policemen are in their civilian clothes and without nameplates or identification cards. The residents were rudely rouse from their sleep by banging on the walls and windows of their houses. The residents were at the point of high-powered guns and herded like cows. Men were ordered to strip down to their briefs for the police to examine their tattoo marks. The residents complained that they're homes were ransacked, tossing their belongings and destroying their valuables. Some of their money and valuables had disappeared after the operation. The residents also reported incidents of maulings, spot-

Bache & Co v Ruiz On 24 Feb 1970, Commissioner Vera of Internal Revenue, wrote a letter addressed to J Ruiz requesting the issuance of a search warrant against petitioners for violation of Sec 46(a) of the NIRC, in relation to all other pertinent provisions thereof, particularly Sects 53, 72, 73, 208 and 209, and authorizing Revenue Examiner de Leon make and file the application for search warrant which was attached to the letter. The next day, de Leon and his witnesses went to CFI Rizal to obtain the search warrant. At that time J Ruiz was hearing a certain case; so, by means of a note, he instructed his Deputy Clerk of Court to take the depositions of De Leon and Logronio. After the session had adjourned, J Ruiz was informed that the depositions had already been taken. The stenographer read to him

beatings and maltreatment. Those who were detained also suffered mental and physical torture to extract confessions and tacticalinformations. The respondents said that such accusations were all lies. Respondents contends that the Constitution grants to government the power to seek and cripple subversive movements for the maintenance of peace in the state. The aerial target zoning were intended to flush out subversives and criminal elements coddled by the communities were the said drives were conducted. They said that they have intelligently and carefully planned months ahead for the actual operation and that local and foreign media joined the operation to witness and record such event. Issue: Whether or Not the saturation drive committed consisted of violation of human rights. Held: It is not the police action per se which should be prohibited rather it is the procedure used or the methods which "offend even hardened sensibilities" .Based on the facts stated by the parties, it appears to have been no impediment to securing search warrants or warrants of arrest before any houses were searched or individuals roused from sleep were arrested. There is no showing that the objectives sought to be attained by the "aerial zoning" could not be achieved even as th rights of the squatters and low income families are fully protected. However, the remedy should not be brought by a tazpaer suit where not one victim complaints and not one violator is properly charged. In the circumstances of this taxpayers' suit, there is no erring soldier or policeman whom the court can order prosecuted. In the absence of clear facts no permanent relief can be given. In the meantime where there is showing that some abuses were committed, the court temporary restraint the alleged violations which are shocking to the senses. Petition is remanded to the RTC of Manila

Pita v CA FACTS: Pursuant to the Anti-Smut Campaign of Mayor Ramon Bagatsng, policemen seized and confiscated from dealers, distributors, newsstand owners and peddlers along Manila sidewalks, magazines, publications and other reading materials believed to be obscene, pornographic, and indecent and later burned the seized materials in public. Among the publications seized and later burned was "Pinoy Playboy" magazines published and co-edited by plaintiff Leo Pita. After his injunctive relief was dismissed by the RTC and his appeal rejected by CA, he seeks review with SC, invoking the guaranty against unreasonable searches and seizure. Issue: W/N the search and seizure was illegal HELD: YES. It is basic that searches and seizure may be done only through a judicial warrant , otherwise, they become unreasonable and subject to challenge. In Burgos v Chief of Staff (133 SCRA 800) , the SC countermanded the orders of the RTC authorizing the serach of the premises WE Forum and Metropolitan Mail, two Metro Manila Dailies, by reason of a defective warrant. There is a greater reason in this case to reprobate the questioned raid, in the complete absence of a warrant, valid or invalid. The fact that the instant case involves an obscenity rap makes it no different from Burgos, a political case, because speech is speech, whether political or "obscene". The authorities must apply for the issuance of the a search warrant from the judge , if in their opinion, an obscenity rap is in order. They must convince the court that the materials sought to be seized are "obscene" and pose a clear and present danger of an evil substantive enough to warrant State interference and action. The judge must determine WON the same are indeed "obscene": the question is to be resolved on a case-to-case basis and on the judge's sound discretion. If probable cause exist, a search warrant will issue.

Valmonte v de Villa F: On 1/20/87, the NCRDC was activated w/ the mission of conducting security operations w/in its area or responsibility and peripheral areas, for the purpose of establishing an effective territorial defense, maintaining peace and order, and providing an atmosphere conducive to the social, economic and political devt of the NCR. As part of its duty to maitain peace and order, the NCRDC installed checkpoints in various parts of Valenzuela and MM. Petitioners aver that, bec. of the institution of said checkpoints, the Valenzuela residents are worried of being harassed and of their sarety being placed at the arbitrary, capricious and whimsical disposition of the military manning the checkpoints, considering that their cars and vehicles are being subjected to regular searches and check-ups, especially at night or at dawn, w/o a SW and/ or court order. Their alleged fear for their safety increased when Benjamin Parpon, was gaunned down allegedly in cold blood by members of the NCRDC for ignoring and/ or continuing to speed off inspite of warning shots fired in the air. HELD: Petitioners concern for their safety and apprehension at being harassed by the military manning the checkpoints are not sufficient grounds to declare the checkpoints per se, illegal. No proof has been presented before the Court to show that, in the course of their routine checks, the military, indeed, committed specific violations of petitioners rights against unlawful search and seizure of other rights. The constitutional right against unreasonable searches and seizures is a personal right invocable only by those whose rights have been infringed, or threatened to be infringed. Not all searches and seizures are prohibited. Those w/c are reasonable are not forbidden. The setting up of the questioned checkpoints may be considered as a security measure to enable the NCRDC to pursue its mission of establishing effective territorial defense and maintaining peace and order for the benfit of the public. Checkpoints may not also be regarded as measures to thwart plots to destabilize the govt, in the interest of public security. Between the inherent right of the state to protect its existence and promote public welfare and an individuals right against a warrantless search w/c is, however, reasonably conducted, the former should prevail. True, the manning of checkpoints by the military is susceptible of abuse by the military in the same manner

that all governmental power is susceptible of abuse. But, at the cost of occasional inconveninece, discomfort and even irritation to the citizen, the checkpoints during these abnormal times, when conducted w/in reasonable limits, are part of the price we pay for an orderly society and a peaceful community. PP v Burgos Facts: The lower court laid down its decision to the defendant-appellant with the crime of illegal possession of firearm in furtherance of subversion. That in the afternoon of May 13, 1982 and thereabout at Tiguman, Digos, Davao del Sur, Philippines, the accused possessed without the necessary license, permit or authority issued by the proper government agencies, one (1) homemade revolver, caliber .38 with Serial No. 8.69221. The firearm was issued to and used by the accused at his area in Davao del Sur on operations by one Alias Commander Pol for the New People's Army (NPA), a subversive organization organized for the purpose of overthrowing the Government of the Republic of the Philippines through lawless and violent means. Issues: a)Whether or not the arrest of the appellant-accused without any valid warrant is legal and for the search of firearm in his house without valid warrant is lawful? b)Whether he violated Presidential Decree No. 9 in relation to General Orders Nos. 6 & 7? Held: Rule 113, Section 6 of the Rules of Court The trial court justified the arrest of the accused-appelant without any warrant as falling under one of the instances when arrests may be validly made without a warrant. Rule 113, Section 6 of the Rules of Court, provides the exceptions as follows: a) When the person to be arrested has committed, is actually committing, or is about to commit an offense in his presence; b) When an offense has in fact been committed, and he has reasonable ground to believe that the person to be arrested has

committed it; c) When the person to be arrested is a prisoner who has escaped from a penal establishment or place where he is serving final judgment or temporarily confined while his case is pending or has escaped while being transferred from one confinement to another. The Court stated that even if there was no warrant for the arrest of Burgos, the fact that "the authorities received an urgent report of accused's involvement in subversive activities from a reliable source (report of Cesar Masamlok) the circumstances of his arrest, even without judicial warrant, is lawfully within the ambit of Section 6-A of Rule 113 of the Rules of Court and applicable jurisprudence on the matter." In proving ownership of the firearm The constitutional right against selfincrimination under Sec. 20 of Art. IV of the Bill of Rights winch provides that, no person shall be compelled to be a witness against himself. Any person under investigation for the commission of an offense shall have the right to remain silent and to counsel, and to be informed of such right. The accused-appellant was not accorded his constitutional right to be assisted by counsel during the custodial interrogation. The lower court correctly pointed out that the securing of counsel, Atty. Anyog, to help the accused when he subscribed under oath to his statement at the Fiscal's Office was too late. It could have no palliative effect. It cannot cure the absence of counsel at the time of the custodial investigation when the extrajudicial statement was being taken. The judgment of conviction rendered by the trial court is REVERSED and SET ASIDE. The accused-appellant is hereby ACQUITTED, on grounds of reasonable doubt, of the crime with which he has been charged. PP v Malmstedt Facts: In an information filed against the accused- appellant Mikael Malmstead was charged before the RTC of La Trinidad, Benguet, for violation of Section 4, Art. II of Republic Act 6425, as amended, otherwise known as the Dangerous Drugs Act of 1972, as amended. Accused Mikael Malmstedt, a Swedish national, entered the Philippines for the third time in December 1988 as a tourist.

He had visited the country sometime in 1982 and 1985. In the evening of 7 May 1989, accused left for Baguio City. Upon his arrival thereat in the morning of the following day, he took a bus to Sagada and stayed in that place for two (2) days. Then in the 7 in the morning of May 11, 1989, the accused went to Nangonogan bus stop in Sagada. At about 8: 00 o'clock in the morning of that same day (11 May 1989), Captain Alen Vasco, the Commanding Officer of the First RegionalCommand (NARCOM) stationed at Camp Dangwa, ordered his men to set up a temporary checkpoint at Kilometer 14, Acop, Tublay, Mountain Province, for the purpose of checking all vehicles coming from the Cordillera Region. The order to establish a checkpoint in the said area was prompted by persistent reports that vehicles coming from Sagada were transporting marijuana and other prohibited drugs. Moreover,information was received by the Commanding Officer of NARCOM, that same morning that a Caucasian coming from Sagada had in his possession prohibited drugs. The group composed of seven (7) NARCOM officers, in coordination with Tublay Police Station, set up a checkpoint at the designated area at about 10:00 o'clock in the morning and inspected all vehicles coming from the Cordillera Region. The two (2) NARCOM officers started their inspection from the front going towards the rear of the bus. Accused who was the sole foreigner riding the bus was seated at the rear thereof. During the inspection, CIC Galutan noticed a bulge on accused's waist. Suspecting the bulge on accused's waist to be a gun, the officer asked for accused's passport and other identification papers. When accused failed to comply, the officer required him to bring out whatever it was that was bulging on his waist. The bulging object turned out to be a pouch bag and when accused opened the same bag, as ordered, the officer noticed four (4) suspicious-looking objects wrapped in brownpacking tape, prompting the officer to open one of the wrapped objects. The wrapped objects turned out to contain hashish, a derivative of marijuana. Thereafter, accused was invited outside the bus for questioning. But before he alighted from the bus, accused stopped to get two

(2) travelling bags from the luggage carrier. Upon stepping out of the bus, the officers got the bags and opened them. A teddy bear was found in each bag. Feeling the teddy bears, the officer noticed that there were bulges inside the same which did not feel like foam stuffing. It was only after the officers had opened the bags that accused finally presented his passport. Accused was then brought to the headquarters of the NARCOM at Camp Dangwa, La Trinidad, Benguet for further investigation. At the investigation room, the officers opened the teddy bears and they were found to also contain hashish. Representative samples were taken from the hashish found among the personal effects of accused and the same were brought to the PC Crime Laboratory for chemical analysis. In the chemistry report, it was established that the objects examined were hashish. a prohibited drug which is a derivative of marijuana. Thus, an information was filed against accused for violation of the Dangerous Drugs Act. ACCUSEDS DEFENSE During the arraignment, accused entered a plea of "not guilty." For his defense, he raised the issue of illegal search of his personal effects. He also claimed that the hashish was planted by the NARCOM officers in his pouch bag and that the two (2) travelling bags were not owned by him, but were merely entrusted to him by an Australian couple whom he met in Sagada. He further claimed that the Australian couple intended to take the same bus with him but because there were no more seats available in said bus, they decided to take the next ride and asked accused to take charge of the bags, and that they would meet each other at the Dangwa Station. The trial court found the guilt of the accused Mikael Malmstedt established beyond reasonable doubt. Seeking the reversal of the decision of the trial court finding him guilty of the crime charged, accused argues that the search of his personal effects was illegal because it was made without a search warrant and, therefore, the prohibited drugs which were discovered during the illegal search are not admissible as evidence against him.

Issue: Whether or Not the contention of the accused is valid, and therefore the RTC ruling be reversed. Held: The Constitution guarantees the right of the people to be secure in their persons, houses, papers and effects against unreasonable searches and seizures. However, where the search is made pursuant to a lawful arrest, there is no need to obtain a search warrant. A lawful arrest without a warrant may be made by a peace officer or a private person under the following circumstances. Sec. 5 Arrest without warrant; when lawful. A peace officer or a private person may, without a warrant, arrest a person: (a) When, in his presence, the person to be arrested has committed is actually committing, or is attempting to commit an offense; (b) When an offense has in fact just been committed, and he has personal knowledge of facts indicating that the person to be arrested has committed it; and (c) When the person to be arrested is a prisoner who has escaped from a penal establishment or place where he is serving final judgment or temporarily confined while his case is pending, or has escaped while being transferred from one confinement to another. Accused was searched and arrested while transporting prohibited drugs (hashish). A crime was actually being committed by the accused and he was caught in flagrante delicto. Thus, the search made upon his personal effects falls squarely under paragraph (1) of the foregoing provisions of law, which allow a warrantless search incident to a lawful arrest. While it is true that the NARCOM officers were not armed with a search warrant when the search was made over the personal effects of accused, however, under the circumstances of the case, there was sufficient probable cause for said officers to believe that accused was then and there committing a crime. Probable cause has been defined as such facts and circumstances which could lead a reasonable, discreet and prudent man to believe that an offense has been committed, and that the objects sought in connection with the offense are in the place sought to be searched. Warrantless search of the

personal effects of an accused has been declared by this Court as valid, because of existence of probable cause, where the smell of marijuana emanated from a plastic bag owned by the accused, 10 or where the accused was acting suspiciously, 11 and attempted to flee. The appealed judgment of conviction by the trial court is hereby affirmed. Costs against the accused-appellant. PP v Lo Ho Wing Peter Lo , together with co-accused Lim Cheng Huat alias Antonio Lim and Reynaldo Tia, were charged with a violation of the Dangerous Drugs Act, for the transport of metamphetamine hydrochloride, otherwise known as "shabu". The drug was contained in tea bags inside tin cans which were placed inside their luggages. Upon arrival from Hongkong, they boarded the taxis at the airport which were apprehended by CIS operatives. Their luggages were subsequently searched where the tea bags were opened and found to contain shabu. Only Lo and Lim were convicted. Tia was discharged as a state witness, who turned out to be a " deep penetration agent" of the CIS in its mission to bust the drug syndicate . Issue: W/N the search and seizure was legal. HELD: YES That search and seizure must be supported by a valid warrant is not an absolute rule. One of the exceptions thereto is a search of a moving vehicle. The circumstance of the case clearly show that the serach in question was made as regards a moving vehicle. Therefore, a valid warrant was not necessary to effect the search on appellant and his co-accused. It was firmly established from the factual findings of the court that the authorities had reasonable ground to believe that appellant would

Yee Sue Kuy v. Almeda F: By virtue of the sworn application of Almeda, the Chief agent of the Anti-Usury Board, a SW was issued to search the store and premises of the petitioner, accused of violating the Anti-Usury Law. Receipt books, PNs and other articles were seized and retained in the possession of the Anti-Usury Board. ISSUE: W/n the requirements for the issuance of valid SW were complied with. RULING: YES. The applicant, Almeda, in his application, swore that "he made his own personal investigation and ascertained that petitioner is lending money without a license, charging usurious rates." The witness he presentted testified before the judge and swore that he knew that YEE was lending without a license because he personally investigated the victims who secured loans from the petitioner. Their affidavits were sufficient for, thereunder, they could be held liable for perjury. The existence of probable cause has been determined by the justice of the peace before issuing the warrant complained of, as shown in the warrant itself. Charo. Pasion vda. de Garcia v. Locsin F: This is a petition for mandamus presented to secure the annulment of a search warrant (SW) & 2 orders of the resp. judge, & the restoration of certain documents alleged to have been illegally seized by an agent of the Anti-Usury Board. Almeda, an agent of the Anti-Usury Board, obtained from the justice of the peace of Tarlac, Tarlac, a SW, commanding any officer of the law "to search the person, house or store of the petitioner for certain books, lists, chits, receipts, documents & other papers relating to her activities as userer." On the same date, Almeda, accompanied by a captain of the PC, went to the office of the petitioner, and after showing the SW to the petitioner''s bookeeper, Salas, & w/o the presence of the petitioner, who was ill and confined at that time, proceeded w/ the execution thereof. Two packages of records & a locked filing cabinet containing several papers and documents were seized by Almeda and a receipt thereof issued by him to Salas. Separate criminal cases were filed against petitioner. Petitioner demanded the return of the documents seized. Bu motion, pet. challenged the legality of the SW and the devolution of the documents demanded. By

resolution, the resp. judge of CFI denied the petitioner''s motion for the reason that though the SW was illegal, there was a waiver on the part of the petitioner. The resolution of 10/5/37 & the order of 1/3/38 are sought, together w/ the SW, to be nullified in these proceedings. HELD: Freedom from unreasonable searches and seizures is declared a popular right and for a SW to be valid, (1) it must be issued upon probable cause; (2) the probable cause must be determined by the judge himself and not by the applicant or another; (3) in the determination of probable cause, the judge must examine, under oath or affirmation, the complainant and such witnesses as the latter may produce; & (4) the warrant issued must particularly describe the place to be searched and persons or things to be seized. In the instant case, the existence of probable cause was determined not by the judge himself but by the applicant. All that the judge did was to accept as true the affidavit made by agent Almeda. It does not appear that he examined the applicant and his witnesses, if any. Even accepting the description of the prop. to be seized to be sufficient and on the assumption that the receipt issued is sufficiently detailed w/in the meaning of the law, the prop. seized were not delivered to the court w/c issued the warrant, as required by law. Instead, they were turned over to the resp. provincial fiscal & used by him in building up cases against petitioner. Considering that at the time the warrant was issued, there was no case pending against the petitioner, the averment that the warrant was issued primarily for exploration purposes is not w/o basis. IS THERE A WAIVER? No express waiver. IS THERE AN IMPLIED WAIVER? None. Burgos v Chief of Staff Facts: Assailed in this petition for certiorari prohibition and mandamus with preliminary mandatory and prohibitory injunction is the validity of two [2] search warrants issued on December 7, 1982 by Judge Ernani CruzPano of the then CFI of Rizal [Quezon City], under which the premises of the "Metropolitan Mail" and "We Forum" newspapers, respectively, were searched, and office and printing machines, equipment, paraphernalia, motor vehicles and other articles used in the printing, publication and distribution of the said newspapers, as well as numerous papers,

documents, books and other written literature alleged to be in the possession and control of petitioner Jose Burgos, Jr. publisher-editor of the "We Forum" newspaper, were seized. Petitioners further pray that a writ of preliminary mandatory and prohibitory injunction be issued for the return of the seized articles, and that respondents be enjoined from using the articles thus seized as evidence against petitioner Jose Burgos, Jr. and the other accused in Criminal Case No. Q- 022782 of the Regional Trial Court of Quezon City, entitled People v. Jose Burgos, Jr. et al. Issue: Was the closure of WE Forum a case of prior restraint? Ruling: Yes. As heretofore stated, the premises searched were the business and printing offices of the "Metropolitan Mail" and the "We Forum newspapers. As a consequence of the search and seizure, these premises were padlocked and sealed, with the further result that the printing and publication of said newspapers were discontinued. Such closure is in the nature of previous restraint or censorship abhorrent to the freedom of the press guaranteed under the fundamental law, and constitutes a virtual denial of petitioners' freedom to express themselves in print. This state of being is patently anathematic to a democratic framework where a free, alert and even militant press is essential for the political enlightenment and growth of the citizenry. Corro v Lising F: Respondent Judge issued a search warrant for the seizure of articles allegedly used by petitioner in committing the crime of sedition. Seized were printed copies of the Philippine Times, newspaper dummies, typewriters, mimeographing machines and tape recorders, video machines and tapes. The petitioner moved to quash the warrant but his motion was denied. HELD: The statements made in the affidavits are mere conclusions of law and do not satisfy the requirement of probable cause. The language used is all embracing as to include all conceivable words and equipment of petitioner regardless of whether they are legal or illegal. The search warrant under consideration was in the nature of a general warrant which is objectionable.

attempt to bring in contraband and transport within the country. The belief was based on intelligence reports gathered from surveillance activities on the suspected syndicate, of which appellant was touted to be amember. Aside from this, they were also certain as to the expected date and time of arrival of the accused from China via Hongkong. But such knowledge was insufficient to enable them to fulfill the requiremnents for the issuance of a search warrant. Still and all, the important thing is that there was probable cause to conduct the warrantless search, which must still be present in the case.

Olaes v. People, 155 SCRA 486 (1987) F: Petitioners claim that the SW issued by resp. judge was invalid. They also question the extrajudicial confession taken from them without according them the right to assistance of a counsel. The articles seized by virtue of the SW consisting of dried marijuana were admitted as evidence for violation of RA 6425 or Dangerous Drugs Act. RULING: While it is true that the caption of the SW states that it is in connection with "the violation of RA 6425," it is clearly recited in the text thereof that "there is probable cause to believe that Olaes of Olongapo City has in his possession and control, marijuana dried stalks which are suject of the offense stated above." Although the specific section of the law is not stated, there is no question at all that the specific offense alleged to have been committed as basis for determining probable cause is alleged. Furthermore, the SW specifically described the place to be searched and the things to be seized. As to the extrajudicial confessions of the accused, the same are deemed inadmissible against them. In People V. Galit, the Court declared that: " At the time the person is arrested, it shall be the duty of the arresting officer to inform him of the reason for the arrrest and he must be shown the warrant of arrest, if any; he shall be informed of his constitutional rights to remain silent and to counsel, and that any statement he might make could be used against him. The person arrested shall have the right to communicate with his lawyer, a relative, or anyone he chooses by the most expedient means - by telephone if possible - or by letter or messenger. It shall be the responsibility of the arresting officer to see to it that this is accomplished. No custodial investigation shall be conducted unless it be in the presence of counsel engaged by the person arrested, by any person on his behalf, or appointed by the court upon petition either of the detainee himself or by anyone on his behalf. The right to counsel may be waived but the wiaver shall not be valid unless made with the assistance of counsel. Any statement obtained in violation of the procedure herein laid down, whether exculpatory or inculpatory, in whole or in part, shall be inadmissible in evidence."
These requirements were even made stricter under the 1987 Constitution which provides that the rights of a person under custodial investigation cannot be waived except when made in writing and in the presence of counsel.

Presidential Anti-Dollar Salting Task Force v. CA, 171 SCRA 348 (1989) F: The PASTF was created by virtue of PD 1936 to serve as the President''s arm called upon to combat the vice of dollar salting or the blackmarketing and salting of foreign exchange. ISSUE: W/N the PASTF is "such other officer as may be authorized by law" to issue warrants under the 1973 Constitition. RULING: NO. The Court, in reviewing the powers of the PASTF under its enabling law, sees nothing that will reveal a legislative intendement to confer upon the body, quasijudicial responsiibilities relative to offenses punishable by PD 1883. Its undertaking is simply to determine w/n probable cause exists to warrant the filing of charges with the proper court, meaning to say, to conduct an inquiry preliminary to a judicial recourse, and to recommend action of appropriate authorities. The Court agrees that PASTF exercises, or was meant to exercise, prosecutorial powers, and on that ground, it cannot be said to be a neutral and detached judge to determine the existence of probable cause for purposes of arrest or search. Unlike a magistrate, a prosecutor is naturally interested in the success of his case. Although his office "is to see to it that justice if done and not necessarily to secure the conviction of the accused," he stands invariably, as the accused''s adversary and his accuser. To permit him to issue warrrants and indeed, warrants of arrest, is to make him both judge and jury in his own right, when he is neither. This makes to our mind and to that extent, PD 1636 as amended by PD 2002, unconstitutional. The "responsible officer" referred to under the Cosntitution is one not only possessing the necessary skills and competence but more significantly, the neutrality and independence comparable to the impartiality presumed of a judicial officer.

Salazar v Achacoso Facts: Rosalie Tesoro of Pasay City in a sworn statement filed with the POEA, charged petitioner with illegal recruitment. Public respondent Atty. Ferdinand Marquez sent petitioner a telegram directing him to appear to the POEA regarding the complaint against him. On the same day, after knowing that petitioner had no license to operate a recruitment agency, public respondent Administrator Tomas Achacoso issued a Closure and Seizure Order No. 1205 to petitioner. It stated that there will a seizure of the documents and paraphernalia being used or intended to be used as the means of committing illegal recruitment, it having verified that petitioner has (1) No valid license or authority from the Department of Labor and Employment to recruit and deploy workers for overseas employment; (2) Committed/are committing acts prohibited under Article 34 of the New Labor Code in relation to Article 38 of the same code. A team was then tasked to implement the said Order. The group, accompanied by mediamen and Mandaluyong policemen, went to petitioners residence. They served the order to a certain Mrs. For a Salazar, who let them in. The team confiscated assorted costumes. Petitioner filed with POEA a letter requesting for the return of the seized properties, because she was not given prior notice and hearing. The said Order violated due process. She also alleged that it violated sec 2 of the Bill of Rights, and the properties were confiscated against her will and were done with unreasonable force and intimidation. Issue: Whether or Not the Philippine Overseas Employment Administration (or the Secretary of Labor) can validly issue warrants of search and seizure (or arrest) under Article 38 of the Labor Code Held: Under the new Constitution, . . . no search warrant or warrant of arrest shall issue except upon probable cause to be determined personally by the judge after examination under oath or affirmation of the complainant and the witnesses he may produce, and particularly describing the place to be searched and the persons or things to be seized. Mayors and prosecuting officers cannot issue warrants of seizure or arrest. The Closure and Seizure Order was based on Article 38 of the Labor

Code. The Supreme Court held, We reiterate that the Secretary of Labor, not being a judge, may no longer issue search or arrest warrants. Hence, the authorities must go through the judicial process. To that extent, we declare Article 38, paragraph (c), of the Labor Code, unconstitutional and of no force and effect The power of the President to order the arrest of aliens for deportation is, obviously, exceptional. It (the power to order arrests) cannot be made to extend to other cases, like the one at bar. Under the Constitution, it is the sole domain of the courts. Furthermore, the search and seizure order was in the nature of a general warrant. The court held that the warrant is null and void, because it must identify specifically the things to be seized. WHEREFORE, the petition is GRANTED. Article 38, paragraph (c) of the Labor Code is declared UNCONSTITUTIONAL and null and void. The respondents are ORDERED to return all materials seized as a result of the implementation of Search and Seizure Order No. 1205. Stonehill v Diokno Facts: Respondents issued, on different dates, 42 search warrants against petitioners personally, and/or corporations for which they are officers directing peace officers to search the persons of petitioners and premises of their offices, warehouses and/or residences to search for personal properties books of accounts, financial records, vouchers, correspondence, receipts, ledgers, journals, portfolios, credit journals, typewriters, and other documents showing all business transactions including disbursement receipts, balance sheets and profit and loss statements and Bobbins(cigarettes) as the subject of the offense for violations of Central Bank Act, Tariff and Customs Laws, Internal Revenue Code, and Revised Penal Code. Upon effecting the search in the offices of the aforementioned corporations and on the respective residences of the petitioners, there seized documents, papers, money and other records. Petitioners then were subjected to deportation proceedings and were constrained to question the legality of the searches and seizures as well as the admissibility of those seized as evidence against them. On March 20, 1962, the SC issued a writ of

preliminary injunction and partially lifted the same on June 29, 1962 with respect to some documents and papers. Held: Search warrants issued were violative of the Constitution and the Rules, thus, illegal or being general warrants. There is no probable cause and warrant did not particularly specify the things to be seized. The purpose of the requirement is to avoid placing the sanctity of the domicile and the privacy of communication and correspondence at the mercy of the whims, caprice or passion of peace officers. Document seized from an illegal search warrant is not admissible in court as a fruit of a poisonous tee. However, they could not be returned, except if warranted by the circumstances. Petitioners were not the proper party to question the validity and return of those taken from the corporations for which they acted as officers as they are treated as personality different from that of the corporation. Papa v Mago Facts: Petitioner Martin Alagao, head of the counter-intelligence unit of the Manila Police Department, acting upon a reliable information received on November 3, 1966 to the effect that a certain shipment of personal effects, allegedly misdeclared and undervalued, would be released the following day from the customs zone of the port of Manila and loaded on two trucks, and upon orders of petitioner Ricardo Papa, Chief of Police of Manila and a duly deputized agent of the Bureau of Customs, conducted surveillance at gate No. 1 of the customs zone. When the trucks left gate No. 1 at about 4:30 in the afternoon of November 4, 1966, elements of the counter-intelligence unit went after the trucks and intercepted them at the Agrifina Circle, Ermita, Manila. The load of the two trucks, consisting of nine bales of goods, and the two trucks, were seized on instructions of the Chief of Police. Upon investigation, a person claimed

ownership of the goods and showed to the policemen a "Statement and Receipts of Duties Collected on Informal Entry No. 1475501", issued by the Bureau of Customs in the name of a certain Bienvenido Naguit. The respondent Mago, filed a petition for mandamus and certiorari before the CFI Manila contending that the search and seizure is illegal for lack of a valid warrant. Moreover, she also contends that such articles sought from her is not included by the law for prohibited importation and that it no longer under the control of the Tariff and Customs code for it (articles) were already sold to the petitioner. She also contends that the search seizure conducted by the respondents are illegally being made outside the jurisdiction of the BOC and that the subsequent search warrant issued by the collector of customs is not valid being not issued by a judge. The respondent Mago filed an exparte motion to release the confiscated articles upon her posting a bond. This motion was then granted by the respondent Judge Jarencio. Issue: Was the seizure of the goods unlawful? And that the BOC has no jurisdiction over the articles sought? Held: No. it is a valid seizure. The Chief of the Manila Police Department, Ricardo G. Papa, having been deputized in writing by the Commissioner of Customs, could, for the purposes of the enforcement of the customs and tariff laws, effect searches, seizures, and arrests, and it was his duty to make seizure, among others, of any cargo, articles or other movable property when the same may be subject to forfeiture or liable for any fine imposed under customs and tariff laws. He could lawfully open and examine any box, trunk, envelope or other container wherever found when he had reasonable cause to suspect the presence therein of dutiable articles introduced into the Philippines contrary to law; and likewise to stop, search and examine any vehicle, beast or person reasonably suspected of holding or conveying such article as aforesaid. It cannot be doubted, therefore, that

petitioner Ricardo G. Papa, Chief of Police of Manila, could lawfully effect the search and seizure of the goods in question. The Tariff and Customs Code authorizes him to demand assistance of any police officer to effect said search and seizure, and the latter has the legal duty to render said assistance. This was what happened precisely in the case of Lt. Martin Alagao who, with his unit, made the search and seizure of the two trucks loaded with the nine bales of goods in question at the Agrifina Circle. He was given authority by the Chief of Police to make the interception of the cargo. Petitioner Martin Alagao and his companion policemen had authority to effect the seizure without any search warrant issued by a competent court. The Tariff and Customs Code does not require said warrant in the instant case. The Code authorizes persons having police authority under Section 2203 of the Tariff and Customs Code to enter, pass through or search any land, inclosure, warehouse, store or building, not being a dwelling house; and also to inspect, search and examine any vessel or aircraft and any trunk, package, box or envelope or any person on board, or stop and search and examine any vehicle, beast or person suspected of holding or conveying any dutiable or prohibited article introduced into the Philippines contrary to law, without mentioning the need of a search warrant in said cases. It is our considered view, therefore, that except in the case of the search of a dwelling house, persons exercising police authority under the customs law may effect search and seizure without a search warrant in the enforcement of customs laws. Note: The Bureau of Customs has the duties, powers and jurisdiction, among others, (1) to assess and collect all lawful revenues from imported articles, and all other dues, fees, charges, fines and penalties, accruing under the tariff and customs laws; (2) to prevent and suppress smuggling and other frauds upon the customs; and (3) to enforce tariff and customs laws. The goods in question were imported from Hongkong, as shown in the "Statement and

Receipts of Duties Collected on Informal Entry." As long as the importation has not been terminated the imported goods remain under the jurisdiction of the Bureau of Customs. Importation is deemed terminated only upon the payment of the duties, taxes and other charges upon the articles, or secured to be paid, at the port of entry and the legal permit for withdrawal shall have been granted. The payment of the duties, taxes, fees and other charges must be in full.

Nolasco vs. Pano Conflicting views between the ponente Justice Melencio-Herrera and Justices Teehankee and Cuevas concerning the validity place of arrest and search of the accused in connection with Lawful Search Incidental to an Arrest Facts: The petitioners, Nolasco, AguilarRoque and Tolentino were charged of the crime of Rebellion, Subversion and/or Conspiracy o commit Rebellion/Subversion and was arrested by Constabulary Security Group (CSG) on August 6th at 11:30 AM at the intersection of Mayon St. and P. Margali St, Quezon City. At 12 noon of that same day, a search was conducted at the residence of the petitioner Aguilar-Roque at 239 B Mayon St. QC. The CSG confiscated 428 written documents, typewriter and 2 wooden boxes. The petitioners alleged that the search warrant and arrest warrants issued was void in the ground that there is no established existing probable cause and that the warrant is a general in nature, violative of their constitutional right. Moreover, they contend that the articles confiscated in inadmissible as evidence as the search is illegal not being a search incidental to an arrest. Issue: Was the search cannot be qualified as a lawful search incidental to lawful arrest?

Held: No. The Search is lawful. UNDER Sec 12, rule 126 of the RRC, Section 12. Search without warrant of person arrested. - A person charged with an offense may be searched for dangerous weapons or anything which may be used as proof of the commission of the offense." The provision is declaratory in the sense that it is confined to the search, without a search warrant, of a person who had been arrested. It is also a general rule that, as an incident of an arrest, the place or premises where the arrest was made can also be search without a search warrant. In this latter case, "the extent and reasonableness of the search must be decided on its own facts and circumstances, and it has been stated that, in the application of general rules, there is some confusion in the decisions as to what constitutes the extent of the place or premises which may be searched". "What must be considered is the balancing of the individual's right to privacy and the public's interest in the prevention of crime and the apprehension of criminals." Considering that AGUILAR-ROQUE has been charged with Rebellion, which is a crime against public order; that the warrant for her arrest has not been served for a considerable period of time; that she was arrested within the general vicinity of her dwelling; and that the search of her dwelling was made within a half hour of her arrest, we are of the opinion that, in her respect, the search at No. 239-B Mayon Street, Quezon City, did not need a search warrant; this, for possible effective results in the interest of public order. The search in the residence of the petitioner Aguilar-Roque is valid since in relation with the nature of the crime of rebellion, which is a crime against public order, it is a search incidental to an arrest. The immediate vicinity of the arrest is not controlling as far as the publics interest in the prevention of the crime and apprehension of the criminals are concerned.

Posadas v CA Facts: While Pat. Ungab and Umpar were conducting a surveillance along Magallanes Street, Davao City, they spotted petitioner carrying a "buri" bag and they noticed him to be acting suspiciously.They approached the petitioner and identified themselves as members of the INP. Petitioner attempted to flee but his attempt to get away was thwarted by the two notwithstanding his resistance. They then checked the "buri" bag of the petitioner where they found one (1) caliber .38 revolver, two (2) rounds of live ammunition for a .38 caliber gun 2 a smoke (tear gas) grenade, 3 and two (2) live ammunitions for a .22 caliber gun. 4 the petitioner was asked to show the necessary license or authority to possess the firearms and ammunitions but failed to do so. Issue: Whether or not the warantless arrest and search was valid. Ruling: An arrest without a warrant may be effected by a peace officer or private person, among others, when in his presence the person to be arrested has committed, is actually committing, or is attempting to commit an offense; or when an offense has in fact just been committed, and he has personal knowledge of the facts indicating that the person arrested has committed it. Contrary to the argument of the Solicitor General that when the two policemen approached the petitioner, he was actually committing or had just committed the offense of illegal possession of firearms and ammunitions in the presence of the police officers and consequently the search and seizure of the contraband was incidental to the lawful arrest in accordance with Section 12, Rule 126 of the 1985 Rules on Criminal Procedure; At the time the peace officers in
this case identified themselves and apprehended the petitioner as he attempted to flee they did not know that he had committed, or was actually committing the offense of illegal possession of firearms and ammunitions. They just suspected that he was hiding something in the buri bag. They did not know what its contents were. The said circumstances did not justify an arrest without a warrant.

PP v de Lara F: After a surveillance conducted, a buybust operation was conducted by the police, as a consequence of which, accused was arrested. The accused already pocketed the marked money and handed two foils to the police when he sensed the presence of police operatives. He tried to retrieve the two foils but he was prevented from doing so. He tried to escape by running inside his house. The police pursued him and were able to subdue him. The accused admitted that he kept prohibited drugs in his house. He even showed the arresting officers a blue plastic bag containing prohibited drugs. The team, together with the accused, proceeded to WPD headquarters for investigation. During the investigation, accused was apprised of his constitutional rights to remain silent and to have the assistance of counsel. When appellant was asked to give a written statement, he refused to do so pending arrival of his lawyer. Accused contends that his arrest and the seizure of the bag containing prohibited drugs was null and void. He also contends that he was not assisted by counsel during custodial investigation, where he was forced to sign the photocopy of the marked money, the Receipt of Property Seized, and the Booking and Information Sheet. ISSUE: Whether or not the arrest of the accused and the seizure of the plastic bag were valid. RULING: YES. The accused was caught in flagrante as a result of a buy-bust operation. There was no need for a warrant. The policemen were not only authorized but were also under obligation to apprehend the drug pusher even without a warrant. The policemens entry into the house of the accused without a search warrant was in hot-pursuit of a person caught committing an offense in flagrante. The arrest that followed the hot-pursuit was valid. The seizure of the plastic bag was the result of the accuseds arrest inside the house. A contemporaneous search may be conducted
upon the person of the arrestee and the immediate vicinity where the arrest was made. ISSUE: Whether the documents signed by the

accused during the investigation were admissible in evidence. RULING: NO. There was no showing that accused was then assisted by counsel nor his waiver thereto put
into writing. (The rejection of these evidence would not affect the conviction of the accused in view of the abundance of other evidence establishing his guilt.)

PP v de Garcia Facts: Orphans, Jackielyn Ong (8yrs old) and her younger brother, were under the guardianship of their aunt Elizabeth Ong. Elizabeth worked abroad so the children stayed with Elizabeths live-in partner, David Garcia. Apparently, David raped Jackielyn since she was 8 (1990) til she was 10 years old (1994). Angelito Ong soon found out and they pressed charges. He was charged with simple crime of rape and the court below found him guilty only of simple rape as charged. (The SC determined that Garcia can only be charged with 10 acts of rape, the first of which was the first rape committed in 1990, and 9 of which were the incidents admitted by Garcia in his letter to Elizabeth, all committed in 94. Read the letter! Mwiwindang kayo!) Issue: What is the proper imposable penalty on appellant in light of his proven 10 acts of rape? Held: Reclusion perpetua for each of 10 felonies of rape ? Having been charged with the simple crime of rape, each of which warrants the imposition of the penalty of reclusion perpetua, both the trial court and the Peoples Tribune agree on that penalty to be imposed for each crime, although both contend that such penalty should be imposed on 183 acts of rape. We have already explained that appellant can be convicted of only ten crimes of rape, but we have not answered the unspoken question, since both the trial court and the Solicitor General have passed sub silentio thereover, on whether the ten convictions we sustain should be for simple rape or for its qualified form under the circumstances stated in Republic Act No. 7659 which amended Article 335 of the Revised Penal Code. Note that the 1st crime of rape in 90 of which Garcia was found guilty is covered by the original provisions of the RPC, while the other 9 committed in 94 are governed by the amendatory provisions of R.A. 7659, with circumstances necessitating higher penalties, and which took effect on Dec. 31, 93. ? Sec. 11 of said act provides that where the victim of the crime of rape is under 18 and offender is a guardian of the victim, death penalty shall be imposed. ? The additional attendant circumstances introduced by R.A 7659 should be considered as special qualifying circumstances specifically applicable to the crime of rape and, accordingly, cannot be offset by mitigating circumstances. The

obvious ratiocination is that, just like the effect of the attendant circumstances therefore added by R.A. 4111, although the crime is still denominated as rape such circumstances have changed the nature of simple rape by producing a qualified form thereof punishable by the higher penalty of death. ? In People vs. De la Cruz, it was held that the guardian referred to in the law is either a legal or judicial guardian as understood in the rules on civil procedure. Since, the inclusion of the guardian in the enumeration of the offenders in Art. 335 is to authorize the imposition of the death penalty on him, this definition of guardian should be used in the Art. As opposed to Art. 344s definition. ? The Court held that Garcia had a relation of guardian to victim, whether as a natural or legal, or even de facto and, much less, judicial guardian. He was merely expected to carry out Elizabeths directions, and Elizabeth continued to be the guardian de facto of the children. In fine, at the very most, he was only an unwilling custodian and caretaker, not unlike a domestic majordomo or steward of the house and the children, and for which services he obtained free board and lodging. ? Ironically, that amorphous role that he played in the lives of the children, and which enabled him to abuse them, offers him salvation from the death penalty which he deserves. The death penalty contemplated for a real guardian under the amendments introduced by R.A. No. 7659 cannot be imposed since he does not fit into that category. ? Furthermore, Art. 335 originally provided only for simple rape punishable by reclusion perpetua, but R.A 4111 introduced amendments thereto by providing for qualified forms of rape carrying the death penalty, that is, when committed with the use of deadly weapon or by two or more persons, when by reason or on the occasion of the rape the victim becomes insane, or, under the same circumstances, a homicide is committed. ? The homicide in the last two instances in effect created a special complex crime of rape with homicide. The first two attendant circumstances are considered as equivalent to qualifying circumstances since they increase the penalties by degrees, and not merely as aggravating circumstances which effect only the period of the penalty but do not increase it to a higher degree. The original provisions of Article 335 and the

amendments of R.A. 4111 are still maintained. ? R.A. 7659 thereafter introduced 7 more attendant circumstances which thus qualifies crime by increasing the penalty 1 degree higher through the imposition of the death penalty. All these new attendant circumstances, just like those introduced by R.A. 4111, partake of the nature of qualifying circumstances, and not merely aggravating circumstances, on the same rationale already explained. ? Qualifying circumstances must be properly pleaded in the indictment, otherwise, if proven, they shall be considered only as aggravating circumstance. Information filed against appellant charged only the felony of simple rape and no attendant qualifying circumstance was alleged. Thus, he cannot be punished with the penalty of death even assuming arguendo that he is such a guardian. Neither can that fact be considered to aggravate his liability as the penalty for simple rape is the single indivisible penalty of reclusion perpetua. ? Thus for the ten crimes of rape of which he was guilty, only the penalty of reclusion perpetua can be imposed. WARRANT OF ARREST, SEARCH WARRANTS SECTION 2 - The right of the people to be secure in their persons, houses, papers and effects against unreasonable searches and seizures of whatever nature and for any purpose shall be inviolable, and no search warrant or warrant of arrest shall issue except upon probable cause to be determined personally by a judge under oath or affirmation of the complainant and the witnesses he may produce, and particularly describing the place to be searched and the persons or things to be seized. CASE DOCTRINES by: C D R A. When is a search a reasonable search? Valmonte v. De Villa checkpoints Section 2 is a personal right invocable only by those whose rights have been infringed or threatened to be infringed; reasonableness is determined by a fixed formula but from the circumstances of the case; not all searches and seizures are not allowed; between the inherent right of the state to protect its existence and promote public welfare and an individual; right against warrantless search which was

reasonably conductedm the former should prevail B. Requisites of a Valid Warrant Alvarez v. CFI of Tayabas definition of a search warrant an order in writing, issued in the name of the People of the Philippine Islands, signed by a judge or justice of peace and directed to a peace officer commanding him to search for personal property and bring it before court, OATH any form of attestation that a party signifies that he is bound by conscience to perform an act faithfully or truthfully People v. CA the general rule is that search warrants must be served during the daytime (protect the public from the abrasiveness of official intrusions). Exception: a search at any reasonable hour of day or night may be made when the application asserts that the property in on the person or place ordered to be searched. Absence of abuse of discretion, a search conducted at night where so allowed is not improper People v. Veloso parliamentary club JOHN DOE WARRANTS Valid IF the best description possible is given in the arrest warrant it must be sufficient to indicate clearly on whom it is to be served by stating his occupation, personal appearance or peculiarities, place of residence or other circumstances which he may be identified Microsoft v. Maxicorp software probable cause such reasons, supported by facts and circumstances as will warrant a cautious man in the belief that his action and the means taken in prosecuting it are legally just and proper; OATH must refer to the truth of the facts WITHIN THE PERSONAL KNOLEDGE OF THE PETITIONER OR HIS WITNESSES; probable cause deals with probability and not absolute certainty Burgos Sr. v. Chief of Staff, AFP Metropolitan Mail and We Forum typographical error in specifying the address to be searched not sufficient to invalidate search warrant where the address intended appears on the face of the warrant; probable cause such facts or circumstances which would lead a reasonably prudent man to believe that an offense has been committed and the objects sought in connection with the offense are in the place sought to be searched

People v. CA Abigails Variety Store VOID warrant the claim that the place actually searched although not the one specified in the warrant is exactly what they had in view when they applied for the warrant is unacceptable. What is material in determining the validity of the warrant is the place stated in the warrant, not the one they had in their thoughts; particularization of description may properly be done only by the judge and only in the warrant itself Corro v. Lising Philippine Times conclusions of law of military officers will not satisfy probable cause requirement for issuance of search warrants Soliven v. Makasiar The judge is not required to personally examine the complainant and his witnesses. He shall: 1) personally evaluate the report and the supporting documents submitted by the fiscal regarding the existence of probable cause and on the basis thereof, issue a warrant of arrest OR 2) if on the basis thereof, he does not find probable cause, he may disregard the fiscals report and require the submission of supporting affidavits of witnesses to aid him in arriving at a conclusion of the existence of probable cause Lim Sr. v. Felix certification by the fiscal of the existence of probable cause does not bind the judge. Preliminary inquiry determines probable cause for the issuance of a search warrant (prosecutor); preliminary examination (judge) investigation for the determination of a probable cause for the issuance of a warrant of arrest; preliminary investigation proper ascertains whether the offender should be held for trial or be released. Yao Sr. v. People GASUL and SHELLANE v. MASAGANA - examination of complainant and witnesses must be probing and exhaustive not merely routinary, general, peripheral, perfunctory or pro forma; law does not require that the thihgs to be seized should be described in very precise and minute details. C. Warrantless Searches and Seizures Nolasco v. Pano articles seized by void warrants should be retuned to its owners Papa v. Mago Customs Search search warrant not necessary except if the place to

be searched is a dwelling or house Tariff and Customs Code People v. CFI of Rizal search of moving vehicle Carroll doctrine search of moving vehicles or automobiles no search warrant needed People. V. Lo Ho Wing - exception to the issuance of search warrant: 1) search incidental to a lawful arrest; 2) search of moving vehicle; 3) seizure of evidence in plain view People v. Evaristo evidence in plain view Harris v. Coolidge, Coolidge v. New Hampshire no search warrant needed; Malacat v. CA valid waiver must be made in writing and in the presence of counsel; search incidental to a lawful arrest v. stop and frisk Terry Case probable cause is not required to conduct stop and frisk but mere suspicion or a hunch will not validate it. A genuine reason must exist. People v. De Gracia Eurocar Sales Office crime was in fact being committed search incidental to lawful arrest valid People v. Johnson inspection at airports Persons may lose the protection of the search and seizure clause by exposure of their persons or property to the public in a manner reflecting lack of subjective expectation of privacy David v. Macapagal-Arroyo PP 1017 case doctrines same as the ones above (related topics: freedom of expression; freedom to peaceably assemble) People v. Nuevas illegal possession of marijuana - in cases of searches incidental to a lawful arrest, the arrest must precede the search; warrantless search, when valid: 1) incidental to lawful arrest; 2) evidence in plain view [a) valid prior intrusion, police are legally present in the pursuit of their official duties, b) evidence was inadvertently discovered, c) evidence immediately apparent, d) plain view justified mere seizure with =out further search]; 3) search of moving vehicle; 4) consented warrantless search; 5) customs search; 6) stop and frisk; 7) exigent and emergency circumstances D. Searches and seizures of whatever nature for any purpose E. Warrantless Arrests

*****LIBERTY OF ABODE AND TRAVEL Umil v. Ramos subversion a continuing offense - arrest without a warrant is justified if the person arrested in caught in flagrante delicto People v. Aminudin - M/V Wilcon; marijuana not caught in flagrante delicto; search was unreasonable; evidence inadmissible Harvey v. Defensor-Santiago pedophiles the rights granted in Section 2 are available to all persons including aliens, whether accused of a crime or not People v. Mengote suspicious man outside a person may not be stopped and frisked in broad daylight on a bust street on a mere unexplained suspicion Posadas v. Ombudsan Sigma Rho v. Scintilla Juris Arrest made without a valid warrant: Rule 113, Section 5 of the Rules of Court when in the presence of a police officer or a private individual: 1) the person arrested has committed, is actually committing, or attempting to commit an offense; 2) when an offense has actually been committed, and he has personal knowledge of the facts indicating that the person to be arrested commited it; 3) when the person arrested is a prisoner who has escaped from a penal establishment or place where his is serving final or temporary judgment (pending), escaped while being transferred Villavicencio v Lukban Issue: The writ of Habeas Corpus was filed by the petitioner, with the prayer that the respondent produce around 170 women whom Justo Lukban et, al deported to Davao. Liberty of abode was also raised versus the power of the executive of the Municipality in deporting the women without their knowledge in his capacity as Mayor. Facts: Justo Lukban as Manila City's Mayor together with Anton Hohmann, the city's Chief of Police, took custody of about 170 women at the night of October 25 beyond the latters consent and knowledge and thereafter were shipped to Mindanao specifically in Davao where they were signed as laborers. Said women are inmates of the houses of prostitution situated in Gardenia Street, in the district of Sampaloc. That when the petitioner filed for habeas corpus, the respondent moved to dismiss the case saying that those women were already out of their jurisdiction and that , it should be filed in the city of Davao instead. The court ruled in favor of the petitioner with the instructions; For the respondents to have fulfilled the court's order, three optional courses were open: (1) They could have produced the bodies of the persons according to the command of the writ; or (2) they could have shown by affidavit that on account of sickness or infirmity those persons could not safely be brought before the court; or (3) they could have presented affidavits to show that the parties in question or their attorney waived the right to be present. Held: The court concluded the case by granting the parties aggrieved the sum of 400 pesos each, plus 100 pesos for nominal damage due to contempt of court. Reasoning further that if the chief executive of any municipality in the Philippines could forcibly and illegally take a private citizen and place him beyond the boundaries of the municipality, and then, when called upon to defend his official action, could calmly fold his hands and claim that the person was

under no restraint and that he, the official, had no jurisdiction over this other municipality. We believe the true principle should be that, if the respondent is within the jurisdiction of the court and has it in his power to obey the order of the court and thus to undo the wrong that he has inflicted, he should be compelled to do so. Even if the party to whom the writ is addressed has illegally parted with the custody of a person before the application for the writ is no reason why the writ should not issue. If the mayor and the chief of police, acting under no authority of law, could deport these women from the city of Manila to Davao, the same officials must necessarily have the same means to return them from Davao to Manila. The respondents, within the reach of process, may not be permitted to restrain a fellow citizen of her liberty by forcing her to change her domicile and to avow the act with impunity in the courts, while the person who has lost her birthright of liberty has no effective recourse. The great writ of liberty may not thus be easily evaded. Caunca v Salazar Facts: This is an action for habeas corpus brought by Bartolome Caunca in behalf of his cousin Estelita Flores who was employed by the Far Eastern Employment Bureau, owned by Julia Salazar, respondent herein. An advanced payment has already been given to Estelita by the employment agency, for her to work as a maid. However, Estelita wanted to transfer to another residence, which was disallowed by the employment agency. Further she was detained and her liberty was restrained. The employment agency wanted that the advance payment, which was applied to her transportation expense from the province should be paid by Estelita before she could be allowed to leave. Issue: Whether or Not an employment agency has the right to restrain and detain a maid without returning the advance payment it gave? Held: An employment agency, regardless of the amount it may advance to a prospective employee or maid, has absolutely no power to curtail her freedom of movement. The fact that no physical force has been exerted to keep her in the house of the respondent

does not make less real the deprivation of her personal freedom of movement, freedom to transfer from one place to another, freedom to choose ones residence. Freedom may be lost due to external moral compulsion, to founded or groundless fear, to erroneous belief in the existence of an imaginary power of an impostor to cause harm if not blindly obeyed, to any other psychological element that may curtail the mental faculty of choice or the unhampered exercise of the will. If the actual effect of such psychological spell is to place a person at the mercy of another, the victim is entitled to the protection of courts of justice as much as the individual who is illegally deprived of liberty by duress or physical coercion. Rubi v Provincial Board of Mindoro Facts: The provincial board of Mindoro adopted resolution No. 25 wherein nonChristian inhabitants (uncivilized tribes) will be directed to take up their habitation on sites on unoccupied public lands. It is resolved that under section 2077 of the Administrative Code, 800 hectares of public land in the sitio of Tigbao on Naujan Lake be selected as a site for the permanent settlement of Mangyanes in Mindoro. Further, Mangyans may only solicit homesteads on this reservation providing that said homestead applications are previously recommended by the provincial governor.
In that case, pursuant to Section 2145 of the Revised Administrative Code, all the Mangyans in the townships of Naujan and Pola and the Mangyans east of the Baco River including those in the districts of Dulangan and Rubi's place in Calapan, were ordered to take up their habitation on the site of Tigbao, Naujan Lake. Also, that any Mangyan who shall refuse to comply with this order shall upon conviction be imprisoned not exceed in sixty days, in accordance with section 2759 of the revised Administrative Code. Said resolution of the provincial board of Mindoro were claimed as necessary measures for the protection of the Mangyanes of Mindoro as well as the protection of public forests in which they roam, and to introduce civilized

It is alleged that the Manguianes are being illegally deprived of their liberty by the provincial officials of that province. Rubi and his companions are said to be held on the reservation established at Tigbao, Mindoro, against their will, and one Dabalos is said to be held under the custody of the provincial sheriff in the prison at Calapan for having run away form the reservation. Issue: Whether or Not Section 2145 of the Administrative Code deprive a person of his liberty pf abode. Thus, WON Section 2145 of the Administrative Code of 1917 is constitutional. Held: The Court held that section 2145 of the Administrative Code does not deprive a person of his liberty of abode and does not deny to him the equal protection of the laws, and that confinement in reservations in accordance with said section does not constitute slavery and involuntary servitude. The Court is further of the opinion that section 2145 of the Administrative Code is a legitimate exertion of the police power. Section 2145 of the Administrative Code of 1917 is constitutional.
Assigned as reasons for the action: (1) attempts for the advancement of the nonChristian people of the province; and (2) the only successfully method for educating the Manguianes was to oblige them to live in a permanent settlement. The Solicitor-General adds the following; (3) The protection of the Manguianes; (4) the protection of the public forests in which they roam; (5) the necessity of introducing civilized customs among the

Marcos v Manglapus Facts: This case involves a petition of mandamus and prohibition asking the court to order the respondents Secretary of Foreign Affairs, etc. To issue a travel documents to former Pres. Marcos and the immediate members of his family and to enjoin the implementation of the President's decision to bar their return to the Philippines. Petitioners assert that the right of the Marcoses to return in the Philippines is guaranteed by the Bill of Rights, specifically Sections 1 and 6. They contended that Pres. Aquino is without power to impair the liberty of abode of the Marcoses because only a court may do so within the limits prescribed by law. Nor the President impair their right to travel because no law has authorized her to do so. They further assert that under international law, their right to return to the Philippines is guaranteed particularly by the Universal Declaration of Human Rights and the International Covenant on Civil and Political Rights, which has been ratified by the Philippines. Issue: Whether or not, in the exercise of the powers granted by the constitution, the President (Aquino) may prohibit the Marcoses from returning to the Philippines. Held: "It must be emphasized that the individual right involved is not the right to travel from the Philippines to other countries or within thePhilippines. These are what the right to travel would normally connote. Essentially, the right involved in this case at bar is the right to return to one's country, a distinct right under international law, independent from although related to the right to travel. Thus, the Universal Declaration of Human Rights and the International Covenant on Civil and Political Rights treat the right to freedom of movement and abode within the territory of a state, the right to leave the country, and the right to enter one's country as separate and distinct rights. What the Declaration speaks of is the "right to freedom of movement and residence within the borders of each state". On the other hand, the Covenant guarantees the right to liberty of movement and freedom to choose his residence and the right to be free to leave any country, including his own. Such rights may only be restricted by laws protecting the nationalsecurity, public order, public

health or morals or the separate rights of others. However, right to enter one's country cannot be arbitrarily deprived. It would be therefore inappropriate to construe the limitations to the right to return to ones country in the same context as those pertaining to the liberty of abode and the right to travel. The Bill of rights treats only the liberty of abode and the right to travel, but it is a well considered view that the right to return may be considered, as a generally accepted principle of International Law and under our Constitution as part of the law of the land. The court held that President did not act arbitrarily or with grave abuse of discretion in determining that the return of the Former Pres. Marcos and his family poses a serious threat to national interest and welfare. President Aquino has determined that the destabilization caused by thereturn of the Marcoses would wipe away the gains achieved during the past few years after the Marcos regime. The return of the Marcoses poses a serious threat and therefore prohibiting their return to the Philippines, the instant petition is hereby DISMISSED. Silverio v CA Facts: Petitioner was charged with violation of Section 2 (4) of the revised securities act. Respondent filed to cancel the passport of the petitioner and to issue a hold departure order. The RTC ordered the DFA to cancel petitioners passport, based on the finding that the petitioner has not been arraigned and there was evidence to show that the accused has left the country with out the knowledge and the permission of the court. Issue: Whether or Not the right to travel may be impaired by order of the court. Held: The bail bond posted by petitioner has been cancelled and warrant of arrest has been issued by reason that he failed to appear at his arraignments. There is a valid restriction on the right to travel, it is imposed that the accused must make himself available whenever the court requires his presence. A person facing criminal charges may be restrained by the Court from leaving the country or, if abroad, compelled to return (Constitutional Law,

Manguianes. One cannot hold that the liberty of the citizen is


unduly interfered without when the degree of civilization of the Manguianes is considered. They are restrained for their own good and the general good of the Philippines. Liberty regulated by law": Implied in the term is restraint by law for the good of the individual and for the greater good of the peace and order of society and the general well-being. No man can do exactly as he pleases. None of the rights of the citizen can be taken away except by due process of law. Therefore, petitioners are not unlawfully imprisoned or restrained of their liberty. Habeas corpus can, therefore, not issue.

customs among them. It appeared that Rubi and those living in his rancheria have not fixed their dwelling within the reservation of Tigbao and are liable to be punished.

Cruz, Isagani A., 1987 Edition, p. 138). So it is also that "An accused released on bail may be re-arrested without the necessity of a warrant if he attempts to depart from the Philippines without prior permission of the Court where the case is pending (ibid., Sec. 20 [2nd par. ]). Article III, Section 6 of the 1987 Constitution should be interpreted to mean that while the liberty of travel may be impaired even without Court Order, the appropriate executive officers or administrative authorities are not armed with arbitrary discretion to impose limitations. They can impose limits only on the basis of "national security, public safety, or public health" and "as may be provided by law," a limitive phrase which did not appear in the 1973 text (The Constitution, Bernas, Joaquin G.,S.J., Vol. I, First Edition, 1987, p. 263). Apparently, the phraseology in the 1987 Constitution was a reaction to the ban on international travel imposed under the previous regime when there was a Travel Processing Center, which issued certificates of eligibility to travel upon application of an interested party (See Salonga vs. Hermoso & Travel Processing Center, No. 53622, 25 April 1980, 97 SCRA 121). Holding an accused in a criminal case within the reach of the Courts by preventing his departure from the Philippines must be considered as a valid restriction on his right to travel so that he may be dealt with in accordance with law. The offended party in any criminal proceeding is the People of the Philippines. It is to their best interest that criminal prosecutions should run their course and proceed to finality without undue delay, with an accused holding himself amenable at all times to Court Orders and processes

Salonga v Hermoso This is not the first time petitioner Jovito R. Salonga came to this Tribunal by way of a mandamus proceeding to compel the issuance to him of a certificate of eligibility to travel. In the first case, Salonga v. Madella, 1 the case became moot and academic as the Office of the Solicitor General, in its answer to the petition, stated that the travel eligibility certificate was not denied and, as a matter of fact, had been granted. Nonetheless, a brief separate opinion was filed, concurring in the resolution, and worded thus: "Clearly this petition had assumed a moot and academic character. Its dismissal is thus indicated. May I just add these few words as my response to the plea of petitioner in his Manifestation and Reply dated October 28, 1978. This is how I would view the matter not only where petitioner is concerned but in all other similar cases. Respondent Travel Processing Center should discharge its injunction conformably to the mandate of the Universal Declaration of Human Rights on the right to travel. One of the highlights of the keynote address of President Marcos in the Manila World Law Conference in celebration of the World Peace Through Law Day on August 21, 1977 was the lifting of 'the ban on international travel.' There should be fidelity to such a pronouncement. It is the experience of the undersigned in his lectures abroad the last few years, in the United States as well as in Malaysia, Singapore and Australia, that respect accorded constitutional rights under the present emergency regime had elicited the commendation of members of the bench, the bar, and the academe in foreign lands. It is likewise worthy of notice that in his keynote address to the International Law Association, President Marcos made reference to martial law being instituted in accordance with law and that the Constitution had been applied in appropriate cases. As an agency of the executive branch, therefore, the Travel Processing Center should ever be on its guard, lest the impression be created that such declarations amount, to paraphrase Justice Jackson, to no more than munificent bequests in a pauper's will. Petitioner, to my mind, is justified, the more so in the light of the Answer of Acting Solicitor General Vicente Mendoza, to an affirmative response to his prayer in his Manifestation and Reply 'that under the circumstances mentioned in the Petition, Petitioner is entitled to travel abroad, and that it is in recognition of this right that Respondents have issued his

Certificate of Eligibility to Travel, as mentioned in the Answer. 2 The present petition is likewise impressed with a moot and academic aspect. In the motion to dismiss of the Solicitor General dated April 21, 1980, it was stated that the certificate of eligibility to travel had been granted petitioner. A xeroxed copy was enclosed. A resolution for dismissal is, therefore, in order. From the docket of this Court, it appears that other petitions of this character had been filed in the past, namely, Santos v. The Special Committee on Travel Abroad, 3 Pimentel v. Travel Processing Center, 4 and Gonzales v. Special Committee on Travel. 5 In the aforesaid cases, as in this and the earlier Salonga petition, there was no occasion to pass on the merits of the controversy as the certificates of eligibility to travel were granted. The necessity for any ruling was thus obviated. Nonetheless, in view of the likelihood that in the future this Court may be faced again with a situation like the present which takes up its time and energy needlessly, it is desirable that respondent Travel Processing Center should exercise the utmost care to avoid the impression that certain citizens desirous of exercising their constitutional right to travel could be subjected to inconvenience or annoyance. In the address of President and Prime Minister Ferdinand E. Marcos before the American
Newspaper Publishers Association last Tuesday April 22, 1980, emphasized anew the respect accorded constitutional rights The freedom to travel is certainly one of the most cherished. He cited with approval the ringing affirmation of Willoughby, who, as he noted was "partial to the claims of liberty." 6 Burdick 7 and Willis, 8 both of whom were equally convinced that there be no erosion to human rights even in times of martial law, likewise received from President Marcos the accolade of his approval. It would appear, therefore, that in case of doubt of the Officer-in-Charge of the Travel Processing Center, the view of General Fabian Ver should immediately be sought. It goes without saying that the petition for such certificate of eligibility to travel be filed at the earliest opportunity to facilitate the granting thereof and preclude any disclaimer as to the person desiring to travel being in any way responsible for any delay. WHEREFORE, the petition is dismissed for being moot and academic.

Manotoc Jr v CA Facts: Petitioner was charged with estafa. He posted bail. Petitioner filed before each of the trial courts a motion entitled, "motion for permission to leave the country," stating as ground therefor his desire to go to the United States, "relative to his business transactions and opportunities." The prosecution opposed said motion and after due hearing, both trial judges denied the same. Petitioner thus filed a petition for certiorari and mandamus before the then Court of Appeals seeking to annul the orders dated March 9 and 26, 1982, of Judges Camilon and Pronove, respectively, as well as the communication-request of the Securities and Exchange Commission, denying his leave to travel abroad. He likewise prayed for the issuance of the appropriate writ commanding the Immigration Commissioner and the Chief of the Aviation Security Command (AVSECOM) to clear him for departure. The Court of Appeals denied the petition. Petitioner contends that having been admitted to bail as a matter of right, neither the courts which granted him bail nor the Securities and Exchange Commission which has no jurisdiction over his liberty could prevent him from exercising his constitutional right to travel. Issue: Whether or Not his constitutional right to travel has been violated. Held: A court has the power to prohibit a person admitted to bail from leaving the Philippines. This is a necessary consequence of the nature and function of a bail bond. The condition imposed upon petitioner to make himself available at all times whenever the court requires his presence operates as a valid restriction on his right to travel. Indeed, if the accused were allowed to leave the Philippines without sufficient reason, he may be placed beyond the reach of the courts. Petitioner has not shown the necessity for his travel abroad. There is no indication that the business transactions cannot be undertaken by any other person in his behalf.

****FREEDOM OF EXPRESSION Near v Minnesota Brief Fact Summary. A Minnesota law that gagged a periodical from publishing derogatory statements about local public officials was held unconstitutional by the Supreme Court of the United States (Supreme Court). Synopsis of Rule of Law. The freedom of press is essential to the nature of a free state but that freedom may be restricted by the government in certain situations. Facts. The Saturday Press (the Press) published attacks on local officials. The Press claimed that the chief of police had illicit relations with gangsters. Minnesota officials obtained an injunction in order to abate the publishing of the Press newspaper under a state law that allowed this course of action. The state law authorized abatement, as a public nuisance, of a malicious, scandalous and defamatory newspaper, or other periodical. A state court order abated the Press and enjoined the Defendants, publishers of the Press (Defendants), from publishing or circulating such defamatory and scandalous periodicals. Issue. Whether a statute authorizing such proceedings is consistent with the conception of the liberty of the press as historically conceived and guaranteed? Held. No. Judgment of the state court reversed. The fact that the liberty of press may be abused by miscreant purveyors of scandal does not effect the requirement that the press has immunity from previous restraints when it deals with official misconduct. Subsequent punishment for such abuses as may exist is the appropriate remedy, consistent with the constitutional privilege. Therefore, a statute authorizing such proceedings is not consistent with the conception of the liberty of the press as historically conceived and guaranteed and is thus, unconstitutional. The statute in question cannot be justified by reason of the fact that the publisher is permitted to show, before injunction issues, that the matter published is true and is published with good motives and for justifiable ends. This statute, if upheld, could lead to a complete system of censorship. Thus, the statute is a substantial infringement on the liberty of the press and in violation of the Fourteenth Amendment of the Constitution. Dissent. This statute does not operate as a

previous restraint on publication within proper meaning of that phrase. Discussion. The Supreme Court of the United States (Supreme Court) in this case extended the presumption against prior restraint in the licensing context to judicial restraints as well. Kingsley v Brown Facts of the Case New York state law authorized the legal counsel for a municipality to seek an injunction against and the destruction of material deemed by the courts to be obscene. Peter Campbell Brown, Corporation Counsel for the City of New York, sought such an injunction against several bookstores. The process of review that followed was a civil, rather than criminal procedure, and the courts ultimately granted the injunction and sought to destroy the obscene material. Question Did the civil procedure as laid out by New York statute violate the booksellers' rights under the Due Process Clause of the Fourteenth Amendment by not allowing for a criminal trial before imposing an injunction and destroying the obscene material? Conclusion Decision: 5 votes for Brown, 4 vote(s) against Legal provision: Amendment 1: Speech, Press, and Assembly No. In a 54 opinion authored by Justice Felix Frankfurter, the Court concluded that the civil procedure complied with the requirements of due process. Justice Frankfurter noted that the procedural safeguards were not significantly different from many state criminal misdemeanor procedures, nor were the penalties any more severe. The opinion also reaffirmed the existing notion that the constitutional protection of speech does not extend to obscene material.

Grosjean v American Press Co 297 U.S. 233 (1936), argued 14 Jan. 1936, decided 10 Feb. 1936, by vote of 9 to 0; Sutherland for the Court. The Court unanimously invalidated a license tax on the business of selling advertising (in the amount of 2 percent of the gross receipts from such sales) imposed byLouisiana in 1934 on all newspapers with a circulation of more than twenty thousand copies per week. The tax was challenged as an abridgment of freedom of the press and as a violation of equal protection. It was noted by counsel that only thirteen of the 163 newspapers in the state had sufficient circulation to be required to pay it, and twelve of those thirteen were actively opposed to the Huey Long administration, at whose instigation the tax was enacted. The Court considered only the free press challenge. It equated the license tax to the taxes on knowledge imposed on newspapers and advertising by Parliament in the eighteenth century, whose purpose was not to raise revenue but to reduce the circulation of newspapers and thus limit public access to criticisms of the Crown. The obviously similar motivation of the Louisiana legislature was plainly crucial to the Court's conclusion here. The tax was held unconstitutional because it was hostile to the press. It applied only to a selected group of newspapers (p. 251) and was a deliberate and calculated device in the guise of a tax to limit the circulation of information pertaining to public affairs (p. 250). The year after Grosjean, the Court made clear that newspapers are not exempt from nondiscriminatory general taxation. In Giragi v. Moore (1937), it dismissed without opinion a claim for such an exemption. The Court has since categorically declared that newspapers are subject to all forms of nondiscriminatory economic regulation, including taxation (Minneapolis Star & Tribune Co. v. Minnesota Commissioner of Revenue, 1983).

New York Times v US Brief Fact Summary. The Supreme Court of the United States (Supreme Court) held that the Government failed to meet the requisite burden of proof needed to justify a prior restraint of expression when attempting to enjoin the New York Times and Washington Post from publishing contents of a classified study. Synopsis of Rule of Law. Any system of prior restraints on expression comes to the Supreme Court bearing a heavy presumption against its invalidity. The Government thus creates a heavy burden of showing justification for the enforcement of such a restraint. Facts. The United States sought to enjoin the New York Times and Washington Post from publishing contents of a confidential study about the Governments decision making with regards to Vietnam policy. The District Court in the New York Times case and the District Court and the Court of Appeals in the Washington Post case held that the Government had not met the requisite burden justifying such a prior restraint. Issue. Whether the United States met the heavy burden of showing justification for the enforcement of such a restraint on the New York Times and Washington Post to enjoin them from publishing contents of a classified study? Held. No. Judgments of the lower courts affirmed. The order of the Court of Appeals for the Second Circuit is reversed and remanded with directions to enter a judgment affirming the District Court. The stays entered June 25, 1971, by the Court are vacated. The mandates shall issue forthwith.

Gonzales v COMELEC Facts: The case is an original action for prohibition, with preliminary injunction. The main facts are not disputed. On March 16, 1967, the Senate and the House of Representatives passed the following resolutions: 1. R. B. H. (Resolution of Both Houses) No. 1, proposing that Section 5, Article VI, of the Constitution of the Philippines, be amended so as to increase the membership of the House of Representatives from a maximum of 120, as provided in the present Constitution, to a maximum of 180, to be apportioned among the several provinces as nearly as may be according to the number of their respective inhabitants, although each province shall have, at least, one (1) member; 2. R. B. H. No. 2, calling a convention to propose amendments to saidConstitution, the convention to be composed of two (2) elective delegates from each representative district, to be "elected in the general elections to be held on the second Tuesday of November, 1971;" and 3. R. B. H. No. 3, proposing that Section 16, Article VI, of the sameConstitution, be amended so as to authorize Senators and members of the House of Representatives to become delegates to the aforementioned constitutional convention, without forfeiting their respective seats in Congress. Subsequently, Congress passed a bill, which, upon approval by the President, on June 17, 1967, became Republic Act No. 4913, providing that the amendments to the Constitution proposed in the aforementioned Resolutions No. 1 and 3 be submitted, for approval by the people, at the general elections which shall be held on November 14, 1967. Issue: Whether or Not a Resolution of Congress, acting as a constituent assembly, violates the Constitution. Held: Inasmuch as there are less than eight (8) votes in favor of declaring Republic Act 4913 and R. B. H. Nos. 1 and 3 unconstitutional and invalid, the petitions in these two (2) cases must be, as they are hereby, dismiss and the writs therein prayed for denied, without special pronouncement

as to costs. It is so ordered. As a consequence, the title of a de facto officer cannot be assailed collaterally. It may not be contested except directly, by quo warranto proceedings. Neither may the validity of his acts be questioned upon the ground that he is merely a de facto officer. And the reasons are obvious: (1) it would be an indirect inquiry into the title to the office; and (2) the acts of a de facto officer, if within the competence of hisoffice, are valid, insofar as the public is concerned. "The judicial department is the only constitutional organ which can be called upon to determine the proper allocation of powers between the several departments and among the integral or constituent units thereof." Article XV of the Constitution provides: . . . The Congress in joint session assembled, by a vote of three-fourths of all the Members of the Senate and of the House of Representativesvoting separately, may propose amendments to this Constitution or call a contention for that purpose. Such amendments shall be valid as part of this Constitution when approved by a majority of the votes cast at an election at which the amendments are submitted to the people for their ratification. From our viewpoint, the provisions of Article XV of the Constitution are satisfied so long as the electorate knows that R. B. H. No. 3 permits Congressmen to retain their seats as legislators, even if they should run for and assume the functions of delegates to the Convention. Santiago v Far East Broadcasting This is an original petition for mandamus. The petitioner prays this court to issue the writ "directed to the respondent, Far Eastern Broadcasting, commanding it that the petitioner be allowed to broadcast any speech or address without previous censorship." The allegations of the petitioner are as follows: 1. That the petitioner is of legal age, resident of the City of Manila and campaign manager of the political party Popular Front Sumulong; the respondent is a

public utility corporation engaged in radio broadcasting service, and organized and existing in and under the laws of the Philippines, having its principal office in the insular Life Building, Manila. 2. That, sometime In September, 1942, the petitioner, as campaign manager of the Popular Front Sumulong, asked the respondent, through its manager, for the purchase of air time for the broadcast of the Popular Front political speeches at the Opera House on September 23, 1941. But the said respondent refused and still refuses to allow to broadcast except on condition that complete manuscript of contemplated speeches should be submitted in advance. 3. That the act of the respondent in refusing to allow the use of its station for broadcasting the speeches except upon prior submission of complete manuscript of the speeches to be broadcast is discriminatory and constitutes unlawful censorship and a violation of the freedom of speech guaranteed by our Constitution. 4. That the present the constitutional guarantees are not suspended, the state not being placed under martial law, and, consequently, the conduct of the respondent is unlawful. 5. That the respondent, in refusing to allow the petitioners to use its station except upon prior cencorship, unlawfully excludes and deprives the latter from enjoyment and exercise of the right and freedom of speech. 6. That the petitioner has no other plain, speedy or adequate remedy to enforce his right other than this action. To that petition the respondent answered as follows: 1. That the respondent is a corporation organized and existing according to the laws of the Commonwealth of the Philippines and in its aforesaid capacity owner and operator of stations,

KZRM, KZRF and KZUM by virtue of a specific franchise granted under Act No. 3180. 2. That with reference to the purchase of air time mentioned in paragraph 2 of the petition, the respondent required the petitioner to submit a manuscript of contemplated speeches in advance of the broadcast in the exercise of a right granted by existing laws and regulations and in compliance with its obligation expressly undertaken to safeguard public morality and to broadcast programs that are educational, entertaining and not prejudicial to public interest. 3. That the act of the respondent mentioned in paragraph 2 hereof is not discriminatory for it is applied to all persons alike, and not violative of the constitutional provision regarding freedom of speech. 4. That the act of the respondent mentioned in paragraph 2 hereof does not signify unlawful censorship and neither it mean that it will refuse to furnish the petitioner with air time because the latter merely refused to submit its speeches and never came back. 5. The mandamus should not issue in the above entitled case because the facts alleged in the petition do not warrant the same, the act of the respondent not constituting an unlawful exclusion of the petitioner from the use and enjoyment of a right to which it is entitled. 6. The mandamus is not the proper remedy in the instant case. The case hinges on whether the petitioner has a clear legal right to broadcast any speech over one of the radio stations owned and operated by the respondent without first submitting the manuscript, and whether there is a corresponding duty devolved by law upon the respondent to permit the petitioner to do so. We note at the outset that the respondent did not actually refuse to sell air time to the

petitioner. It required the petitioner to submit the script of the speeches to be broadcasted, and the petitioner refused to do so. So the question narrows down to whether or not the Far Eastern Broadcasting has the right to require the submission of the script. Section 2 of Act No. 3180, which is the franchise of the respondent corporation, provides that the broadcasting service shall be open to the general public subject to the general regulations of the grantee for the allotment of time and the class of communications acceptable for broadcasting. Commonwealth Act No. 98 created a Radio Board, among the duties of which is "to censor all programs, sustaining programs, sustaining or sponsored, to be broadcasted or rebroadcasted by all broadcasting stations." Section 2 of said Act Provides that pending the appointment of the Radio Board by the President, the Secretary of the Interior "shall examine all programs, sustaining or sponsored, of all broadcasting stations," with the power "to eliminate or cancel from the program such number or parts thereof as in his opinion are neither moral, educational nor entertaining, and prejudicial to public interest." Section 3 provides that "any license or owner of broadcasting station who shall broadcast any program or parts thereof not duly approved or otherwise eliminated, by the Secretary of the Interior or the Radio Board, shall, upon his or its recommendation, as the case may be, forfeit his license." Pursuant to said Act, the Secretary of the Interior, on November 18, 1936, promulgated Department Order No. 13. Paragraph 4 of said order requires all broadcasting stations to submit daily to the Secretary of the Interior at least twenty-four hours in advance of the actual broadcasting hour, two copies of all programs to be broadcasted by the stations. Paragraph 6 provides that "if a program contains any speeches, ... copies of these or a gist thereof, may be required by the Secretary of the Interior to be submitted together with the program. The respondent denies that its requirement that the petitioner submit the script of the speeches to be broadcasted was discriminatory, asserting that "it is applied

to all persons alike"; and the petitioner has not adduced any proof of its allegation. It seems clear from the laws and the regulations hereinbefore adverted to that the respondent had the right to require the petitioner to submit the manuscript of the speeches which he intended to broadcast. In his memorandum counsel for the respondent sustains, and cites numerous authorities in support of his contention, that Commonwealth Act No. 98 is valid as a proper exercise by the state of its police power. However, we are not called upon here to inquire into the constitutionaly and validity of said Act, which directly governs this case, because the petitioner has not specifically raised that question and the general rule is that the constitutionality of a law will not be considered unless the point is specially pleaded, insisted upon, and adequately argued. (11 Am. Jur., 774; 16 C. J. S., 220-338.) "The basic principle which underlies the entire field of legal concepts pertaining to the validity of legislation is that by enactment of legislation, a constitutional measure is presumed to be created. (11 Am. Jur., Constitutional Law, sec. 128.) The petitioner argues that the broadcasting of the speeches of the candidates of the Sumulong Popular Front Party in the evening of September 23, 1941, could not by any means offer any danger to public safety or public morality, and that the respondent was not justified "to previously censor and approve the complete texts of the speeches of said candidates allowing the same to be broadcasted in its station." The best proof, the petitioner asserts, that there was not the slightest danger to public peace and order was that the speeches that were delivered in the Opera House in the evening of September 23, 1941, were heard by the vast audience and published in some newspapers without causing a single untoward incident. We fail to perceive the cogency of such argument. It does not bolster up the case for the petitioner. It impliedly admits and correctly, we think that a speech that may endanger public safety may be censored and disapproved for broadcasting. How could the censor verify the petitioner's claim that the speeches he intended to broadcast offered no danger to public safely or public morality, if the petitioner refused to submit the manuscript or even the gist thereof? If the petitioner had complied with respondent's requirement

and the respondent had arbitrarily and unreasonably refused to permit said speeches to be broadcasted, he might have reason to complain. We find it is not the duty of the respondent as a public service corporation to broadcast speeches without requiring the submission of the manuscript thereof in advance, but that, on the contrary, the laws and regulations expressly authorize the respondent to make such requirement. Hence the petition for mandamus must be, as it hereby, is, denied, with cost against the petitioner, So ordered. Primcias v Fugoso Facts: An action was instituted by the petitioner for the refusal of the respondent to issue a permit to them to hold a public meeting in Plaza Miranda for redress of grievances to the government. The reason alleged by the respondent in his defense for refusing the permit is, "that there is a reasonable ground to believe, basing upon previous utterances and upon the fact that passions, specially on the part of the losing groups, remains bitter and high, that similar speeches will be delivered tending to undermine the faith and confidence of the people in their government, and in the duly constituted authorities, which might threaten breaches of the peace and a disruption of public order." Giving emphasis as well to the delegated police power to local government. Stating as well Revised Ordinances of 1927 prohibiting as an offense against public peace, and penalizes as a misdemeanor, "any act, in any public place, meeting, or procession, tending to disturb the peace or excite a riot; or collect with other persons in a body or crowd for any unlawful purpose; or disturb or disquiet any congregation engaged in any lawful assembly." Included herein is Sec. 1119, Free use of Public Place.1 Issue: Whether or Not the freedom of speech was violated. Held: Yes. Dealing with the ordinance, specifically, Sec. 1119, said section provides for two constructions: (1) the Mayor of the City of Manila is vested with unregulated discretion to grant or refuse, to grant permit for the holding of a lawful assembly or meeting, parade, or procession in the streets and other public places of the City of Manila; (2) The right of the Mayor is subject

to reasonable discretion to determine or specify the streets or public places to be used with the view to prevent confusion by overlapping, to secure convenient use of the streets and public places by others, and to provide adequate and proper policing to minimize the risk of disorder. The court favored the second construction. First construction tantamount to authorizing the Mayor to prohibit the use of the streets. Under our democratic system of government no such unlimited power may be validly granted to any officer of the government, except perhaps in cases of national emergency. The Mayors first defense is untenable. Fear of serious injury cannot alone justify suppression of free speech and assembly. It is the function of speech to free men from the bondage of irrational fears. To justify suppression of free speech there must be reasonable ground to fear that serious evil will result if free speech is practiced. There must be reasonable ground to believe that the danger apprehended is imminent. There must be reasonable ground to believe that the evil to be prevented is a serious one . The fact that speech is likely to result in some violence or in destruction of property is not enough to justify its suppression. There must be the probability of serious injury to the state.

Mutuc v COMELEC FACTS: The Commission on Elections (COMELEC) prohibited petitioner Amelito Mutuc, a candidate for the position of a delegate to the Constitutional Convention, from using jingles in his mobile units equipped with sound systems and loud speakers on 22 October 1970. Petitioner impugned the act of respondent as violative of his right to free speech. Respondent however contended that the prohibition was premised on a provision of the Constitutional Convention Act, which made it unlawful for candidates to purchase, produce, request or distribute sample ballots, or electoral propaganda gadgets such as pens, lighters, fans (of
whatever nature), flashlights, athletic goods or materials, wallets, bandanas, shirts, hats, matches, cigarettes, and the like, whether of domestic or foreign origin. It was its contention that the jingle proposed to be used by petitioner is the recorded or taped voice of a singer and therefore a tangible propaganda material, under the phrase and the like.

ISSUE: Whether jingles falls down on the prohibited electoral propaganda gadgets of R.A. No. 6132. RULING: For respondent Commission, the last three words sufficed to justify such an order. We view the matter differently. What was done cannot merit our approval under the wellknown principle of ejusdem generis, the general words following any enumeration being applicable only to things of the same kind or class as those specifically referred to. It is quite apparent that what was contemplated in the Act was the distribution of gadgets of the kind referred to as means of inducement to obtain a favorable vote for the candidate responsible for distribution. Navarro v Villegas "In Case G.R. No. L-31687 (Navarro vs. Villegas), the Court, after considering the pleadings and arguments of the parties, issued the following Resolution: Without prejudice to a more extended opinion and taking into account the following considerations: That respondent Mayor has not denied nor absolutely refused the permit sought by petitioner; That as stated in Primicias v. Fugoso, 80 Phil. 75, respondent Mayor possesses reasonable discretion to determine or specify the streets or public places to be used for the assembly in order to secure convenient use thereof by others and provide adequate and proper policing to minimize the risks of disorder and maintain public safety and order; That respondent Mayor has expressly stated his willingness to grant permits for peaceful assemblies at Plaza Miranda during Saturdays, Sundays and holidays when they
would not cause unnecessarily great disruption of the normal activities of the community and has further offered Sunken Gardens as an alternative to Plaza Miranda as the site of the demonstration sought to be held this afternoon; That experiences in connection with present assemblies and demonstrations do not warrant the Court's disbelieving respondent Mayor's

appraisal that a public rally at Plaza Miranda, as compared to one at the Sunken Gardens as he suggested, poses a clearer and more imminent danger of public disorders, breaches of the peace, criminal acts, and even bloodshed as an aftermath of such assemblies, and petitioner has manifested that it has no means of preventing such disorders; That, consequently, every time that such assemblies are announced, the community is placed in such a state of fear and tension that offices are closed early and employees

ground that there was no showing of the existence of a clear and present danger of a substantive evil that could justify the denial of a permit. However Justice Aquino dissented that the rally is violative of Ordinance No. 7295 of the City of Manila prohibiting the holding of rallies within a radius of five hundred (500) feet from any foreign mission or chancery and for

other purposes. Hence the Court resolves. Issue: Whether or Not the freedom of expression and the right to peaceably assemble violated. Held: Yes. The invocation of the right to freedom of peaceable assembly carries with it the implication that the right to free speech has likewise been disregarded. It is settled law that as to public places, especially so as to parks and streets, there is freedom of access. Nor is their use dependent on who is the applicant for the permit, whether an individual or a group. There can be no legal objection, absent the
existence of a clear and present danger of a substantive evil, on the choice of Luneta as the place where the peace rally would start. Time immemorial Luneta has been used for purposes of assembly, communicating thoughts between citizens, and discussing public questions. Such use of the public places has from ancient times, been a part of the privileges, immunities, rights, and liberties of citizens. With regard to the ordinance, there was no showing that there was violation and even if it could be shown that such a condition is satisfied it does not follow that respondent could legally act the way he did. The validity of his denial of the permit sought could still be challenged. A summary of the application for permit for rally: The applicants for a permit to hold an assembly should inform the licensing authority of the date, the public place where and the time when it will take place. If it were a private place, only the consent of the owner or the one entitled to its legal possession is required.

dismissed, storefronts boarded up, classes suspended, and transportation disrupted, to the general detriment of the public: That civil rights and liberties can exist and be preserved only in an order society; The petitioner has failed to show a clear specific legal duty on the part of respondent Mayor to grant their application for permit unconditionally; The Court resolved to DENY the writ prayed for and to dismiss the petition. Reyes v Bagatsing Facts: Petitioner sought a permit from the City of Manila to hold a peaceful march and rally on October 26, 1983 from 2:00 to 5:00 in the afternoon, starting from the Luneta to the gates of the United States Embassy. Once there, and in an open space of public property, a short program would be held. The march would be attended by the local and foreign participants of such conference. That would be followed by the handing over of a petition based on the resolution adopted at the closing session of the AntiBases Coalition. There was likewise an assurance in the petition that in the exercise of the constitutional rights to free speech and assembly, all the necessary steps would be taken by it "to ensure a peaceful march and rally. However the request was denied. Reference was made to persistent intelligence reports affirming the plans of subversive/criminal elements to infiltrate or disrupt any assembly or congregations where a large number of people is expected to attend. Respondent suggested that a permit may be issued if it is to be held at the Rizal Coliseum or any other enclosed area where the safety of the participants themselves and the general public may be ensured. An oral argument was heard and the
mandatory injunction was granted on the

Ruiz v Gordon Facts: Hector S. Ruiz, Coordinator of Olongapo Citizen's Alliance for National Reconciliation, filed a petition for mandamus against Richard Gordon to be allowed to hold a parade/march from Gordon Avenue to the Rizal Triangle starting at 1:00 P.M. The Court required the respondents to answer. Respondents replied by stating the request for a prayer rally was received in the Office of the Mayor and that respondent had repeatedly announced in his regular program on Sunday over the radio (DWGO) and at the Monday morning flag ceremony before hundreds of government employees that he would grant the request of any group that would like to exercise their freedom of speech and assembly. When interviewed on the matter by the Editor-in Chief of the 'Guardian', he mentioned the fact that he had granted the permit of the petitioner, which interview appeared in the November 22-28, 1983 issue of the said newspaper. Given these, the respondent prayed for the dismissal of the petition. This was complied with. Issue: Can the petition be granted? Held No. Petition dismissed. Ratio: The Reyes case was given some discussion in the course of this petition as to the role of the judiciary in petitions for permits to hold peaceable assembles. "The applicants for a permit to hold an assembly should inform the licensing authority of the date, the public place where and the time when it will take place. If it were a private place, only the consent of the owner or the one entitled to its legal possession is required. Such application should be filed well ahead in time to enable the public official concerned to appraise whether there may be valid objections to the grant of the permit or to its grant but at another public place. It is an indispensable condition to such refusal or modification that the clear and present danger test be the standard for the decision reached. If he is of the view that there is such an imminent and grave danger of a substantive evil, the applicants must be heard on the matter.

Such application should be filed well ahead in time to enable the public official concerned to appraise whether there may be valid objections to the grant of the permit or to its grant but at another public place. It is an indispensable condition to such refusal or modification that the clear and present danger test be the standard for the decision reached. Notice is given to applicants for the denial.

Thereafter, his decision must be transmmitted to them at the earliest opportunity. They can have recourse to the proper judicial authority. Free speech and peaceable assembly, along with the other intellectual freedoms, are highly ranked in our scheme of constitutional values. It cannot be too strongly stressed that on the judiciary, even more so than on the other departments rests the grave and delicate responsibility of assuring respect for and deference to such preferred rights. As shown both in the manifestation and the answer, this action for mandamus could have been obviated if only petitioner took the trouble of verifying on November 23 whether or not a permit had been issued. A party desirous of exercising the right to peaceable assembly should be the one most interested in ascertaining the action taken on a request for a permit. Necessarily, after a reasonable time or, if the day and time was designated for the decision on the request, such party or his representative should be at the office of the public official concerned. If he fails to do so, a copy of the decision reached, whether adverse or favorable, should be sent to the address of petitioner.

appellant can only be convicted of simple rape because he cannot be punished for a graver offense than that with which he was charged. Under the rules of criminal procedure, a qualifying circumstance to be considered as such must be so alleged in the information, which is not required of aggravating circumstances. The requirement for complete allegations on the particulars of the indictment is based on the right of the accused to be fully informed of the nature of the charge against him, so that be may adequately prepare for this defense pursuant to the due process clause of the Constitution. American Communications Assoc v Douds 339 U.S. 382 (1950), argued 1011 Oct. 1949, decided 8 May 1950 by vote of 5 to
1; Vinson for the Court, Frankfurter concurring in part, Jackson concurring and dissenting, black in dissent, Douglas, Clark, and Minton not participating. This case involved the constitutionality of Cold Warera anticommunist legislation. The Supreme Court upheld section 9(h) of the Taft Hartley Act (1947), which required officers of labor unions to sign affidavits indicating that they were not Communist party members or supporters and did not believe in unlawful overthrow of the U.S. government. Unions whose officers did not sign affidavits were unable to seek relief before the National Labor Relations Board for unfair labor practices. The Court did not rest its judgment on a threat to national security, but on a threat to interstate commerce. The majority found that the statute fell within the broad scope of Congress's commerce power because the Communist party could reasonably be expected to engage in political strikes that were disruptive of the national economy. The Court recognized that the statute had a chilling effect on political rights protected by the First Amendment. Nevertheless, it ruled that the First Amendment was not violated because that statute protected the public from harmful conductpolitical strikesnot harmful ideas. The Court then applied the clear and present danger test as a simple balancing test and concluded that Congress's interest in protecting the nation from political strikes outweighed the burden the act placed on the rights of union members. Although Douds has not been specifically overturned, it is dubious authority. The statute replacing section 9(h) was struck down by the court in United States v. Brown (1965).

US v Bustos Facts: In the latter part of 1915, numerous citizens of the Province of Pampanga assembled, and prepared and signed a petition to theExecutive Secretary(privileged communication) through the law office of Crossfield and O'Brien, and five individuals signed affidavits, charging Roman Punsalan, justice of the peace of Macabebe and Masantol, Pampanga, with malfeasance in office and asking for his removal. The specific charges against the justice of the peace include the solicitation of money from persons who have pending cases before the judge. Now, Punsalan alleged that accused published a writing which was false, scandalous, malicious, defamatory, and libelous against him. Issue: Whether or Not accused is entitled to constitutional protection by virtue of his right to free speech and free press. Held: Yes. The guaranties of a free speech and a free pressinclude the right to criticize judicial conduct. The administration of the law is a matter of vital public concern. Whether the law is wisely or badly enforced is, therefore, a fit subject for proper comment. If the people cannot criticize a justice of the peace or a judge the same as any other public officer, public opinion will be effectively suppressed. It is a duty which every one owes to society or to the State to assist in the investigation of any alleged misconduct. It is further the duty of all who know of any official dereliction on the part of a magistrate or the wrongful act of any public officer to bring the facts to the notice of those whose duty it is to inquire into and punish them. The right to assemble and petition is the necessary consequence of republican institutions and the complement of the part of free speech. Assembly means a right on the part of citizens to meet peaceably for consultation in respect to public affairs. Petition means that any person or group of persons can apply, without fear of penalty, to the appropriate branch or office of the government for a redress of grievances. The persons assembling and petitioning must, of course, assume responsibility for the charges made. All persons have an interest in the pure and efficient administration of justice and of public affairs.

Public policy, the welfare of society, and the orderly administration ofgovernment have demanded protection for public opinion. The inevitable and incontestable result has been the development and adoption of the doctrine of privilege. All persons have an interest in the pure and efficient administration of justice and of public affairs. The duty under which a party is privileged is sufficient if it is social or moral in its nature and this person in good faith believes he is acting in pursuance thereof although in fact he is mistaken. Although the charges are probably not true as to the justice of the peace, they were believed to be true by the petitioners. Good faith surrounded their action. Probable cause for them to think that malfeasance or misfeasance in office existed is apparent. The ends and the motives of these citizens to secure the removal from office of a person thought to be venal were justifiable. In no way did they abuse the privilege. In the usual case malice can be presumed from defamatory words. Privilege destroys that presumption. A privileged communication should not be subjected to microscopic examination to discover grounds of malice or falsity. Rosenbloom v Metromedia Respondent's radio station, which broadcast news reports every half hour, broadcast news stories of petitioner's arrest for possession of obscene literature and the police seizure of "obscene books," and stories concerning petitioner's lawsuit against certain officials alleging that the magazines he distributed were not obscene and seeking injunctive relief from police interference with his business. These latter stories did not mention petitioner's name, but used the terms "smut literature racket" and "girlie-book peddlers." Following petitioner's acquittal of criminal obscenity charges, he filed this diversity action in District Court seeking damages under Pennsylvania's libel law. The jury found for petitioner and awarded $25,000 in general damages; and $725,000 in punitive damages, which was reduced by the court on remittitur to $250,000. The Court of Appeals reversed, holding that the New York Times Co. v. Sullivan, 376 U.S. 254, standard applied, and "the fact that plaintiff was not a public figure cannot be accorded decisive significance."

PP v Perez FACTS: Accused-appellant was charged for rape. The information alleged that the victim was his stepdaughter but did not allege that the victim was only 13 years old at the time of the rape. During arraignment, appellant pleaded not guilty to the accusation against him. During trial it was proved that the victim was the stepdaughter of the accused and was in fact 13 years old at the time the crime of rape was committed. Accused was then convicted of qualified rape and was meted out the penalty of death. ISSUE: WON an accused may be convicted of qualified rape when the information alleged only simple rape? HELD: NO. Citing People vs. Garcia, the court held that it would be a denial of the right of the accused to be informed of the charges against him and, consequently, a denial of due process, if he is charged with simple rape and be convicted of its qualified form punishable by death, although the attendant circumstance qualifying the offense and resulting in capital punishment was not alleged in the indictment on which he was arraigned. Procedurally, then, while the minority of Maribel and the relationship of appellant and his victim were established during the trial,

Held: The judgment is affirmed. Pp. 40-62. MR. JUSTICE BRENNAN, joined by THE CHIEF JUSTICE and MR. JUSTICE BLACKMUN, concluded that the New York Times standard of knowing or reckless falsity applies in a state civil libel action brought by a private individual for a defamatory falsehood uttered in a radio news broadcast about the individual's involvement in an event of public or general interest. Pp. 40-57. MR. JUSTICE BLACK concluded that the First Amendment protects the news media from libel judgments even when statements are made with knowledge that they are false. P. 57. MR. JUSTICE WHITE concluded that, in the absence of actual malice as defined in New York Times, supra, the First Amendment gives the news media a privilege to report and comment upon the official actions of public servants in full detail, without sparing from public view the reputation or privacy of an individual involved in or affected by any official action. Pp. 59-62. [p30] Lagunzad v Sotto vda. De Gonzales Facts: The present controversy stems from a "Licensing Agreement" entered into by and between petitioner Manuel M. Lagunzad and private respondent Maria Soto Vda. de Gonzales. Petitioner Manuel Lagunzad, a newspaperman, began the production of a movie entitled "The Moises Padilla Story" under the name of his own business outfit, the "MML Productions." It was based mainly on the copyrighted but unpublished book of Atty. Ernesto Rodriguez, Jr., entitled "The Long Dark Night in Negros" subtitled "The Moises Padilla Story," 1 the rights to which petitioner had purchased from Atty. Rodriguez in the amount of P2,000.00. Although the emphasis of the movie was on the public life of Moises Padilla, there were portions which dealt with his private and family life including the portrayal in some scenes, of his mother, Maria Soto Vda. de Gonzales, private respondent herein, and of one "Auring" as his girl friend. Mrs. Amante, for and in behalf of her mother, private respondent, demanded in writing for certain changes, corrections and deletions in the movie. Petitioner takes the position that he

was pressured into signing the Agreement because of private respondent's demand, through Mrs. Amante, for payment for the "exploitation" of the life story of Moises Padilla, otherwise, she would "call a press conference declaring the whole picture as a fake, fraud and a hoax and would denounce the whole thing in the press, radio, television and that they were going to Court to stop the picture. Issue: Whether or not the Licensing agreement infringes his right to freedom of speech and the press Ruling: Petition for Review is denied and the judgment appealed from hereby affirmed. Costs against petitioner . While it is true that petitioner had purchased the rights to the book entitled "The Moises Padilla Story," that did not dispense with the need for prior consent and authority from the deceased heirs to portray publicly episodes in said deceased's life and in that of his mother and the members of his family. As held in Schuyler v. Curtis, 14 "a privilege may be given the surviving relatives of a deceased person to protect his memory, but the privilege exists for the benefit of the living, to protect their feelings and to prevent a violation of their own rights in the character and memory of the deceased." Petitioner's averment that private respondent did not have any property right over the life of Moises Padilla since the latter was a public figure, is neither well taken. Being a public figure ipso facto does not automatically destroy in toto a person's right to privacy. The right to invade a person's privacy to disseminate public information does not extend to a fictional or novelized representation of a person, no matter how public a figure he or she may be. . A contract is valid even though one of the parties entered into it against his own wish and desires, or even against his better judgment. neither do we find merit in petitioner's contention that the Licensing Agreement infringes on the constitutional right of freedom of speech and of the press, in that, as a citizen and as a newspaperman, he had the right to express his thoughts in film on the public life of Moises Padilla without prior restraint. The right of freedom of expression, indeed, occupies a preferred position in the "hierarchy of civil liberties. Another criterion for permissible limitation on freedom of speech and of the press, which includes

such vehicles of the mass media as radio, television and the movies, is the "balancingof-interests test. the interests observable are the right to privacy asserted by respondent and the right of -freedom of expression invoked by petitioner. the Licensing Agreement entered into by petitioner, the validity of such agreement will have to be upheld particularly because the limits of freedom of expression are reached when expression touches upon matters of essentially private concern. Ayer Production ltd. V Judge Capulong F: Pivate respondent Juan Ponce Enrile filed an action in the RTC of Makati to enjoin the petitioners from producing the movie "The Four Day Revolution," a documentary of the EDSARevolution in 1986 on the ground that it violated his right to privacy. Petitioners contended that the movie would not involve his private life not that of his family. But the trial court issued a writ of preliminary injunction and ordered petitioners to desist from making the movie making reference whatsoever to Ponce Enrile. This, this action for certiorari. HELD: Freedom of speech and expression includes freedom to produce motion pictures and to exhibit them. What is involved is a prior restraint by the Judge upon the exercise of speech and of expression by petitioners. Because of the preferred character of speech and of expression, a weighty presumption of invalidity vitiates measures of prior restraint. The Judge should have stayed his hand considering that the movie was yet uncompleted and therefore there was no "clear and present danger." The subject matter of the movie does not relate to the private life of Ponce Enrile. The intrusion is no more than necessary to keep the film a truthful historical account. He is, after all, a public figure. The line of equilibrium in the specific context of the instant case between freedom of speech and of expression and the right of privacy may be marked out in terms of a requirement that the proposed motion picture must be fairly truthful and historical in its presentation of facts. There must be no showing of a reckless disregard of truth. Notes: Ayer sought to produce a movie on the 4-day revolution. Enrile, who had previously been asked for the use of his character in the movie and had refused the offer, sued to enjoin the filming because he

did not want any mention of his and his family's name. The SC lifted the injunction issued by the lower court on the ground that it amounted to prior restraint, which is no better if imposed by the courts than if imposed by administrative bodies or by ecclesiatical officials. In Ayer, the reference to Enrile is unavoidable because his name is part of history and this cannot be changed or altered; thus his name can be used so long as only his public life is dwelled only. But in Lagunzad, although Moises Padilla was also a public figure, the movie dealth with both the public and private lives of Moises Padilla. Zaldivar v Sandiganbayan Facts: The case stemmed from the resolution of the Supreme Court stopping the respondent from investigating graft cases involving Antique Gov. Enrique Zaldivar. The Court ruled that since the adoption of the 1987 Constitution, respondents powers as Tanodbayan have been superseded by the creation of the Office of the Ombudsman, he however becomes the Special Prosecutor of the State, and can only conduct an investigation and file cases only when so authorized by the Ombudsman. A motion for reconsideration was filed by the respondent wherein he included statements which were unrelated in the Issue raised in the Court. This include: (a)That he had been approached twice by a leading member of the court and he was asked to 'go slow on Zaldivar and 'not to be too hard on him; (b) That he "was approached and asked to refrain from investigating the COA report on illegal disbursements in the Supreme Court because 'it will embarass the Court;" and (c) that in several instances, the undersigned respondent was called over the phone by a leading member of the Court and was asked to dismiss the cases against two Members of the Court." Statements of the respondent saying that the SCs order '"heightens the people's apprehension over the justice system in this country, especially because the people have been thinking that only the small fly can get it while big fishes go scotfree was publicized in leading newspapers. Now, the Court Resolved to require respondent to explain in writing why he should not be punished for contempt of court for making such public statements reported in the media. Respondent then sought to get some members of the Court to inhibit themselves in the resolution of the Zaldivar case for alleged bias and prejudice

against him. A little later, he in effect asked the whole Court to inhibit itself from passing upon the Issue involved in proceeding and to pass on responsibility for this matter to the Integrated Bar of the Philippines, upon the ground that respondent cannot expect due process from this Court, that the Court has become incapable of judging him impartially and fairly. The Court found respondent guilty of contempt of court and indefinitely suspended from the practice of law. Now, he assails said conviction, invoking his freedom of speech. Counsel for respondent urges that it is error "for this Court to apply the "visible tendency" rule rather than the "clear and present danger" rule in disciplinary and contempt charges." Issue: Whether or Not there was a violation of the freedom of speech/expression. Held: There was no violation. The Court did not purport to announce a new doctrine of "visible tendency," it was simply paraphrasing Section 3 (d) of Rule 71 of the Revised Rules of Court which penalizes a variety of contumacious conduct including: "any improper conduct tending, directly or indirectly, to impede, obstruct or degrade the administration of justice." Under either the "clear and present danger" test or the "balancing-of-interest test," the Court held that the statements made by respondent Gonzalez are of such a nature and were made in such a manner and under such circumstances, as to transcend the permissible limits of free speech. What is here at stake is the authority of the Supreme Court to confront and prevent a "substantive evil" consisting not only of the obstruction of a free and fair hearing of a particular case but also the avoidance of the broader evil of the degradation of the judicial system of a country and the destruction of the standards of professional conduct required from members of the bar and officers of the courts, which has some implications to the society. US v Kottinger People vs. Kottinger 45 Phil. 352 FACTS: Accused Kottinger's camera business store was raided. Among the materials confiscated were some pictures that show Filipino inhabitants in their native

dress. Using these items, he was charged of violating section 12 of Act No. 277, the Philippine Libel Law. His pictures were being used as post cards of the non-Christian natives of the country. HELD: (Malcolm, J.) Although Philippine laws do not define what obscenity means, the Court defined obscene or obscenity as "something offensive to chastity, decency, or delicacy." There are two tests whether something is obscene: (1) whether it corrupts the mind of the viewers to such immoral influences, or (2) it shocks the ordinary and common sense of men as an indecency. In the case at bar, the pictures merely portrayed the inhabitants in their native dress as testified by a UP Professor. Moreover, there are pictures of similar nature which are imported and circulated in the Philippines, such as a book about the Ifugaos. As such, the pictures were not obscene within the meaning of the law. DISSENTING: (Araullo, C.J.) The pictures may not be indecent in the places where they were taken, however, should the pictures be shown in Manila, they will be taken as indecent. Gonzales v Katigbak FACTS: Petitioner was the producer of the movie Kapit sa Patalim which the Board of Review for Motion Pictures and Televisions allowed on condition that certain deletions were made and that it was shown on adults only. The petitioner brought an action, claiming violation of their freedom of expression. HELD: Motion pictures are important both as a method for the communication of ideas and the expression of the artistic impulse. The power of the Board is limited to the classification of films. For freedom of expression is the rule and restrictions the exception. The power to impose prior restraint is not to be presumed, rather the presumption is against its validity. Censorship is allowable only under the clearest proof of a clear and present danger of a substantive evil to public safety, public morals, public health or any other legitimate public interest. The Board committed an abuse of discretion in subjecting petitioner to difficulty and travail before the movie was classified as "For adults only" without deletion. However there is not enough votes to consider the abuse of discretion grave as it explained that there were reasons for its

action because of the scenes showing women erotically dancing naked and kissing and caressing each other like lesbians. VV. Notes: The movie involved in this case was "Kapit sa Patalim" which the censors wanted to cut in some part and to label "For Adults". The SC rules that movies are within the constitutional protection of freedom of expression, so that censorship is presumed to be valid as constituting prior restraint. The only case whe the Board of Censors can order a deletion is when there is a clear and present danger of a substantive evil against national security or public morals or other public interest. In all other cases, the Board can only classify. But a different standard must be followed in television because of the pervasive and intrusive influence of the medium on people who watch its programs without having to pay anything. On the issue of obscenity, the SC held that sex along is not necessarily obscenity, the test being whether, using contemporary community standards, the dominant appeal us to the prurient interest. (Miller v. California). Thus on this score, it found abuse of discretion of the part of the Board for subjecting the producer to difficulty and for entertaining a narrow view of obscenity, but it lacked the votes to rules that the abuse was grave. People vs. Go Pin (1955) Go Pin, an alien and Chinese citizen, was charged with violation of Article 201 (immoral doctrines, obscene publications and exhibitions) for exhibiting in Manila at the Globe Arcade (a recreation center) a large number of allegedly indecent and/or immoral films. He was convicted under the trial court; hence, this present appeal. Go Pin claims that there are paintings and pictures of women in the nude, for the sake of art. Held: The pictures in the present case were not used for arts sake but for commercial purposes. The supposed artistic qualities of the said pictures were being commercialized so that the cause of art was of secondary or minor importance. The people who come to see the pictures in the present case would be desirous of satisfying their lust and love for excitement, not to appreciate their artistic value. Decision appealed from is affirmed.

Miller v California Appellant was convicted of mailing unsolicited sexually explicit material in violation of a California statute that approximately incorporated the obscenity test formulated in Memoirs v. Massachusetts, 383 U.S. 413, 418 (plurality opinion). The trial court instructed the jury to evaluate the materials by the contemporary community standards of California. Appellant's conviction was affirmed on appeal. In lieu of the obscenity criteria enunciated by the Memoirsplurality, it is held: 1. Obscene material is not protected by the First Amendment. Roth v. United States, 354 U.S. 476, reaffirmed. A work may be subject to state regulation where that work, taken as a whole, appeals to the prurient interest in sex; portrays, in a patently offensive way, sexual conduct specifically defined by the applicable state law; and, taken as a whole, does not have serious literary, artistic, political, or scientific value. Pp. 23-24. 2. The basic guidelines for the trier of fact must be: (a) whether "the average person, applying contemporary community standards" would find that the work, taken as a whole, appeals to the prurient interest, Roth, supra, at 489, (b) whether the work depicts or describes, in a patently offensive way, sexual conduct specifically defined by the applicable state law, and (c) whether the work, taken as a whole, lacks serious literary, artistic, political, or scientific value. If a state obscenity law is
thus limited, First Amendment values are adequately protected by ultimate independent appellate review of constitutional claims when necessary. Pp. 24-25. 3. The test of "utterly without redeeming social value" articulated in Memoirs, supra, is rejected as a constitutional standard. Pp. 24-25. 4. The jury may measure the essentially factual issues of prurient appeal and patent offensiveness by the standard that prevails in the forum community, and need not employ a "national standard." Pp. 30-34. Vacated and remanded.

Dela Cruz v Ela Petitioners, in their behalf and for the benefit of other Jehovahs Witnesses in the province of Zambales, brought this action to compel Respondent to grant them a permit to hold a public meeting at the public plaza of Sta. Cruz, Zambales, together with the kiosk, on such date and time as may be applied for by them. Respondent in his answer stated that he had not refused the request of Petitioners to hold a religious meeting at the public plaza as in fact he grave them permission to use the northwestern part of the plaza on July 27, 1952, but they declined to avail of it. He prayed that the action be dismissed. The questions of fact raised in the pleadings being not controverted, and Petitioners having submitted a motion for judgment on the pleadings, which was concurred in by Respondent, the court rendered a decision dismissing the case without pronouncement as to costs. Plaintiffsappealed from this decision. It appears that Petitioners are members of the Watch Tower Bible and Tract Society, commonly known as Jehovahs Witnesses, whose tenets and principles are derogatory to those professed by the Catholic organization. In its publication FACE THE FACTS, that society branded the latter as a religious organization which is a part of the monstrosity now appearing in and claiming the right to rule the earth. Desiring to hold a meeting in furtherance of its objectives, Petitionersasked Respondent to give them permission to use the public plaza together with the kiosk, but, instead of granting the permission, Respondent allowed them to hold their meeting on the northwestern part corner of the plaza. He adopted as a policy not to allow the use of the kiosk for any meeting by any religious denomination as it is his belief that said Kiosk should only be used for legal purposes. And when their request for reconsideration was denied, Petitionersinstituted the present action for mandamus. It is now contended by Petitioners that the action taken by Respondent is unconstitutional being an abridgment of the freedom of speech, assembly, and worship guaranteed by our Constitution. The issue raised involves a little digression on the extent to which the right to peacefully assemble guaranteed by the Constitution may be invoked. Fortunately, this issue has

already been passed upon by this Court in Primicias vs. Fugoso, 45 Official Gazette, 3280, wherein this Court said:chanroblesvirtuallawlibrary The right to freedom of speech, and to peacefully assemble and petition the government for redress of grievances, are fundamental personal rights of the people recognized and guaranteed by the constitutions of democratic countries. But it is a settled principle growing out of the nature of well-ordered civil societies that the exercise of those rights is not absolute for it may be so regulated that it shall not be injurious to the equal enjoyment of others having equal rights, nor injurious to the rights of the community or society. The power to regulate the exercise of such and other constitutional rights is termed the sovereign police power, which is the power to prescribe regulations, to promote the health, morals, peace, education, good order or safety, and general welfare of the people. This sovereign police power is exercised by the government through its legislative branch by the enactment of laws regulating those and other constitutional and civil rights, and it may be delegated to political subdivisions, such as towns, municipalities and cities by authorizing their legislative bodies celled municipal and city councils to enact ordinances for the purpose. It therefore appears that the right to freedom of speech and to peacefully assemble, though guaranteed by our Constitution, is not absolute, for it may be regulated in order that it may not be injurious to the equal enjoyment of others having equal rights, nor injurious to the rights of the community or society, and this power may be exercised under the police power of the state, which is the power to prescribe regulations to promote the health, morals, peace, education, good order or safety, and general welfare of the people. It is true that there is no law nor ordinance which expressly confers upon Respondents the power to regulate the use of the public plaza, together with its kiosk, for the purposes for which it was established, but such power may be exercised under his broad powers as chief executive in connection with his specific duty to issue orders relating to the police or to public safety within the municipality (section 2194, paragraph c, Revised Administrative Code). And it may even be said that the above regulation has been adopted as an implementation of the

constitutional provision which prohibits any public property to be used, directly or indirectly, by any religious denomination (paragraph 3, section 23, Article VI of the Constitution). The power exercised by Respondent cannot be considered as capricious or arbitrary considering the peculiar circumstances of this case. It appears that the public plaza, particularly the kiosk, is located at a short distance from the Roman Catholic Church. The proximity of said church to the kiosk has caused some concern on the part of the authorities that to avoid disturbance of peace and order, or the happening of untoward incidents, they deemed it necessary to prohibit the use of that kiosk by any religious denomination as a place of meeting of its members. This was the policy adopted by Respondent for sometime previous to the request made by Petitioners.Respondent never denied such request but merely tried to enforce his policy by assigning them the northwestern part of the public plaza. It cannot therefore be said that Petitioners were denied their constitutional right to assemble for, as was said, such right is subject to regulation to maintain public order and public safety. This is especially so considering that the tenets ofPetitioners congregation are derogatory to those of the Roman Catholic Church, a factor whichRespondent must have considered in denying their request. It is true that the foregoing conclusion is predicated on facts which do not appear in the pleadings nor are supported by any evidence because none was presented for the reason that the case was submitted on a motion for judgment on the pleadings, but those facts like the situation of the kiosko and the occurrence of religious controversies which disturbed the peace and order in the municipality of Sta. Cruz are matters which may be deemed to come within the judicial knowledge of the court as in fact they were so considered by the trial judge in his decision. This is what he said on this point:chanroblesvirtuallawlibrary The presiding judge, through information, personal experience and through the papers, has known of unfortunate events which caused the disturbance of peace and order in the community. If the Petitioners should be allowed to use the kiosko which is within the hearing distance of the catholic church, this may give rise to disturbance of other religious ceremonies performed in the church. (Italics supplied.) This action of the judge may be justified

under section 5, Rule 123, of the Rules of Court, which is elaborated by this Court in the following wise:chanroblesvirtuallawlibrary There are facts, indeed of which courts should take judicial cognizance. These facts refer to a variety of subjects legislative, political, historical, geographical, commercial, scientific, and artificial in addition to a wide range of matters, arising in the ordinary course of nature or the general current of human events. The matter of judicial notice is ever expanding and will surely keep pace with the advance of the science and the arts. But, a matter to be judicially cognizable must be wellestablished or authoritatively settled, or of common or general knowledge. Obviously, courts should take notice of whatever is or should be generally known because judges should not be more ignorant than the rest of mankind. (The Municipal Board of the City of Manila, et al. vs. Segundo Agustin, 65 Phil., 144.) (Italics supplied.) The contention that the northwestern part of the plaza cannot be considered as part of said plaza but of the road in the northwestern portion beyond the concrete fence is untenable, for it appears that portion is part of the plaza and has a space capable of accommodating hundreds of people. In fact, during the past celebrations of the traditional town fiesta of the municipality, said portion has been utilized by the authorities as a place for staging dramas, zarzuelas, and cinematograph shows. Verily, the pretense of Petitioners cannot be attributed to the unsuitability of that portion as a meeting place but rather to their obstinate desire to use the kiosk knowing it to be contrary to the policy of the municipality. 1 The decision appealed from is affirmed, with costs against Petitioners.

Malabanan v Ramento Facts: Petitioners were officers of the Supreme Student Council of respondent University. They sought and were granted by the school authorities a permit to hold a meeting from 8:00 A.M. to 12:00 P.M, on August 27, 1982. Pursuant to such permit, along with other students, they held a general assembly at the Veterinary Medicine and Animal Science basketball court (VMAS), the place indicated in such permit, not in the basketball court as therein stated but at the second floor lobby. At such gathering they manifested in vehement and vigorous language their opposition to the proposed merger of the Institute of Animal Science with the Institute of Agriculture. The same day, they marched toward the Life Science Building and continued their rally. It was outside the area covered by their permit. Even they rallied beyond the period allowed. They were asked to explain on the same day why they should not be held liable for holding an illegal assembly. Then on September 9, 1982, they were informed that they were under preventive suspension for their failure to explain the holding of an illegal assembly. The validity thereof was challenged by petitioners both before the Court of First Instance of Rizal against private respondents and before the Ministry of Education, Culture, and Sports. Respondent Ramento found petitioners guilty of the charge of illegal assembly which was characterized by the violation of the permit granted resulting in the disturbance of classes and oral defamation. The penalty was suspension for one academic year. Hence this petition. Issue: Whether on the facts as disclosed resulting in the disciplinary action and the penalty imposed, there was an infringement of the right to peaceable assembly and its cognate right of free speech. Held: Yes. Student leaders are likely to be assertive and dogmatic. They would be ineffective if during a rally they speak in the guarded and judicious language of the academe. But with the activity taking place in the school premises and during the daytime, no clear and present danger of public disorder is discernible. This is without prejudice to the taking of disciplinary action for conduct, "materially disrupts classwork or involves substantial disorder or invasion

of the rights of others." The rights to peaceable assembly and free speech are guaranteed students of educational institutions. Necessarily, their exercise to discuss matters affecting their welfare or involving public interest is not to be subjected to previous restraint or subsequent punishment unless there be a showing of a clear and present danger to a substantive evil that the state, has a right to present. As a corollary, the utmost leeway and scope is accorded the content of the placards displayed or utterances made. The peaceable character of an assembly could be lost, however, by an advocacy of disorder under the name of dissent, whatever grievances that may be aired being susceptible to correction through the ways of the law. If the assembly is to be held in school premises, permit must be sought from its school authorities, who are devoid of the power to deny such request arbitrarily or unreasonably. In granting such permit, there may be conditions as to the time and place of the assembly to avoid disruption of classes or stoppage of work of the nonacademic personnel. Even if, however, there be violations of its terms, the penalty incurred should not be disproportionate to the offense.

matter should not unduly prejudice the normal operation of the Company. Workers who without previous leave of absence approved by the Company, particularly , the officers present who are the organizers of the demonstration, who shall fail to report for work the following morning shall be dismissed, because such failure is a violation of the existing CBA and, therefore, would be amounting to an illegal strike. Because the petitioners and their members numbering about 400 proceeded with the demonstration despite the pleas of the respondent Company that the first shift workers should not be required to participate in the demonstration and that the workers in the second and third shifts should be utilized for the demonstration from 6 A.M. to 2 P.M. on March 4, 1969, filed a charge against petitioners and other employees who composed the first shift, for a violation of Republic Act No. 875(Industrial Peace Act), and of the CBA providing for 'No Strike and No Lockout.' Petitioners were held guilty in by CIR for bargaining in bad faith, hence this appeal. Issue: Whether or Not the petitioners right to freedom of speech and to peaceable assemble violated. Held: Yes. A constitutional or valid infringement of human rights requires a more stringent criterion, namely existence of a grave and immediate danger of a substantive evil which the State has the right to prevent. This is not present in the case. It was to the interest herein private respondent firm to rally to the defense of, and take up the cudgels for, its employees, so that they can report to work free from harassment, vexation or peril and as consequence perform more efficiently their respective tasks enhance its productivity as well as profits. Herein respondent employer did not even offer to intercede for its employees with the local police. In seeking sanctuary behind their freedom of expression well as their right of assembly and of petition against alleged persecution of local officialdom, the employees and laborers of herein private respondent firm were fighting for their very survival, utilizing only the weapons afforded them by the Constitution the untrammelled enjoyment of their basic human rights. The pretension of their employer that it would suffer loss or damage by reason of the absence of its employees from 6 o'clock in the morning to

PBM Employees Assoc v PBM Facts: The petitioner Philippine Blooming Mills Employees Organization (PBMEO) is a legitimate labor union composed of the employees of the respondent Philippine Blooming Mills Co., Inc., and petitioners. Benjamin Pagcu and Rodulfo Munsod are officers and members of the petitioner Union. Petitioners claim that on March 1, 1969, they decided to stage a mass demonstration at Malacaang on March 4, 1969, in protest against alleged abuses of the Pasig police. PBMEO thru Pagcu confirmed the planned demonstration and stated that the demonstration or rally cannot be cancelled because it has already been agreed upon in the meeting. Pagcu explained further that the demonstration has nothing to do with the Company because the union has no quarrel or dispute with Management. The Management, thru Atty. C.S. de Leon, Company personnel manager, informed PBMEO that the demonstration is an inalienable right of the union guaranteed by the Constitution but emphasized that any demonstration for that

2 o'clock in the afternoon, is a plea for the preservation merely of their property rights. The employees' pathetic situation was a stark reality abused, harassment and persecuted as they believed they were by the peace officers of the municipality. As above intimated, the condition in which the employees found themselves vis-a-vis the local police of Pasig, was a matter that vitally affected their right to individual existence as well as that of their families. Material loss can be repaired or adequately compensated. The debasement of the human being broken in morale and brutalized in spirit-can never be fully evaluated in monetary terms. As heretofore stated, the primacy of human rights freedom of expression, of peaceful assembly and of petition for redress of grievances over property rights has been sustained. To regard the demonstration against police officers, not against the employer, as evidence of bad faith in collective bargaining and hence a violation of the collective bargaining agreement and a cause for the dismissal from employment of the demonstrating employees, stretches unduly the compass of the collective bargaining agreement, is "a potent means of inhibiting speech" and therefore inflicts a moral as well as mortal wound on the constitutional guarantees of free expression, of peaceful assembly and of petition. Circulation is one of the aspects of freedom of expression. If demonstrators are reduced by one-third, then by that much the circulation of the Issue raised by the demonstration is diminished. The more the participants, the more persons can be apprised of the purpose of the rally. Moreover, the absence of one-third of their members will be regarded as a substantial indication of disunity in their ranks which will enervate their position and abet continued alleged police persecution.

In Re Edillion Facts: This is an administrative case against Edillon who refuses to pay his IBP membership dues assailing the provisions of the Rule of Court 139-A and the provisions of par. 2, Section 24, Article III, of the IBP By-Laws pertaining to the organization of IBP, payment of membership fee and suspension for failure to pay the same. He contends that the stated provisions constitute an invasion of his constitutional rights of being compelled to be a member of the IBP in order to practice his profession and thus deprives his rights to liberty and property and thereby null and void. Issue: Whether or not it assailed provisions constitutes a deprivation of liberty and property of the respondent. Held: The court held that the IBP is a Stateorganized Bar as distinguished from bar associationsthat are organized by individual lawyers themselves, membership of which is voluntary. The IBP however is an official national body of which all lawyers must be a member and are subjected to the rules prescribed for the governance of the Bar which includes payment of reasonable annual fee for the purpose of carrying out its objectives and implementation of regulations in the practice of law. The provisions assailed does not infringe the constitutional rights of the respondent as it is a valid exercise of police power necessary to perpetuate its existence with regulatory measures to implement. The name of Edillon was stricken out from the rolls of attorney for being a delinquent member of the bar. Subido v Ozaeta FACTS: Petitioner was the editor of the Manila Post, who sought the inspection of real estates sold to aliens and registered with the RD. He was denied to do so which prompted him to file a petition for mandamus. HELD: Except when it is clear that the purpose of the inspection is unlawful, it is not the duty of the registration officers to concern themselves with the motives, purposes, and objects of the person seeking to inspect the records. It is not their prerogative to see that the information which the records contain is not flaunted before the public gaze.

Baldoza v Dimaano In a verified letter-complaint dated September 9, 1975, the Municipal Secretary of Taal, Batangas, charges Municipal Judge Rodolfo B. Dimaano, of the same municipality, with abuse of authority in refusing to allow employees of the Municipal Mayor to examine the criminal docket records of the Municipal Court to secure data in connection with their contemplated report on the peace and order conditions of the said municipality. Respondent, in answer to the complaint, stated that there has never been an intention to refuse access to official court records; that although court records are among public documents open to inspection not only by the parties directly involved but also by other persons who have legitimate interest to such inspection, yet the same is always subject to reasonable regulation as to who, when, where and how they may be inspected. He further asserted that a court has unquestionably the power to prevent an improper use or inspection of its records and the furnishing of copies therefrom may be refused where the person requesting is not motivated by a serious and legitimate interest but acts out of whim or fancy or mere curiosity or to gratify private spite or to promote public scandal. In his answer, the respondent significantly observed: Restrictions are imposed by the Court for fear of an abuse in the exercise of the right. For fear that the dirty hands of partisan politics might again be at play, Some of the cases filed and decided by the Court after the declaration of Martial Law and years after the election still bore the stigma of partisan politics as shown in the affidavits and testimonies of witnesses. Without casting aspersion on any particular individual, it is worth mentioning, that the padlocks of the door of the Court has recently been tampered by inserting papers and matchsticks. Under the circumstances, to allow an indiscriminate and unlimited exercise of the right to free access, might do more harm than good to the citizenry of Taal.

Disorder and chaos might result defeating the very essence of their request. The undersigned is just as interested as Mr. Baldoza in the welfare of the community and the preservation of our democratic principles. Be that as it may, a request of this magnitude cannot be immediately granted without adequate deliberation and upon advisement, especially so in this case where the undersigned doubts the propriety of such request. Hence, it is believed that authority should first be secured from the Supreme Court, through the Executive Judge, for the formulation of guidelines and policies on this matter. The case was thereupon referred to Judge Francisco Mat. Riodique for investigation and report. At the preliminary hearing on October 16, 1975, Taal Mayor Corazon A. Caniza filed a motion to dismiss the complaint to preserve harmony and (cooperation among officers in the same municipality. This motion was denied by the Investigating Judge, but after formal investigation, he recommended the exoneration of respondent. Pertinent portion of his report reads as follows: * * * When this case was heard, complainant Dominador Baldoza informed the Court that he is aware of the motion to dismiss filed by Mayor Corazon A. Caniza and that he is in conformity with the dismissal of the administrative charge against Judge Rodolfo Dimaano. The Court asked him if he could prove his case and he said he can. So, the Court denied his oral motion to dismiss and required him to present his evidence. Complainant only manifested to the Court that he has no oral evidence. The only evidence he has are the exchanged communication which were all in writing and attached to the record between him and the respondent. The Court asked the respondent what he has to say on the documentary evidence of the complainant. He manifested that all his answers to the complaint are all embodied in his answers filed with the Court. A careful perusal, scrutiny, and study of the communications between the complainant and the

respondent, together with the answers filed by the latter, reveal that there is no showing of abuse of authority on the part of the respondent. The respondent allowed the complainant to open and view the docket books of the respondent under certain conditions and under his control and supervision. Complainant admitted that he was aware of the rules and conditions imposed by the respondent when he went to his office to view his docket books for the purpose mentioned in his communication. He also agreed that he is amenable to such rules and conditions which the respondent may impose. Under these conditions, therefore, the Court finds that the respondent has not committed any abuse of authority. The complainant was warned to be more cautious in filing any administrative charge against any public official especially, members of the judiciary, considering that an administrative charge against a member of the judiciary may expose the latter to public ridicule and scandal thereby minimizing if not eradicating public trust and After a careful evaluation of the recommendation, We find that the respondent did not act arbitrarily in the premises. As found by the Investigating Judge, the respondent allowed the complainant to open and view the docket books of respondent certain conditions and under his control and supervision. it has not been shown that the rules and conditions imposed by the respondent were unreasonable. The access to public records predicated on the right of the people to acquire information on matters of public concern. Undoubtedly in a democracy, the public has a legitimate interest in matters of social and political significance. In an earlier case, 1 this Court held that mandamus would lie to compel the Secretary of Justice and the Register of Deeds to examine the records of the latter office. Predicating the right to examine the records on statutory provisions, and to a certain degree by general principles of democratic institutions, this Court stated that while the Register of Deeds has discretion to exercise as to the manner in which persons desiring to inspect, examine or copy the records in his office

may exercise their rights, such power does not carry with it authority to prohibit. Citing with approval People ex rel. TitleGuarantee & T. Co. vs. Railly, 2 this Court said: The subject is necessarily committed, to a great degree, 'to his (register of deeds') discretion as to how much of the conveniences of the office are required to be preserved for the accomodation of these persons. It is not his duty to permit the office to be thronged needlessly with persons examining its books of papers, but it is his duty to regulate, govern, and control his office in such a manner as to permit the statutory advantages to be enjoyed by other persons not employed by him as largely and extensibly as that consistently can be done * * *. What the law expects and requires from him is the exercise of an unbiased and impartial judgment, by which all persons resorting to the office, under legal authority, and conducting themselves in an orderly manner, shall be secured their lawful rights and privileges, and that a corporation formed in the manner in which the relator has been, shall be permitted to obtain all the information either by searches, abstracts, or copies, that the law has entitled it to obtain. Except, perhaps, when it is clear that the purpose of the examination is unlawful, or sheer, Idle curiosity, we do not believe it is the duty under the law of registration officers to concern themselves with the motives, reasons, and objects of the person seeking access to the records. It is not their prerogative to see that the information which the records contain is not flaunted before public gaze, or that scandal is not made of it. If it be wrong to publish the contents of the records, it is the legislature and not the officials having custody thereof which is called upon to devise a remedy. As to the moral or material injury which the publication might inflict on other parties, that is the publisher's responsibility and lookout. The publication is made subject to the consequences of the law. The concurring opinion of Justice Briones predicated such right not on statutory grounds merely but on the constitutional right of the press to have access to

information as the essence of press freedom. 3 The New Constitution now expressly recognizes that the people are entitled to information on matters of public concern and thus are expressly granted access to official records, as well as documents of official acts, or transactions, or decisions, subject to such limitations imposed by law. 4 The incorporation of this right in the Constitution is a recognition of the fundamental role of free exchange of information in a democracy. There can be no realistic perception by the public of the nation's problems, nor a meaningful democratic decision making if they are denied access to information of general interest. Information is needed to enable the members of society to cope with the exigencies of the times. As has been aptly observed: "Maintaining the flow of such information depends on protection for both its acquisition and its dissemination since, if either process is interrupted, the flow inevitably ceases. " 5 However, restrictions on access to certain records may be imposed by law. Thus, access restrictions imposed to control civil insurrection have been permitted upon a showing of immediate and impending danger that renders ordinary means of control inadequate to maintain order. 6 WHEREFORE, the case against respondent is hereby dismissed.

Legaspi v CSC Facts: The respondent CSC had denied petitioner Valentin Legaspis request for information on the civil service eligibilities of Julian Sibonghanoy and Mariano Agas who were employed as sanitarians in the Health Department of Cebu City. Sibonghanoy and Agas had allegedly represented themselves as civil service eligibles who passed the civil service examinations for sanitarians. Claiming that his right to be informed of the eligibilities of Sibonghanoy and Agas is guaranteed by the Constitution, and that he has no other plain, speedy and adequate remedy to acquire the information, petitioner prays for the issuance of the extraordinary writ of mandamus to compel the respondent CSC to disclose said information. The respondent CSC takes issue on the personality of the petitioner to bring the suit. It is asserted that the petition is bereft of any allegation of Legaspis actual interest in the civil service eligibilities of Sibonghanoy and Agas. Issue: Whether or not the petitioner has legal standing to bring the suit Held: The petitioner has firmly anchored his case upon the right of the people to information on matters of public concern, which, by its very nature, is a public right. It has been held in the case of Tanada vs. Tuvera, 136 SCRA 27, that when the question is one of public right and the object of the mandamus is to procure the enforcement of a public duty, the people are regarded as the real party in interest, and the person at whose instigation the proceedings are instituted need not show that he has any legal or special interest in the result, it being sufficient to show that he is a citizen and as such interested in the execution of the laws.
It becomes apparent that when a mandamus proceeding involves the assertion of a public right, the requirement of personal interest is satisfied by the mere fact that the petitioner is a citizen, and therefore, part of the general public which possesses the right. The petitioner, being a citizen who as such, is clothed with personality to seek redress for the alleged obstruction of the exercise of the public right.

Valmonte vs. Belmonte Facts: Ricardo Valmonte wrote Feliciano Belmonte Jr. on 4 June 1986, requesting to be "furnished with the list of names of the opposition members of (the) Batasang Pambansa who were able to secure a clean loan of P2 million each on guaranty (sic) of Mrs. Imelda Marcos" and also to "be furnished with the certified true copies of the documents evidencing their loan. Expenses in connection herewith shall be borne by" Valmonte, et. al. Due to serious legal implications, President & General Manager Feliciano Belmonte, Jr. referred the letter to the Deputy General Counsel of the GSIS, Meynardo A. Tiro. Tiro replied that it is his opinion "that a confidential relationship exists between the GSIS and all those who borrow from it, whoever they may be; that the GSIS has a duty to its customers to preserve this confidentiality; and that it would not be proper for the GSIS to breach this confidentiality unless so ordered by the courts." On 20 June 1986, apparently not having yet received the reply of the Government Service and Insurance System (GSIS) Deputy General Counsel, Valmonte wrote Belmonte another letter, saying that for failure to receive a reply "(W)e are now considering ourselves free to do whatever action necessary within the premises to pursue our desired objective in pursuance of public interest." On 26 June 1986, Ricardo Valmonte, Oswaldo Carbonell, Doy Del Castillo, Rolando Bartolome, Leo Obligar, Jun Gutierrez, Reynaldo Bagatsing, Jun "Ninoy" Alba, Percy Lapid, Rommel Corro, and Rolando Fadul filed a special civil action for mandamus with preliminary injunction invoke their right to information and pray that Belmonte be directed: (a) to furnish Valmonte, et. al. the list of the names of the Batasang Pambansa members belonging to the UNIDO and PDP-Laban who were able to secure clean loans immediately before the February 7 election thru the intercession/marginal note of the then First Lady Imelda Marcos; and/or (b) to furnish petitioners with certified true copies of the documents evidencing their respective loans; and/or (c) to allow petitioners access to the public records for the subject information. Issue: Whether Valmonte, et. al. are entitled as citizens and taxpayers to inquire upon GSIS records on behest loans given by the former First Lady Imelda Marcos to

Batasang Pambansa members belonging to the UNIDO and PDP-Laban political parties. Held: The GSIS is a trustee of contributions from the government and its employees and the administrator of various insurance programs for the benefit of the latter. Undeniably, its funds assume a public character. More particularly, Secs. 5(b) and 46 of PD 1146, as amended (the Revised Government Service Insurance Act of 1977), provide for annual appropriations to pay the contributions, premiums, interest and other amounts payable to GSIS by the government, as employer, as well as the obligations which the Republic of the Philippines assumes or guarantees to pay. Considering the nature of its funds, the GSIS
is expected to manage its resources with utmost prudence and in strict compliance with the pertinent laws or rules and regulations. Thus, one of the reasons that prompted the revision of the old GSIS law (CA 186, as amended) was the necessity "to preserve at all times the actuarial solvency of the funds

administered by the Systems [Second Whereas Clause, PD 1146.] Consequently, as Feliciano Belmonte himself admits, the GSIS "is not supposed to grant 'clean loans.'" It is therefore the legitimate concern of the public to ensure that these funds are managed properly with the end in view of maximizing the benefits that accrue to the insured government employees. Moreover, the supposed borrowers were Members of the defunct Batasang Pambansa who themselves appropriated funds for the GSIS and were therefore expected to be the first to see to it that the GSIS performed its tasks with the greatest degree of fidelity and that all its transactions were above board. In sum, the public nature of the loanable funds of the GSIS and the public office held by the alleged borrowers make the information sought clearly a matter of public interest and concern. Still, Belmonte
maintains that a confidential relationship exists between the GSIS and its borrowers. It is argued that a policy of confidentiality restricts the indiscriminate dissemination of information. Yet, Belmonte has failed to cite any law granting the GSIS the privilege of confidentiality as regards the documents subject of the present petition. His position is apparently based merely on considerations of policy. The judiciary does not settle policy issues. The Court can only declare what the law is, and not what the law should be. Under our system of government, policy issues are within the domain of the political branches of the government, and of the people themselves as the repository of all State power.

BADOY vs. COMELEC (35 SCRA 285) Campaign, A. Lawful / Prohibited Election Propaganda Facts: Anacleto D. Badoy, Jr. avers that he is a candidate for delegate to theConstitutional Convention for the lone district of North Cotabato. He prays thatSection 12(F) of RA 6132 be declared unconstitutional as the same deniesindividuals, who are not candidates, their freedom of speech and of the press; andcandidates the right to speak and write, discuss and debate in favor of theircandidacies or against the candidacies of others. Section 12 (F) provides that theComelec shall endeavor to obtain free space from newspapers, magazines andperiodicals which shall be known as Comelec space, and shall allocate this spaceequally and impartially among all candidates within the areas in which thenewspapers are circulated. Outside of said Comelec space, it shall be unlawful toprint or publish, or cause to be printed or published, any advertisement, paidcomment or paid article in furtherance of or in opposition to the candidacy of anyperson for delegate, or mentioning the name of any candidate and the fact of hiscandidacy, unless all the names of all other candidates in the district in which thecandidate is running are also mentioned with equal prominence. ComelecResolution RR-724, as amended, merely restates the ban in Section 12 (F). Issue: Whether the ban in Section 12 (F) is valid or constitutional. Held: Under Section 12 (F), the moneyed candidate or individual who can afford topay for advertisements, comments or articles in favor of his candidacy or against thecandidacy of another or which mention his name and the fact of his candidacy, isrequired to mention all the other candidates in the same district with equalprominence, to exempt him from the penal sanction of the law. The evident purposeof the limitation is to give the poor candidates a fighting chance in the election. Therestriction is only one of the measures devised by the law to preserve suffrage pureand undefiled and to achieve the desired equality of chances among all thecandidates. Considering the foregoing limitation in Section 12(F) in the light of theother provisions of RA 6132 designed to maximize, if not approximate, equality ofchances among the various candidates in the same district, the said restriction on

thefreedom of expression appears too insignificant to create any appreciable dent on theindividuals liberty of expression. It should be noted that Section 8(a) of the same law,prohibiting political parties from aiding candidates and thus was more restrictive thanSection 12(F), was previously upheld to be valid. The limitation in Section 12(F) is areasoned and reasonable judgment on the part of Congress. It is not unconstitutional. Tolentino v Sec of Finance Facts: The value-added tax (VAT) is levied on the sale, barter or exchange of goods and properties as well as on the sale or exchange of services. RA 7716 seeks to widen the tax base of the existing VAT system and enhance its administration by amending the National Internal Revenue Code. There are various suits challenging the constitutionality of RA 7716 on various grounds. One contention is that RA 7716 did not originate exclusively in theHouse of Representatives as required by Art. VI, Sec. 24 of the Constitution, because it is in fact the result of the consolidation of 2 distinct bills, H. No. 11197 and S. No. 1630. There is also a contention that S. No. 1630 did not pass 3 readings as required by the Constitution. Issue: Whether or not RA 7716 violates Art. VI, Secs. 24 and 26(2) of the Constitution Held: The argument that RA 7716 did not originate exclusively in theHouse of Representatives as required by Art. VI, Sec. 24 of the Constitution will not bear analysis. To begin with, it is not the law but the revenue bill which is required by the Constitution to originate exclusively in the House of Representatives. To insist that a revenue statute and not only the bill which initiated the legislative processculminating in the enactment of the law must substantially be the same as the House bill would be to deny the Senates power not only to concur with amendments but also to propose amendments. Indeed, what the Constitution simply means is that the initiative for filing revenue, tariff or tax bills, bills authorizing an increase of the public debt, private bills and bills of local application must come from theHouse of Representatives on the theory that, elected

as they are from the districts, the members of the House can be expected to be more sensitive to the local needs and problems. Nor does the Constitution prohibit the filing in the Senate of a substitute bill in anticipation of its receipt of the bill from the House, so long as action by the Senate as a body is withheld pending receipt of the House bill. The next argument of the petitioners was that S. No. 1630 did not pass 3 readings on separate days as required by the Constitution because the second and third readings were done on the same day. But this was because the President had certified S. No. 1630 as urgent. The presidential certification dispensed with the requirement not only of printing but also that of reading the bill on separate days. That upon the certification of a bill by the President the requirement of 3 readings on separate days and of printing and distribution can be dispensed with is supported by the weight of legislative practice.

********RIGHTS OF THE ACCUSED Fifth Amendment of the United States Constitution No person shall be held to answer for a capital, or otherwise infamous crime, unless on a presentment or indictment of a Grand Jury, except in cases arising in the land or naval forces, or in the Militia, when in actual service in time of War or public danger; nor shall any person be subject for the same offence to be twice put in jeopardy of life or limb; nor shall be compelled in any criminal case to be a witness against himself, nor be deprived of life, liberty, or property, without due process of law; nor shall private property be taken for public use, without just compensation. Miranda v. Arizona Facts: The Supreme Courts decision in Miranda v. Arizona addressed four different cases involving custodial interrogations. In each of these cases, the defendant was questioned by police officers, detectives, or a prosecuting attorney in a room in which he was cut off from the outside world. In none of these cases was the defendant given a full and effective warning of his rights at the outset of the interrogation process. In all the cases, the questioning elicited oral admissions and, in three of them, signed statements that were admitted at trial.

robbery ad was place under formal arrest. He was then taken to the 70 th Precinct for detention, where he was questioned by an assistant district attorney in the presence of a hearing reporter who transcribed the questions and answers. At trial, the oral confession and the transcript were presented to the jury. Vignera was found guilty of first degree robbery and sentenced to 30-60 years imprisonment. The conviction was affirmed without opinion by the Appellate Division and the Court of Appeals.

murder and sentenced to death. The Supreme Court of California reversed, holding that Stewart should have been advised of his right to remain silent and his right to counsel. Issues: Whether statements obtained from an individual who is subjected to custodial police interrogation are admissible against him in a criminal trial and whether procedures which assure that the individual is accorded his privilege under the Fifth Amendment to the Constitution not to be compelled to incriminate himself are necessary. Supreme Court holding: The Court held that there can be no doubt that the Fifth Amendment privilege is available outside of criminal court proceedings and serves to protect persons in all settings in which their freedom of action is curtailed in any significant way from being compelled to incriminate themselves. As such, the prosecution may not use statements, whether exculpatory or inculpatory, stemming from custodial interrogation of the defendant unless it demonstrates the use of procedural safeguards effective to secure the privilege against self-incrimination. By custodial interrogation, we mean questioning initiated by law enforcement officers after a person has been taken into custody or otherwise deprived of his freedom of action in any significant way. The Court further held that without proper safeguards the process of in-custody interrogation of persons suspected or accused of crime contains inherently compelling pressures which work to undermine the individuals will to resist and to compel him to speak where he would otherwise do so freely. Therefore, a defendant must be warned prior to any questioning that he has the right to remain silent, that anything he says can be used against him in a court of law, that he has the right to the presence of an attorney, and that if he cannot afford an attorney one will be appointed for him prior to any questioning if he so desires. The Supreme Court reversed the judgment of the Supreme Court of Arizona in Miranda, reversed the judgment of the New York Court of Appeals in Vignera, reversed the judgment of the Court of Appeals for the

Ninth Circuit in Westover, and affirmed the judgment of the Supreme Court of California in Stewart. Argued: Feb. 28, March 1 and 2, 1966 Decided: June 13, 1966 Vote: 5-4 Majority opinion written by Chief Justice Warren and joined by Justices Black, Douglas, Brennan, Fortas Dissenting opinion written by Justice Harlan and joined by Justices Stewart and White Dissenting in part opinion written by Justice Clark. Follow-Up: Miranda v. Arizona: After Mirandas conviction was overturned by the Supreme Court, the State of Arizona retried him. At the second trial, Mirandas confession was not introduced into evidence. Miranda was once again convicted and sentenced to 2030 years in prison.

Miranda v. Arizona: Miranda was arrested at his home and taken in custody to a police station where he was identified by the complaining witness. He was then interrogated by two police officers for two hours, which resulted in a signed, written confession. At trial, the oral and written confessions were presented to the jury. Miranda was found guilty of kidnaping and rape and was sentenced to 20-30 years imprisonment on each count. On appeal, the Supreme Court of Arizona held that Mirandas constitutional rights were not violated in obtaining the confession. Vignera v. New York: Vignera was picked up by New York police in connection with the robbery of a dress shop that had occurred three days prior. He was first taken to the 17th Detective Squad headquarters. He was then taken to the 66th Detective Squad, where he orally admitted the

Westover v. United States: Westover was arrested by local police in Kansas City as a suspect in two Kansas City robberies and taken to a local police station. A report was also received from the FBI that Westover was wanted on a felony charge in California. Westover was interrogated the night of the arrest and the next morning by local police. Then, FBI agents continued the interrogation at the station. After 2 and one-half hours of interrogation by the FBI, Westover signed separate confessions, which had been prepared by one of the agents during the interrogation, to each of the two robberies in California. These statements were introduced at trial. Westover was convicted of the California robberies and sentenced to 15 years imprisonment on each count. The conviction was affirmed by the Court of Appeals for the Ninth Circuit. California v. Stewart: In the course of investigating a series of purse-snatch robberies in which one of the victims died of injuries inflicted by her assailant, Stewart was identified as the endorser of checks stolen in one of the robberies. Steward was arrested at his home. Police also arrested Stewarts wife and three other people who were visiting him. Stewart was placed in a cell, and, over the next five days, was interrogated on nine different occasions. During the ninth interrogation session, Stewart stated that he had robbed the deceased, but had not meant to hurt her. At that time, police released the four other people arrested with Stewart because there was no evidence to connect any of them with the crime. At trial, Stewarts statements were introduced. Stewart was convicted of robber and first degree

Republic Act No. 7438 1992

April 27,

AN ACT DEFINING CERTAIN RIGHTS OF PERSON ARRESTED, DETAINED OR UNDER CUSTODIAL INVESTIGATION AS WELL AS THE DUTIES OF THE ARRESTING, DETAINING AND INVESTIGATING OFFICERS, AND PROVIDING PENALTIES FOR VIOLATIONS THEREOF Be it enacted by the Senate and House of Representatives of the Philippines in Congress assembled:: Section 1. Statement of Policy. It is the policy of the Senate to value the dignity of every human being and guarantee full respect for human rights. Section 2. Rights of Persons Arrested, Detained or Under Custodial Investigation; Duties of Public Officers. (a) Any person arrested detained or under custodial investigation shall at all times be assisted by counsel. (b) Any public officer or employee, or anyone acting under his order or his place, who arrests, detains or investigates any person for the commission of an offense shall inform the latter, in a language known to and understood by him, of his rights to remain silent and to have competent and independent counsel, preferably of his own choice, who shall at all times be allowed to confer privately with the person arrested, detained or under custodial investigation. If such person cannot afford the services of his own counsel, he must be provided with a competent and independent counsel by the investigating officer.lawphi1 (c) The custodial investigation report shall be reduced to writing by the investigating officer, provided that before such report is signed, or thumbmarked if the person arrested or detained does not know how to read and write, it shall be read and adequately explained to him by his counsel or by the assisting counsel provided by the

investigating officer in the language or dialect known to such arrested or detained person, otherwise, such investigation report shall be null and void and of no effect whatsoever. (d) Any extrajudicial confession made by a person arrested, detained or under custodial investigation shall be in writing and signed by such person in the presence of his counsel or in the latter's absence, upon a valid waiver, and in the presence of any of the parents, elder brothers and sisters, his spouse, the municipal mayor, the municipal judge, district school supervisor, or priest or minister of the gospel as chosen by him; otherwise, such extrajudicial confession shall be inadmissible as evidence in any proceeding. (e) Any waiver by a person arrested or detained under the provisions of Article 125 of the Revised Penal Code, or under custodial investigation, shall be in writing and signed by such person in the presence of his counsel; otherwise the waiver shall be null and void and of no effect. (f) Any person arrested or detained or under custodial investigation shall be allowed visits by or conferences with any member of his immediate family, or any medical doctor or priest or religious minister chosen by him or by any member of his immediate family or by his counsel, or by any national nongovernmental organization duly accredited by the Commission on Human Rights of by any international non-governmental organization duly accredited by the Office of the President. The person's "immediate family" shall include his or her spouse, fianc or fiance, parent or child, brother or sister, grandparent or grandchild, uncle or aunt, nephew or niece, and guardian or ward. As used in this Act, "custodial investigation" shall include the practice of issuing an "invitation" to a person who is investigated in connection with an offense he is suspected to have committed, without prejudice to the liability of the "inviting" officer for any violation of law.

Section 3. Assisting Counsel. Assisting counsel is any lawyer, except those directly affected by the case, those charged with conducting preliminary investigation or those charged with the prosecution of crimes. The assisting counsel other than the government lawyers shall be entitled to the following fees; (a) The amount of One hundred fifty pesos (P150.00) if the suspected person is chargeable with light felonies;lawphi1alf (b) The amount of Two hundred fifty pesos (P250.00) if the suspected person is chargeable with less grave or grave felonies; (c) The amount of Three hundred fifty pesos (P350.00) if the suspected person is chargeable with a capital offense. The fee for the assisting counsel shall be paid by the city or municipality where the custodial investigation is conducted, provided that if the municipality of city cannot pay such fee, the province comprising such municipality or city shall pay the fee: Provided, That the Municipal or City Treasurer must certify that no funds are available to pay the fees of assisting counsel before the province pays said fees. In the absence of any lawyer, no custodial investigation shall be conducted and the suspected person can only be detained by the investigating officer in accordance with the provisions of Article 125 of the Revised Penal Code. Section 4. Penalty Clause. (a) Any arresting public officer or employee, or any investigating officer, who fails to inform any person arrested, detained or under custodial investigation of his right to remain silent and to have competent and independent counsel preferably of his own choice, shall suffer a fine of Six thousand pesos (P6,000.00) or a penalty of imprisonment of not less than eight (8) years but not more than ten (10) years, or both. The penalty of

perpetual absolute disqualification shall also be imposed upon the investigating officer who has been previously convicted of a similar offense. The same penalties shall be imposed upon a public officer or employee, or anyone acting upon orders of such investigating officer or in his place, who fails to provide a competent and independent counsel to a person arrested, detained or under custodial investigation for the commission of an offense if the latter cannot afford the services of his own counsel. (b) Any person who obstructs, prevents or prohibits any lawyer, any member of the immediate family of a person arrested, detained or under custodial investigation, or any medical doctor or priest or religious minister chosen by him or by any member of his immediate family or by his counsel, from visiting and conferring privately with him, or from examining and treating him, or from ministering to his spiritual needs, at any hour of the day or, in urgent cases, of the night shall suffer the penalty of imprisonment of not less than four (4) years nor more than six (6) years, and a fine of four thousand pesos (P4,000.00).lawphi1 The provisions of the above Section notwithstanding, any security officer with custodial responsibility over any detainee or prisoner may undertake such reasonable measures as may be necessary to secure his safety and prevent his escape. Section 5. Repealing Clause. Republic Act No. No. 857, as amended, is hereby repealed. Other laws, presidential decrees, executive orders or rules and regulations, or parts thereof inconsistent with the provisions of this Act are repealed or modified accordingly. Section 6. Effectivity. This Act shall take effect fifteen (15) days following its publication in the Official Gazette or in any daily newspapers of general circulation in the Philippines. Approved: April 27, 1992.lawphi1

People vs. Andan, G.R. No. 116437, March 3, 1997 Under these circumstances, it cannot be successfully claimed that appellant's confession before the mayor is inadmissible. It is true that a municipal mayor has "operational supervision and control" over the local police and may arguably be deemed a law enforcement officer for purposes of applying Section 12 (1) and (3) of Article III of the Constitution. However, appellant's confession to the mayor was not made in response to any interrogation by the latter. In fact, the mayor did not question appellant at all. No police authority ordered appellant to talk to the mayor. It was appellant himself who spontaneously, freely and voluntarily sought the mayor for a private meeting. The mayor did not know that appellant was going to confess his guilt to him. When appellant talked with the mayor as a confidant and not as a law enforcement officer, his uncounseled confession to him did not violate his constitutional rights. Thus, it has been held that the constitutional procedures on custodial investigation do not apply to a spontaneous statement, not elicited through questioning by the authorities, but given in an ordinary manner whereby appellant orally admitted having committed the crime. What the Constitution bars is the compulsory disclosure of incriminating facts or confessions. The rights under Section 12 are guaranteed to preclude the slightest use of coercion by the state as would lead the accused to admit something false, not to prevent him from freely and voluntarily telling the truth. Hence we hold that appellant's confession to the mayor was correctly admitted by the trial court. Appellant's confessions to the media were likewise properly admitted. The confessions were made in response to questions by news reporters, not by the police or any other investigating officer. We have held that statements spontaneously made by a suspect to news reporters on a televised interview are deemed voluntary and are admissible in evidence. PP v Marra In an information filed before the Regional Trial Court, Branch 43, Dagupan City, Samuel Marra y Zarate, John Doe, Peter Doe, Paul Doe and Tom Doe were charged with the crime of murder for the fatal shooting of one Nelson Tandoc on March 7,

1992. 1 On June 4, 1992, an amended information was filed wherein Allan Tan, alias "Allan Yao," was indicated as an accused instead of John Doe. 2 A warrant of arrest was thereafter issued against Allan Tan 3 but the same was returned unserved, 4 hence trial proceeded with regard to herein accused-appellant Samuel Marra alone. Duly assisted by counsel, appellant pleaded not guilty upon arraignment on May 15, 1992. 5 After trial on the merits, judgment was rendered by the court below on October 8, 1992 finding appellant guilty beyond reasonable doubt of the crime charged, attended by the aggravating circumstance of nighttime, and sentencing him to suffer the penalty of reclusion perpetua. He was further ordered to pay the heirs of Nelson Tandoc the sums of P50,000.00 as death indemnity, P50,000.00 as actual damages, P100,000.00 as moral damages, and the costs.6 The prosecution's eyewitness, Jimmy Din, positively identified appellant as the triggerman in the killing of Nelson Tandoc. Din recounted that at around 2:00 A.M. on March 7, 1992, he and his friend, Nelson Tandoc, were conversing with each other in front of Lucky Hotel located at M.H. del Pilar Street, Dagupan City, which was owned by the witness' father and of which he was the administrator. He noticed a man pass by on the opposite side of the street. The man made a dirty sign with his finger and Din informed Tandoc thereof. The man repeated his offensive act and called them by waving his hands. Infuriated, they followed the man until the latter stopped in front of the Dunkin' Donuts store at the corner of Arellano and Fernandez streets. They demanded an explanation from the man but they were not given any. 7 At that instant, two men arrived and one of them inquired what was going on. Tandoc informed him that they were just demanding an explanation from the man. Din was surprised when Tandoc unexpectedly slapped one of the two men. A brawl ensued, with Tandoc clashing with the two men while Din exchanged blows with the man who made the dirty finger sign. After the fisticuffs, their three opponents ran away in a westward direction. 8

Tandoc and Din then decided to walk back to the hotel. When they were about to enter the place, they noticed that the men with whom they just had a fight were running towards them. Sensing danger, they ran inside the annex building of the hotel and immediately secured the lock of the sliding outer door. They entered a room and waited until they felt that the situation had normalized. After ten to fifteen minutes, thinking that the men were no longer in the vicinity, they left the room. Having decided to go home, Tandoc opened the sliding door. All of a sudden, Din saw Appellant, who at that time was wearing a security guard's uniform, shoot Tandoc with a revolver. There was a fluorescent bulb installed at the front of the hotel which enabled Din to identify the assailant. Tandoc was shot in the middle of the chest and he fell down. Then, Din saw four to five men scamper away from the scene. 9 Aware of his injury, Tandoc told Din, "Tol, I was shot." The latter tried to chase appellant and his companions but he failed to catch up with them. Din and his wife then brought Tandoc to the Villaflor Hospital. The victim was taken to the emergency room but he expired an hour later. 10 At about 3:45 A.M. of March 7, 1992, SPO3 Reynaldo de Vera of the Dagupan City Police Station received a report about a shooting incident at the annex building of the Lucky Hotel. He proceeded to the crime scene along with SPO4 Orlando Garcia, SPO3 Mauricio Flores and SPO3 Noli de Castro. Upon their arrival about five minutes later, they were informed by the wife of Jimmy Din that the victim had been brought to the Villaflor Hospital. They proceeded to the hospital where Din informed them that he could recognize the man who killed Tandoc and that the killer was, at that time, wearing the polo shirt of a security guard's uniform. 11 They decided to proceed to an eatery called "Linda's Ihaw-Ihaw." Seeing the security guard of a nearby bus company, they inquired from him if he knew of any unusual incident that happened in the vicinity. The guard said that he saw the guard of "Linda's Ihaw-Ihaw," together with some companions, chasing two persons running towards M. H. del Pilar Street. He further added that the man was wearing a polo shirt of a security guard's uniform. Asked where

that particular guard might be, he pointed to a man eating inside the eatery nearby. The man eating was not in a security guard's uniform. 12 They approached the man and inquired whether he was the security guard of "Linda's Ihaw-Ihaw," which the latter answered in the affirmative. After a series of questions, they learned that he was Samuel Marra, that his tour of duty was from 7:00 P.M. of a preceding day to 6:00 A.M. the following day, that he was still on duty at around 2:30 in the morning of March 7, 1992, and that the firearm issued to him was in his house. Upon their request to see the firearm, they proceeded to Marra's residence at Interior Nueva Street. 13 When they arrived, Marra took a .38 caliber revolver from inside an aparador and handed it to De Vera. De Vera also found five live bullets and one spent shell. Smelling gunpowder from the barrel of the gun, De Vera asked Marra when he last fired the gun but the latter denied ever having done so. Abruptly, De Vera asked him pointblank why he shot Tandoc. Marra at first denied the accusation but when informed that someone saw him do it, he said that he did so in self-defense, firing at the victim only once. Tandoc allegedly had a samurai sword with him at the time of the incident. However, persistent efforts on the part of the policemen to thereafter locate said bladed weapon proved futile. Marra also admitted that prior to the incident, he chased the victim and Din. The officers then took Marra to the police station where he was detained. 14 Meanwhile, De Vera went to Villaflor Hospital from where he fetched Din and brought him to the police station. There, Din definitely identified Marra as the assailant. During the investigation, De Vera also found out that Marra had not firearm license. 15 Dr. Tomas G. Cornel, Assistant City Health Officer of Dagupan City, testified that he conducted an autopsy on a certain Nelson Tandoc. He found a gunshot wound on the victim with the point of entry of the left side of the anterior chest wall and the point of exit at the lower left portion of the right shoulder. 16

Prosecutor Gregorio Gaerlan, stepfather of the victim, testified on the funeral, burial and other expenses incurred by the family. He declared that they paid Funeraria Quiogue P25,000.00 for its services; Villaflor Hospital, P2,875.00 for the confinement of Tandoc; St. John Memorial Cathedral, P350.00; Eternal Garden, P3,000.00 for the interment fee and P150.00 for the rent of the tent during the burial; and that they spent P2,300.00 for the video tape expenses and P11,800.00 for food and drinks during the wake. 17 Understandably, appellant gave a different version of the incident. Marra declared in court that he used to work as a security guard at "Linda's Ihaw-Ihaw" from seven o'clock in the evening to six o'clock in the morning of the following day. On March 6, 1992, he reported for duty at seven o'clock that evening as was his usual practice. At around four o'clock down of the following day, he went home to change his clothes. He proceeded to the Five Star Bus Terminal which was adjacent to "Linda's Ihaw-Ihaw." He saw Neneng, the cashier of said eatery, and together they ordered arroz caldo. Later, at about 5:00 A.M., he was approached by four policemen who inquired if he was a security guard. He answered in the affirmative. He was also asked about his sidearm. When he answered that it was at his residence, they all went to his house to look for it. After he handed over the firearm to the policemen, he was brought to the city hall where he was detained. 18 Under cross-examination, he insisted that when he handed the gun to the policeman, there were five live bullets, and not four live bullets and one empty shell as claimed by the prosecution. Prior to the incident, he had never met Jimmy Din nor does he know of any cause why Din would harbor any ill feelings against him. 19 After a careful scrutiny of the records and an objective evaluation of the evidence, the Court is not disposed to reverse the judgment of the lower court, the decision of the latter being amply supported by the established facts and fully sustained by the applicable law. In assailing the decision of the court below, the defense argues that "Jimmy Din . . . was not able to identify the assailant in a definite and believable manner." It goes on to state

further that " Jimmy Din was inside the hotel when Nelson Tandoc was shot and his vision was o(b)structed by the door. Jimmy Din was also not familiar with the accused. Under the circumstances by which he allegedly witnessed the shooting, how could be identify clearly an assailant at the distance of 45 meters?" 20 Appellant's counsel is only partly correct, having conveniently failed to mention other vital parts of Din's testimony. An impartial review of said testimony readily reveals that Din was indeed in a position to know the identity of the assailant. Firstly, Din knew for a fact that the persons he and Tandoc fought with near the Dunkin' Donuts store were the same men who chased them while they were on their way back to the hotel because he was able to take a good look at them. During the chase, he naturally turned around to look at the men who were running after them and who were at that time in front of the Balingit Trading store which was well-lighted. 21 It logically follows that they were the same persons who were waiting for them when they later came out of the hotel, and he was familiar with their identities because of their previous encounter. Secondly, we do not agree with appellant that the door blocked the view of Din. Said door, partly made of plywood, had a spring hinge which makes it possible for the door to close by itself. However, at that time the spring hinge had been weakened by long and constant use such that it would take some time for it to close the door, thereby allowing Din sufficient opportunity to have an unobstructed view of the scene outside. 22 Thirdly, Din was quite near the victim and appellant, which proximity, enabled him to clearly see what really happened. He thus readily perceived the actual shooting at the time when Tandoc pushed the door open. At that precise moment, Din was at the left side of Tandoc and about four to five meters away from the assailant. 23 Lastly, the place was brightly illuminated by a 20-watt fluorescent bulb installed on the outside wall in front of the hotel. Marra was only about three meters away therefrom. Such physical conditions would undeniably afford a clear view from inside the hotel of

the immediate area outside and in front of the same where the incident took place. The prosecution presented another vital witness in the person of Sgt. Reynaldo de Vera, whose testimony we shall repeat here for easy reference. In capsulized form, De Vera narrated the sequence of events that happened after he and his companions went to the crime scene to conduct an investigation. Having received information that a man in a security guard's uniform was involved in the incident, they sought information from a security guard of a nearby bus terminal. Said security guard pointed them to Marra, who at that time was eating in acarinderia nearby. Informed by Marra that his gun was at his residence, they all went to Marra's residence to get the same. After receiving said firearm, De Vera asked appellant why he killed Tandoc but Marra initially denied any participation in the killing. Nevertheless, when confronted with the fact that somebody saw him do it, Marra admitted the act although he alleged it was done in self-defense. This testimony of De Vera as to the confession of Marra is of significant weight, but the admissibility thereof shall also be passed upon. Section 12(1), Article III of the 1987 Constitution provides that "(a)ny person under investigation for the commission of an offense shall have the right to be informed of his right to remain silent and to have competent and independent counsel preferably of his own choice. . . . ." The critical inquiry then is whether or not Marra was under custodial investigation when he admitted the killing but invoked selfdefense. We believe that he was not so situated. Custodial investigation involves any questioning initiated by law enforcement officers after a person has been taken into custody or otherwise deprived of his freedom of action in any significant way. It is only after the investigation ceases to be a general inquiry into an unsolved crime and begins to focus on a particular suspect,the suspect is taken into custody, and the police carries out a process of interrogations that lends itself to eliciting incriminating statements that the rule begins to operate. 24 In the case at bar, appellant was not under custodial investigation when he made the

admission. There was no coercion whatsoever to compel him to make such a statement. Indeed, he could have refused to answer questions from the very start when the policemen requested that they all go to his residence. The police inquiry had not yet reached a level wherein they considered him as a particular suspect. They were just probing into a number of possibilities, having been merely informed that the suspect was wearing what could be a security guard's uniform. As we held in People vs. Dy: 25 "What was told by the accused to Pat. Padilla was a spontaneous statement not elicited through questioning, but given in an ordinary manner. No written confession was sought to be presented in evidence as a result of formal custodial investigation. 26 The trial Court, therefore, cannot be held to have erred in holding that compliance with the constitutional procedure on custodial investigation is not applicable in the instant case, . . . ." Accordingly, the testimony of Sgt. de Vera assumes a dominant dimension because it totally destroys the defense of denial cum alibi subsequently raised by appellant. In his answers to Sgt. De Vera, appellant expressly admitted that he shot Tandoc, albeit with an exculpatory explanation. This admission of Marra is in complete contrast to the statements he later made in open court. In addition, the law provides that the declaration of an accused acknowledging his guilt of the offense charged, or of any offense necessarily included therein may be given in evidence against him and, in certain circumstances, this admission may be considered as part of the res gestae. In a similar situation involved in the aforecited case ofPeople vs. Dy, this Court held: . . . the oral confession made by the accused to Pat. Padilla that "he had shot a tourist" and that the gun he had used in shooting the victim was in his bar which he wanted surrendered to the Chief of Police (t.s.n., October 17, 1984, pp. 6-9) is competent evidence against him. The declaration of an accused acknowledging his guilt of the offense charged may be given in evidence against him (Sec. 29 [now Sec. 33], Rule 130). It may in a sense be also regarded as part of the res gestae. The rule is that, any person,

otherwise competent as a witness, who heard the confession, is competent to testify as to the substance of what he heard if he heard and understood all of it. An oral confession need not be repeated verbatim, but in such a case it must be given in substance. (23 C.J.S. 196, cited in People vs. Tawat, G.R. No. 62871, May 25, 1985, 129 SCRA 431). (Italics supplied.) In any event, even without his admission, the case against appellant has been duly established by the other evidence of the prosecution, as earlier discussed. However, persistently arguing for an acquittal, the defense points out that when the police officers saw Marra, he was not in a blue uniform whereas Din testified that the person who shot Tandoc was wearing the polo shirt of a security guard's uniform. This is a puerile argument since appellant himself removed any lingering doubts on this point. He said that on ending his tour of duty at 4:00 A.M. of March 7, 1992, he decided to go home to change clothes, after which he went to "Linda's Ihaw-Ihaw" to eat. This explains why, at the time the police officers saw him, he was already in civilian clothes. The shooting had taken place earlier at around 2:00 A.M. At that time, Marra was still in his security guard's uniform, being then on duty. However, while we agree that the crime committed by appellant was murder qualified by treachery, we reject the finding that the same was aggravated by nighttime. No evidence was presented by the prosecution to show that nocturnity was specially sought by appellant or taken advantage of by him to facilitate the commission of the crime or to ensure his immunity from capture. 27 At any rate, whether or not such aggravating circumstance should be appreciated, the penalty to be imposed on appellant would not be affected considering the proscription against the imposition of the death penalty at the time when the offense in the instant case was committed. WHEREFORE, the judgment of the court a quo finding accused-appellant Samuel Marra y Zarate guilty of the crime of murder and imposing upon him the penalty and civil liabilities therein stated is hereby AFFIRMED.

PP v Bolanos Facts: Oscar Pagdalian was murdered in Marble Supply, Balagtas Bulacan. According to Pat. Rolando Alcantara and Francisco Dayao, deceased was with two companions on the previous night, one of whom the accused who had a drinking spree with the deceased. When they apprehended the accused they found the firearm of the deceased on the chair where the accused was allegedly seated. They boarded accused along with Magtibay, other accused on the police vehicle and brought them to the police station. While in the vehicle Bolanos admitted that he killed the deceased. RTC convicted him hence the appeal. Issue: Whether or Not accused-appellant deprived of his constitutional right to counsel. Held: Yes. Being already under custodial investigation while on boardthe police patrol jeep on the way to the Police Station where formal investigation may have been conducted, appellant should have been informed of his Constitutional rights under Article III, Section 12 of the 1987 Constitution, more particularly par. 1 and par. 3. PP v Bandula Facts: Six armed men barged into the compound of Polo Coconut Plantation in Tanjay, Negros Oriental. The armed men were identified by Security Guard, including accused. Salva and Pastrano, security guards were hogtied and accused proceeded to the Atty. Garay, counsel of plantation. They ransacked the place and took with them money and other valuables. Atty. Garay was killed. Accused-appellant is charged with robbery with homicide along with 3 others who were acquitted for insufficiency of evidence. Appellant was convicted. Now, appellant argues that the extrajudicial confessions he and accused Dionanao executed suffer from constitutional infirmities, hence, inadmissible in evidence considering that they were extracted under duress and intimidation, and were merely countersigned later by the municipal attorney who, by the nature of his position, was not entirely an independent counsel nor counsel of their choice. Consequently,
without the extrajudicial confessions, the prosecution is left without sufficient evidence to convict him of the crime charged.

Issue: Whether or Not extrajudicial confessions of appellant is admissible as evidence against him. Held: No. When accused-appellant Bandula and accused Dionanao were investigated immediately after their arrest, they had no counselpresent. If at all, counsel came in only a day after the custodial investigation with respect to accused Dionanao, and two weeks later with respect to appellant Bandula. And, counsel who supposedly assisted both accused was Atty. Ruben Zerna, the Municipal Attorney of Tanjay. On top of this, there are telltale signs that violence was used against the accused. Certainly, these are blatant violations of the Constitution which mandates in Sec. 12, Art. III. Irregularities present include: 1. The investigators did not inform the accused of their right to remain silent and to have competent and independent counsel, preferably of their own choice, even before attempting to elicit statements that would incriminate them. 2. Investigators continuously disregard the repeated requests of the accused for medical assistance. Reason for Accused Sedigos "black eye" which even Pat. Baldejera admitted is not established, as well as Bandulasfractured rib. 3. Counsel must be independent. He cannot be a special counsel, public or private prosecutor, counsel of the police, or a municipal attorney whose interest is admittedly adverse to the accused. People v Judge Ayson Facts: Felipe Ramos was a ticket freight clerk of the Philippine Airlines, assigned at its Baguio City station. It was alleged that he was involved in irregularities in the sales of plane tickets, the PAL management notified him of an investigation to be

guilty. Evidence by the prosecution contained Ramos written admission and statement, to which defendants argued that the confession was taken without the accused being represented by a lawyer. Respondent Judge did not admit those stating that accused was not reminded of his constitutional rights to remain silent and to have counsel. A motion for reconsideration filed by the prosecutors was denied. Hence this appeal. Issue: Whether or Not the respondent Judge correct in making inadmissible as evidence the admission and statement of accused. Held: No. Section 20 of the 1987 constitution provides that the right against self-incrimination (only to witnesses other than accused, unless what is asked is relating to a different crime charged- not present in case at bar). This is accorded to every person who gives evidence, whether voluntarily or under compulsion of subpoena, in any civil, criminal, or administrative proceeding. The right is not to "be compelled to be a witness against himself. It prescribes an "option of refusal to answerincriminating questions and not a prohibition of inquiry." the right can be claimed only when the specific question, incriminatory in character, is actually put to the witness. It cannot be claimed at any other time. It does not give a witness the right to disregard a subpoena, to decline to appear before the court at the time appointed, or to refuse to testify altogether. It is a right that a witness knows or should know. He must claim it and could be waived. Rights in custodial interrogation as laid down
in miranda v. Arizona: the rights of the accused include: 1) he shall have the right to remain silent and to counsel, and to be informed of such right. 2) nor force, violence, threat, intimidation, or any other means which vitiates the free will shall be used against him. 3) any confession obtained in violation of these rights shall be inadmissible in evidence. The individual may knowingly and intelligently waive these rights and agree to answer or make a statement. But unless and until such rights and waivers are demonstrated by the prosecution at the trial, no evidence obtained as a result of interrogation can be used against him.

conducted. That investigation was scheduled in accordance with PAL's Code of Conductand Discipline, and the Collective Bargaining Agreement signed by it with the Philippine Airlines Employees' Association (PALEA) to which Ramos pertained. A letter

was sent by Ramos stating his willingness to settle the amount of P76,000. The findings of the Audit team were given to him, and he refuted that he misused proceeds of tickets also stating that he was prevented from settling said amounts. He proffered a compromise however this did not ensue. Two months after a crime of estafa was charged against Ramos. Ramos pleaded not

Gamboa v Cruz Facts: Petitioner was arrested for vagrancy without a warrant. During a line-up of 5 detainees including petitioner, he was identified by a complainant to be a companion in a robbery, thereafter he was charged. Petitioner filed a Motion to Acquit on the ground that the conduct of the lineup, without notice and in the absence of his counsel violated his constitutional rights to counsel and to due process. The court denied said motion. Hearing was set, hence the petition. Issue: Whether or Not petitioners right to counsel and due process violated. Held: No. The police line-up was not part of the custodial inquest, hence, petitioner was not yet entitled, at such stage, to counsel. He had not been held yet to answer for a criminal offense. The moment there is a move or even an urge of said investigators to elicit admissions or confessions or even plain information which may appear innocent or innocuous at the time, from said suspect, he should then and there be assisted by counsel, unless he waives the right, but the waiver shall be made in writing and in the presence of counsel. On the right to due process, petitioner was not, in any way, deprived of this substantive and constitutional right, as he was duly represented by a counsel. He was accorded all the opportunities to be heard and to present evidence to substantiate his defense; only that he chose not to, and instead opted to file a Motion to Acquit after the prosecution had rested its case. What due process abhors is the absolute lack of opportunity to be heard. PP v Caguioa The heinous and abominable crime of rape committed by an accused upon his own flesh and blood arouses our repugnance and indignation as no other crime can. Rodolfo Caguioa, Sr. was charged with two counts of rape against his own daughter in two separate Informations reading as follows: In Criminal Case No. L-4494: That on or about the first week of April 1991 in Barrio Bunagan, Mangatarem, Pangasinan and within the jurisdiction of this Honorable

Court, the said accused, by making Aurora Caguioa smell some sort of a chemical causing her to lose consciousness did then and there wilfully and feloniously lie and had carnal knowledge of said Aurora Caguioa while unconscious against her will and consent, to her damage and prejudice. In Criminal Case No. L-4495:

indemnify the offended party the sum of fifty thousand pesos (P50,000.00) as civil indemnity without subsidiary imprisonment in case of insolvency. The penalty imposed by the court against the accused should be served successively or one after the other. (p. 22, Rollo.)

That on or about the second week of April 1991 in Barrio Bunagan, Mangatarem, Pangasinan and within the jurisdiction of this Honorable Court, the said accused, by means of force, violence and intimidation, that is, by then and there threatening Aurora Caguioa with a knife, which he was holding at the time, should she not agree to submit herself to her criminal design, did, then and there wilfully, unlawfully and feloniously lie with and had carnal knowledge of said Aurora Caguioa against her will and consent, to her damage and prejudice. After trial, the court a quo found the accused guilty on both counts in a decision dated April 7, 1992, the dispositive of portion of which reads: In Criminal Case No. L-4494, the court finds and holds the accused Rodolfo Caguioa, Sr. guilty beyond reasonable doubt of the crime of Rape charged in the information filed against him, defined and penalized under the provisions of Article 335 of the Revised Penal Code as amended, and conformable thereto, hereby sentences the said accused to suffer the penalty of reclusion perpetua (life imprisonment) and its accessory penalties and to pay the costs of the proceedings. In Criminal Case No. L-4495, the court likewise finds and holds the accused Rodolfo Caguioa, Sr. guilty beyond reasonable doubt of the crime of Rape charged in the information filed against him, defined and penalized under the provisions of Article 335 of the Revised Penal Code and conformable thereto, hereby sentences said accused to suffer the penalty of reclusion perpetua(life imprisonment), its accessory penalties and to further pay the costs of the proceedings. The court further orders the accused to acknowledge and support the child, Babellin Caguioa, as his own spurious child and to

From said decision, the instant appeal has been interposed on the catch-all argument that no strong evidence exists which would link accused-appellant to the crime, his Sinumpaang Salaysay being inadmissible. The facts of the case, as borne out by the evidence, are concisely narrated by the appellees brief submitted by Solicitor General Raul L. Goco, Assistant Solicitor General Roman G. del Rosario, and Solicitor Esperanza Fabon-Victorino, to wit: Aurora Caguioa is a fifteen-year old barrio lass who used to work as domestic helper at the residence of Mrs. Virginia Organo in Las Pias, Metro Manila. Sometime on the last days of March 1991, she went home for a vacation in their house at Bonogon, Mangatarem, Pangasinan. She first stayed in the house of her cousin at the Poblacion (Nov. 20, 1991, pp. 4 & 18). Two days thereafter, she proceeded to the barrio where her father appellant Rodolfo Caguioa, brother Randy and sisters Arlene and Rodina live (Id. 22). On April 5, 1991, at around 10 in the evening, Aurora was alone inside her room when a man held her and placed a piece of cloth on her face which caused her to dozed of. When she opened her eyes the following morning, she felt pain all over her body. There was bloodstain on her underwear and her genitalia was swollen. She knew she was sexually abused but did not know the culprit. She weep in helpless protest but kept the ordeal from her sisters and brothers who slept in the other room (Id., pp. 5-6) as well as to appellant who slept on the papag about two meters away from her room. On April 10, 1991, Aurora was left in the house together with appellant. Arlene went to Camiling while Rodina and Randy went to

Poroc. It was midday, when appellant approached and asked Aurora to submit herself to him as he will repeat what he had previously done to her. He told her to agree otherwise he will not allow her to return to Manila. Aurora refused but appellant poked a knife on her neck and threatened to kill her if she will not give in to his beastly desire. Aurora trembled with fear as appellant undressed her. While holding Aurora, appellant removed his short pants and penetrated her (Id., pp. 6-8). After he had unleashed his lust on his own daughter, appellant left the crime site leaving Aurora, who, just like the first time, felt the pain all over her body. Meantime, Aurora found refuge in the house of his elder brother Jerry in Parian, Mangatarem, Pangasinan. After three days, she went back to her employers house in Las Pias and narrated to Mrs. Organo her ordeal. The latter accompanied Aurora to the Human Rights Commission where she executed a Sinumpaang Salaysay dated May 23, 1991 narrating the criminal incidents (Exh. A). Aurora was referred to the National Bureau of Investigation (NBI) where she was investigated by Agents Pedro L. Rivera and Ruel M. Lasala. She executed Sinumpaang Salaysay dated July 5, 1991 naming appellant as the culprit (Exh. B). Subsequently, appellant was arrested and brought to the NBI. In the presence of the authorities and press, Aurora identified appellant as the person who raped her twice in their house in Barrio Bunagan, Mangatarem, Pangasinan (Exh. E-E-2). On May 24, 1991, NBI Medico Legal Ruberto M. Sombilon examined Aurora and thereafter issued Medico Legal Certificate MG-91-573. The report disclosed: 1. No evident sign of extra-genital physical injuries acted on the body of the subject at the time of examination. 2. Hymen intact but distensible and its orifice wide (2.0 cm. in diameter as to allow complete penetration by an averaged size adult Filipino male organ in full erection without producing any genital injury).

3. Presumptive signs of pregnancy present, age of which may correspond to the middle part of the first trimester of gestation. (TSN, November 20, pp. 32-33) On July 17, 1991, an affidavit of complaint was filed with the fiscals office for proper action (Exh. C). As a result of the criminal act, Aurora became pregnant and gave birth to a baby girl she named Babellin Caguioa who has the physical features of appellant. (pp. 4-8, Appellees Brief; ff. p. 106, Rollo.) Accused-appellant puts forth the defense of alibi, asserting that on April 10, 1991, he was out the whole day plowing a ricefield about one kilometer away from their house and that he did not go home that day to take his lunch. We have consistently hewed to the legal precept that alibi is one of the weakest defenses that can be resorted to by an accused not only because it is inherently weak and unreliable but also because it is easy of fabrication and cannot prevail over the positive identification of the accused (People vs. Calope, 229 SCRA 413 [1994]). Accused-appellant was positively identified by his own daughter as the culprit who ravished her. It is inconceivable and contrary to human experience for a daughter, who is attached to her father by the natural bond of love and affection, to accuse him of rape, unless he is the one who raped and defoliated her. And the victims positive identification of accusedappellant as her rapist negates alibi (People vs. Escoto, 229 SCRA 430 [1994]). Moreover, alibi must be supported by credible corroboration from disinterested witnesses, and where the defense of alibi is not corroborated, it is fatal to the accused (People vs. Calope, supra). The record discloses not a shred of evidence to corroborate accused-appellants alibi. Further, for alibi to prosper, it must be shown that it was physically impossible for the accused to be present at the place where the crime was committed at the time of its commission (People vs. Apa-ap, Jr., 235 SCRA 468 [1994]). By his own testimony, he was in the ricefield only one kilometer away from his house on April 10, 1991. He could have easily traversed the one-kilometer distance from the rice field to

his house. Clearly, there was no physical impossibility for him to be present at the scene of the crime at the time of the commission thereof. In a further bid at exculpation, accused-appellant assails the admissibility of his Sinumpaang Salaysay (Exhibit G) in which he admitted having raped his daughter. There is no necessity to delve into the admissibility of the Sinumpaang Salaysay. Excluding from consideration the Sinumpaang Salaysay, there is more than ample evidence to establish the guilt of accused-appellant. The testimony of the victim, Aurora Caguioa, stating that her father, accused-appellant, raped her on April 10, 1991, is sufficient to convict accusedappellant. Her testimony is clear, positive, and candid. She narrated her ordeal at the hands of her own father in detail and in a straightforward manner. She recounted how her father raped her on April 10, 1991. Her father told her that he wanted to repeat what he had previously done to her and she had to yield to his demonic demand lest she would not be allowed to return to Manila. Aurora refused but she was seized with fear when accused-appellant poked a knife at her neck and threatened to kill her. She struggled against accusedappellant to ward off accused-appellant from consummating his bestial designs, but accused-appellant succeeded in satisfying his lust on his own daughter. The testimony of Aurora must be given full faith and credibility for there is nothing on record to show that she was actuated by ill motives in making the accusation of rape against her own father. Well-entrenched is the rule that the testimony of a rape victim is credible where she has no ill motive to testify against the accused (People vs. Matanorosa, 231 SCRA 509 [1994]). The fact that accused-appellant can not proffer any explanation as to why the complainant implicated him indicates that no improper motive impelled the complainant to accuse the former of such a serious offense (People vs. Sagaban, 231 SCRA 744 [1994]). Rape is committed by having carnal knowledge of a woman by using force or intimidation (Article 335, Revised Penal Code; People vs. Paliete, 229 SCRA 543 [1994]). Clearly, the facts of the case show that accused-appellant is guilty of having raped his own daughter on April 10, 1991. However, as to the alleged rape committed on April 5, 1991 (Criminal Case No. L-4494), it is our considered opinion

that there is no sufficient evidence to prove the same. The only piece of evidence adduced to prove it is the testimony of the victim that accused-appellant allegedly told her that he wanted to repeat what he had previously done to her. In fact, she was even candid to admit that she did not know who raped her the first time. We believe that her testimony that the accused told her that he wanted to repeat what he had previously done is merely a conclusion that she made, considering that her father raped her on April 10, 1991. WHEREFORE, the decision appealed from is hereby AFFIRMED in regard to accused-appellants conviction in Criminal Case No. L-4495, but REVERSED as to Criminal Case No. L-4494, in which accusedappellant is hereby acquitted. PP v Nicandro FACTS: The Drug Enforcement Unit of Western Police District received complaints from concerned citizens regarding the illegal sale of prohibited drugs by one alias Nel in Commodore Pension House in Ermita Manila. Responding to said complaints and reports, CorPoraL Guitan and Private First Class Joves of the Drug Enforcement Unit of said Police Station No. 5 placed the Commodore Pension House and its surroundings under surveillance for about a week. After the complaints and reports were verified to be true, an entrapment with the confidential informant acting as the buyer of marijuana was organized. At about 9:00 p.m. on November 1981, the police team formed to carry out the entrapment plan. When the team saw the accused handed the suspected marijuana cigarettes to the informant, and after the pre-arranged signal was given, the team immediately nabbed said suspect and at the same time identified themselves as police officers. Pat. Joves informed the accused of her constitutional rights and then he questioned her about the marijuana that were confiscated and she verbally admitted the sell of marijuana cigarettes and the possession and ownership of the other marijuana leaves which was confiscated from her possession. Subsequently, the accused refused to place her statement in writing.

ISSUE: WON the evidence obtained was a violation of the constitutional rights of the accused. RULING: Yes. When the Constitution requires a person under investigation "to be informed" of his right to remain silent and to counsel, it must be presumed to contemplate the transmission of meaningful information rather than just the ceremonial and perfunctory recitation of an abstract constitutional principle. As a rule, therefor, it would not be sufficient for a police officer just to repeat to the person under investigation the provisions of Section 20, Article IV of the Constitution. He is not only duty-bound to tell the person the rights to which the latter is entitled; he must also explain their effects in practical terms, e.g., what the person under interrogation may or may not do, and in a language the subject fairly understands. In other words, the right of a person under interrogation "to be informed" implies a correlative obligation on the part of the police investigator to explain, and contemplates an effective communication that results in understanding what is conveyed. Short of this, there is a denial of the right, as it cannot truly be said that the person has been "informed" of his rights. Now, since the right "to be informed" implies comprehension, the degree of explanation required will necessary vary, depending upon the education, intelligence and other relevant personal circumstances of the person under investigation. Suffice it to say that a simpler and more lucid explanation is needed where the subject is unlettered. According to Pat. Joves, he informed appellant of her constitutional rights when she was under custodial investigation. What specific rights he mentioned to appellant, he did not say. Neither did he state the manner in which the appellant was advised of her constitutional rights so as to make her understand them. This is particularly significant in the instant case because appellant is illiterate and cannot be expected to be able to grasp the significance of her right to silence and to counsel upon merely hearing an abstract statement thereof.

PP v Galit FACTS: Florentino Valentino, the husband of the daughter of the wife of the accused, and the only witness of the crime, testified that when he returned home at about 4:00am from the police station of Marikina, Rizal, the accused and his wife were quarreling; that he heard that the accused was leaving the house because he and his companions had robbed "Aling Nene", the owner of a poultry farm and piggery in Montalban, Rizal; that the wife of the accused was imploring him not to leave, but the latter was insistent; that he saw the accused carrying a bag containing about two handfuls of coins which he had taken from Aling Nene; that upon learning of what the accused had done, he went to the Montalban police the next day and reported to the police chief about what he had heard; and that a week later, Montalban policemen went to their house and arrested the accused. The accused had been detained and interrogated almost continuously for five days, to no avail. He consistently maintained his innocence. There was no evidence to link him to the crime. There were no eyewitnesses, no property recovered from the accused, no state witnesses, and not even fingerprints of the accused at the scene of the crime. So the investigating officers began to maul him and to torture him physically. Still the prisoner insisted on his innocence. They continued to maltreat and beat him. 'They covered his face with a rag and pushed his face into a toilet bowl full of human waste. He could not take any more. His body could no longer endure the pain inflicted on him and the indignities he had to suffer. He admitted what the investigating officers wanted him to admit and he signed the confession they prepared. Later, against his will, he posed for pictures as directed by his investigators, purporting it to be a reenactment. ISSUE: WON the alleged confession and the pictures of the supposed re-enactment are inadmissible as evidence. RULING: YES. When Accused was informed of his rights in Tagalog, "Oo" was his answer. Such a long question followed by a monosyllabic answer does not satisfy the requirements of the law that the accused be informed of his rights under the Constitution and our laws. Instead there should be several short and clear questions and every right explained in simple words in a dialect

or language known to the person under investigation. Accused is from Samar and there is no showing that he understands Tagalog. Moreover, at the time of his arrest, accused was not permitted to communicate with his lawyer, a relative, or a friend. In fact, his sisters and other relatives did not know that he had been brought to the NBI for investigation and it was only about two weeks after he had executed the salaysay that his relatives were allowed to visit him. His statement does not even contain any waiver of right to counsel and yet during the investigation he was not assisted by one. At the supposed reenactment, again accused was not assisted by counsel of his choice. These constitute gross violations of his rights. THE PEOPLE OF THE PHILIPPINES, vs. JUAN NABALUNA Y LLAMOS et. al. FACTS: In the decision dated September 26, 1981 rendered in Criminal Case No. CC-XIV-1805 by the former Circuit Criminal Court of Cebu City, the accused-appellants herein, Juan Nabaluna and Edgardo Empuerto, were found guilty beyond reasonable doubt of the crime of Robbery with Homicide and were sentenced to suffer the penalty of death. They were also ordered to indemnify the heirs of the offended party, Nazario Bendanillo, by way of actual and compensatory damages the sum of P15,000.00 as well as moral and exemplary damages in the amount of P25,000.00, and to cause the return of P200.00 to the aforesaid heirs; and to pay proportionate costs. The uniform and main thrust of appellants' principal argument is that, their extrajudicial statements were improperly admitted as evidence against them as these were obtained in violation of the Constitution, particularly because they were not then assisted by counsel when they were undergoing investigation. Appellants aver that while their extra-judicial statements carry a foreword that the accused were respectively advised of their constitutional rights, nevertheless, they were never informed that if they cannot afford a lawyer to assist them in the investigation the State will provide them with one, free of charge. ISSUE: Can the voluntariness test in the Galit doctrine be applied retroactively?

RULING: No. The Court is mindful of the structures and pronouncements found in the case of Morales vs. Ponce Enrile, G.R. Nos. 61106 and 61107, promulgated on April 26, 1983, 121 SCRA 538, which was quoted and reiterated in the case of People vs. Galit, L51770, March 20, 1985 particularly as to the requisite steps before a person under custodial investigation may be deemed to have properly waived his right to counsel such, as a counsel being present to assist him when the accused manifests such waiver. However, the stated requirements were laid down in the said cases, to serve as governing guidelines, only after the judgment in this case had already been rendered by the trial court. Consequently, no error should attach to the admission by the trial court of the extra-judicial statements given by the accused as evidence in this case. The trial court was then sufficiently convinced that the accused had waived assistance of counsel and there was at that time no pronounced guidelines requiring that the waiver of counsel by accused can be properly made only with the presence and assistance of a counsel. It may also be added that the facts of the present case can be differentiated from those stated in the cited case of People vs. Galit, where the acquittal therein was due to absence of any other evidence aside from the supposed confession of the accused. What stands out, however, is that, in both extrajudicial statements given by appellants Empuerto and Nabaluna it is therein duly acknowledged by herein appellants that they were duly informed of their constitutional rights to remain silent and to be assisted by counsel. Of added significance is the fact that when the second set of extra-judicial statements were later given by the accused Juan Nabaluna on before special Counsel Gabriel Trocio, Jr., the latter again informed him of his constitutional rights and even told said accused that he, could recommend said accused to the Citizens' Legal Assistance Office (CLAO) if the latter could not afford to hire the services of counsel. However, appellant Juan Nabaluna waived his right to be assisted by counsel.

People v Judge Donato Facts: Private respondent and his coaccused were charged of rebellion on October 2, 1986 for acts committed before and after February 1986. Private respondent filed with a Motion to Quash alleging that: (a) the facts alleged do not constitute an offense; (b) the Courthas no jurisdiction over the offense charged; (c) the Court has no jurisdiction over the persons of the defendants; and (d) the criminalaction or liability has been extinguished. This was denied. May 9, 1987 Respondent filed a petition for bail, which was opposed that the respondent is not entitled to bail anymore since rebellion became a capital offense under PD 1996, 942 and 1834 amending ART. 135 of RPC. On 5 June 1987 the President issued Executive Order No. 187 repealing, among others, P.D. Nos. 1996, 942 and 1834 and restoring to full force and effect Article 135 of the Revised Penal Code as it existed before the amendatory decrees. Judge Donato now granted the bail, which was fixed at P30,000.00 and imposed a condition that he shall report to the court once every two months within the first ten days of every period thereof. Petitioner filed a supplemental motion for reconsideration indirectly asking the court to deny bail to and to allow it to present evidence in support thereof considering the "inevitable probability that the accused will not comply with this main condition of his bail. It was contended that: 1. The accused has evaded the authorities for thirteen years and was an escapee from detention when arrested; (Chairman of CPPNPA) 2. He was not arrested at his residence as he had no known address; 3. He was using the false name "Manuel Mercado Castro" at the time of his arrest and presented a Driver's License to substantiate his false identity; 4. The address he gave "Panamitan, Kawit, Cavite," turned out to be also a false address; 5. He and his companions were on board a private vehicle with a declared owner whose identity and address were also found to be false; 6. Pursuant to Ministry Order No. 1-A dated 11 January 1982 , a reward of P250,000.00 was offered and paid for his arrest. This however was denied. Hence the appeal.

Issue: Whether or Not the private respondent has the right to bail. Held: Yes. Bail in the instant case is a matter of right. It is absolute since the crime is not a capital offense, therefore prosecution has no right to present evidence. It is only when it is a capital offense that the right becomes discretionary. However it was wrong for the Judge to change the amount of bail from 30K to 50K without hearing the prosecution. Republic Act No. 6968 approved on 24 October 1990, providing a penalty of reclusion perpetua to the crime of rebellion, is not applicable to the accused as it is not favorable to him. Accused validly waived his right to bail in another case(petition for habeas corpus). Agreements were made therein: accused to remain under custody, whereas his codetainees Josefina Cruz and Jose Milo Concepcion will be released immediately, with a condition that they will submit themselves in the jurisdiction of the court. Said petition for HC was dismissed. Bail is the security given for the release of a person in custody of the law. Ergo, there was a waiver. We hereby rule that the right to bail is another of the constitutional rights which can be waived. It is a right which is personal to the accused and whose waiver would not be contrary to law, public order, public policy, morals, or good customs, or prejudicial to a third person with a right recognized by law. Commendador v de Villa Facts: The petitioners in G.R. Nos. 93177 and 96948 who are officersof the AFP were directed to appear in person before the PreTrial Investigating Officers for the alleged participation the failed coup on December 1 to 9, 1989. Petitioners now claim that there was no pre-trial investigation of the charges as mandated by Article of War 71. A motion for dismissal was denied. Now, their motion for reconsideration. Alleging denial of due process. In G.R. No. 95020, Ltc Jacinto Ligot applied for bail on June 5, 1990, but the application was denied by GCM No.14. He filed with the RTC a petition for certiorari and mandamus with prayer for provisional liberty and a writ of preliminary injunction.

Judge of GCM then granted the provisional liberty. However he was not released immediately. The RTC now declared that even military men facing court martial proceedings can avail the right to bail. The private respondents in G.R. No. 97454 filed with SC a petition forhabeas corpus on the ground that they were being detained in Camp Crame without charges. The petition was referred to RTC. Finding after hearing that no formal charges had been filed against the petitioners after more than a year after their arrest, the trial court ordered their release. Issues: (1) Whether or Not there was a denial of due process. (2) Whether or not there was a violation of the accused right to bail. Held: NO denial of due process. Petitioners were given severalopportunities to present their side at the pre-trial investigation, first at the scheduled hearing of February 12, 1990, and then again after the denial of their motion of February 21, 1990, when they were given until March 7, 1990, to submit their counter-affidavits. On that date, they filed instead a verbal motion for reconsideration which they were again asked to submit in writing. They had been expressly warned in the subpoena that "failure to submit counter-affidavits on the date specified shall be deemed a waiver of their right to submit controverting evidence." Petitioners have a right to preemptory challenge. (Right to challenge validity of members of G/SCM) It is argued that since the private respondents are officers of the Armed Forces accused of violations of the Articles of War, the respondentcourts have no authority to order their release and otherwise interfere with the court-martial proceedings. This is without merit. * The Regional Trial Court has concurrent jurisdiction with the Court of Appeals and the Supreme Court over petitions for certiorari, prohibition or mandamus against inferior courts and other bodies and on petitions for habeas corpus and quo warranto.

The right to bail invoked by the private respondents has traditionally not been recognized and is not available in the military, as an exception to the general rule embodied in the Bill of Rights. The right to a speedy trial is given more emphasis in the military where the right to bail does not exist. On the contention that they had not been charged after more than one year from their arrest, there was substantial compliance with the requirements of due process and the right to a speedy trial. The AFP Special Investigating Committee was able to complete the pre-charge investigation only after one year because hundreds of officers and thousands of enlisted men were involved in the failed coup. Accordingly, in G.R. No. 93177, the petition is dismissed for lack of merit. In G.R. No. 96948, the petition is granted, and the respondents are directed to allow the petitioners to exercise the right of peremptory challenge under article 18 of the articles of war. In G.R. Nos. 95020 and 97454, the petitions are also granted, and the orders of the respondent courts for the release of the private respondents are hereby reversed and set aside. No costs.

On September 15, 1964, on petitioner's motion that the original bond previously given be reinstated, respondent judge resolved to fix "the bond anew in real property in the amount of P60,000.00, but to be posted only by residents of the province of Marinduque actually staying therein" with properties which "must be in the possession and ownership of said residents for five years." ISSUE: Whether or not he bail bond is constitutional; RULING: Yes RATIO: The following are certain guidelines in bail fixing: (1) ability of the accused to give bail; (2) nature of the offense; (3) Penalty for the offense charged; (4) character and reputation of the accused; (5) health of the accused; (6) character and strength of the evidence; (7) probability of the accused appearing in trial; (8) forfeiture of other bonds; (9) whether the accused was a fugitive from justice when arrested; and (10) if the accused is under bond for appearance at trial in other cases. Here petitioner is charged with a capital offense, direct assault upon an agent of a person in authority with murder. A complex crime, it may call for the imposition of the capital punishment. Then, Circular 47 dated July 5, 1946 of the Department of Justice, reiterated in Circular 48 of July 18, 1963, directed prosecuting attorney's to recommend bail at the rate of P2,000.00 per year of imprisonment, corresponding to the medium period of the penalty prescribed for the offense charged, unless circumstances warrant a higher penalty. The reasonableness of this circular has already received this Court's imprimatur in one case.16 We are unprepared to downgrade this method of computation, what with a compound of reduced peso value and the aggravated crime climate.

REYNALDO C. VILLASEOR, petitioner, vs. HON. MAXIMO ABANO, Judge of the Court of First Instance of Marinduque and THE PROVINCIAL FISCAL OF MARINDUQUE, respondents. SANCHEZ, J.: FACTS: Petitioner, defendant below, was, on motion, admitted to a P60,000.00-bail. The amount of the bond was, on verbal representation of petitioner's wife, reduced to P40,000.00. On May 29, 1964, petitioner posted a property bond, was set at provisional liberty. Before arraignment on the murder charge, however, respondent Provincial Fiscal amended the information. This time he accused petitioner with "Direct Assault Upon an Agent of a Person in Authority with Murder."

People v Holgado
Facts: Appellant Frisco Holgado was charged in the court of First Instance of Romblon with slight illegal detention because according to the information, being a private person, he did "feloniously and without justifiable motive, kidnap and detain one Artemia Fabreag in the house of Antero Holgado for about eight hours thereby depriving said Artemia Fabreag of her personal liberty. He pleaded guilty (without a counsel) and said that he was just instructed by Mr. Ocampo, which no evidence was presented

Borja v Mendoza Facts: Borja was accused of slight physical injuries in the City of Cebu. However, he was not arraigned. That not withstanding, respondent Judge Senining proceeded with the trial in absentia and rendered a decision
finding petitioner guilty of the crime charged. The case was appealed to the Court o First Instance in Cebu presided by respondent Judge Mendoza. It was alleged that the failure to arraign him is a violation of his constitutional rights. It was also alleged that without any notice to petitioner and without requiring him to submit his memorandum, a decision on the appealed case was rendered The Solicitor General commented that the decision should

observance of the provisions of the Rules of Court, starting with the arraignment of petitioner.

to indict the latter.

Issue: Whether or Not there was any irregularity in the proceedings in the trial court. Held: Yes. Rule 112, section 3 of ROC that : If the defendant appears without attorney, he must be informed by the court that it is his right to have attorney being arraigned., and must be asked if he desires the aid of attorney, the Court must assign attorney de oficio to defend him. A reasonable time must be allowed for procuring attorney. This was violated. Moreso the guarantees of our Constitution that "no person shall be held to answer for a criminal offense without due process of law", and that all accused "shall enjoy the right to be heard by himself and counsel." In criminal cases there can be no fair hearing unless the accused be given the opportunity to be heard by counsel.
The trial court failed to inquire as to the true import of the qualified plea of accused. The record does not show whether the supposed instructions of Mr. Ocampo was real and whether it had reference to the commission of the offense or to the making of the plea guilty. No investigation was opened by the court on this matter in the presence of the accused and there is now no way of determining whether the supposed instruction is a good defense or may vitiate the voluntariness of the confession. Apparently the court became satisfied with the fiscal's information that he had investigated Mr. Ocampo and found that the same had nothing to do with this case. Such attitude of the court was wrong for the simple reason that a mere

be annulled because there was no arraignment. Issue: Whether or Not petitioners constitutional right was violated when he was not arraigned. Held: Yes. Procedural due process requires
that the accused be arraigned so that he may be informed as to why he was indicted and what penal offense he has to face, to be convicted only on a showing that his guilt is shown beyond reasonable doubt with full opportunity to disprove the evidence against him. It is also not just due process that requires an arraignment. It is required in the Rules that an accused, for the first time, is granted the opportunity to know the precise charge that confronts him. It is imperative that he is thus made fully aware of possible loss of freedom, even of his life, depending on the nature of the crime imputed to him. At the very least then, he must be fully informed of why the prosecuting arm of the state is mobilized against him. Being arraigned is thus a vital aspect of the constitutional rights guaranteed him. Also, respondent Judge Senining convicted petitioner notwithstanding the absence of an arraignment. With the violation of the constitutional right to be heard by himself and counsel being thus manifest, it is correct that the Solicitor General agreed with petitioner that the sentence imposed on him should be set aside for being null. The absence of an arraignment can be invoked at anytime in view of the requirements of due process to ensure a fair and impartial trial. Wherefore, the petition for certiorari is granted. The decision of respondent Judge Romulo R. Senining dated December 28, 1973, finding the accused guilty of the crime of slight physical injuries, is nullified and set aside. Likewise, the decision of respondent Judge Rafael T. Mendoza dated November 16, 1976, affirming the aforesaid decision of Judge Senining, is nullified and set aside. The case is remanded to the City Court of Cebu for the prosecution of the offense of slight physical injuries, with due respect and

statement of the fiscal was not sufficient to overcome a qualified plea of the accused. But above all, the court should have seen to it that the accused be assisted by counsel especially because of the qualified plea given by him and the seriousness of the offense found to be capital by the court.

People v Salas Facts: At about 6:00 o'clock in the morning of March 6, 1992, a 60 year old woman, identified as Virginia Talens was found lying dead in a canal at Bo. San Nicolas, Mexico, Pampanga; she was last seen alive at about 3:00 o'clock early morning of March 6, 1992 by Orlando Pangan and Richard Pangan who were with her going home coming from the wake of one Leonardo Flores; both Orlando and Richard Pangan testified that accused was with them in going home at about 3:00 o'clock in the morning of March 6, 1992; Orlando and Richard Pangan reached first their house and left the two on the way and that was the last time Virginia was seen alive; just a few minutes after reaching his house and while inside his house, Orlando Pangan heard a shout; another woman, one Serafia Gutierrez, testified that she likewise was awakened by a shout at about 3:00 in the morning; Dr. Aguda who autopsied the victim found hematoma on the head and chest, an abrasion on the left chin and stabwound on the neck which stabwound, the doctor claims, was the cause of death of the victim; Police Investigator Gonzales who immediately responded upon report, recovered at the scene a pin, the victim's wristwatch, earring, a ring and P135.00 money; he likewise found on March 9, 1992 when he continued his investigation bloodstain on the front door of the house of the accused which bloodstain when submitted for examination was found to be of human blood; one Resultay was with Virginia Talens at about 5:00 afternoon of March 5, 1992 in going to the wake, who claims that Virginia had money on a purse as while they were on the way Virginia bet on a jueteng she saw Virginia got money from her purse a P500.00 bill but as she had no change she instead took P8.00 from her other pocket; one Ramil Talens, a son of the victim corroborated the claim of Resultay that Virginia had with her at that time money worth P2,000.00 as in the morning of March 5, 1992 he gave her mother for safekeeping the sum of P1,500.00 which he claims his mother placed in her purse and claims further that at the wake, he asked and was given P50.00 by his mother as he also participated in the gambling thereat, however, the purse of Virginia containing about P2,000.00 was no longer to be found when she was found dead; Orlando Pangan saw the accused gambled in the wake;

Virginia likewise gambled at the wake; accused had been working for three days before March 6 at Sta. Ana, Pampanga and up to March 5, 1992, but the following day, he did not anymore report for work at Sta. Ana, Pampanga, was no longer to be found and was last seen at about 3:00 morning together with Virginia Talens on their way home coming from the wake; the parents of [the] accused were informed by Investigator Gonzales that their son was the suspect and adviced them to surrender him, but since March 6, 1992 when accused left Mexico, Pampanga, he returned only on September 19, 1992 at Arayat, Pampanga, not at Mexico, Pampanga where he was ultimately apprehended by the Mexico Police on September 22, 1992 after chancing on a radio message by the police of Arayat to their Provincial commander that a vehicular incident occurred at Arayat, Pampanga where one Elmer Salas was the victim and was hospitalized at the district hospital at Arayat, Pampanga where he used the name of Rommel Salas and not Elmer Salas. The trial court rendered convicting Salas for Robbery with Homicide Issues: (1) Whether or Not there is evidence sufficient to sustain a conviction of the appellant of the crime of Robbery with Homicide. (2) Whether or Not the appellants crime homicide or robbery with homicide. Held: There was no eyewitness or direct evidence; either to the robbery or to the homicide and none of the things allegedly stolen were ever recovered. However, direct evidence is not the only matrix from which the trial court may draw its findings and conclusion of culpability. Resort to circumstantial evidence is essential when to insist on direct testimony would result in setting felons free. For circumstantial evidence to be sufficient to support a conviction, all the circumstances must be consistent with each other, consistent with the theory that the accused is guilty of the offense charged, and at the same time inconsistent with the hypothesis that he is innocent and with every other possible, rational hypothesis excepting that of guilt. All the circumstances established must constitute an unbroken

chain which leads to one and fair and reasonable conclusion pointing solely to the accused, to the exclusion of all other persons, as the author of the crime. The facts and circumstances consistent with the guilt of the accused and inconsistent with his innocence can constitute evidence which, in weight and probative value, may be deemed to surpass even direct evidence in its effect on the court. The fatal stabbing of Virginia Talens occurred at around 3:00 a.m. of March 6, 1992. Appellant hastily abandoned his house in Barrio San Nicolas, Mexico, Pampanga, his residence since childhood, on that very date. Appellant was nowhere when his coworker and barrio mate, Eduardo Bagtas, came to appellant's house to fetch him for work at around 6:30 to 7:00 a.m. of March 6, 1992. Appellant also abandoned his job as a painter in Sta. Ana, Pampanga, on March 6, 1992, the date of the crime, leaving behind an unfinished painting project. He was not seen again from said date. Police investigators found human bloodstains on the front door of appellant's house, on his clothing, and on his yellow slippers after the victim was killed. Despite efforts of the police to find appellant as the principal suspect, a fact known to appellant's family and neighbors, appellant did not present himself to the authorities. Appellant was apprehended only a full six months after the date of the crime, following his confinement in a hospital in Arayat, Pampanga because he was sideswiped by a Victory Liner bus in Arayat. When hospitalized, appellant used the alias Rommel Salas, instead of his true name Elmer Salas. These circumstances denote flight, which when unexplained, has always been considered by the courts as indicative of guilt. Both appellant and victim gambled at the wake they attended. The victim was, in fact, enjoying a winning streak when her son, Ramil Talens, came to fetch her but which he failed to do because his mother was winning, and she refused to leave. The purse of Talens containing cash was gone when her corpse was found in the canal with a stab wound and bruises. What was left was a safety pin which victim used to fasten the missing purse to her clothes. Denial is an inherently weak defense which must be buttressed by strong evidence of non-culpability to merit credibility. Denial is negative and self-serving and cannot be

given greater evidentiary weight over the testimonies of credible witnesses who positively testified that appellant was at the locus criminis and was the last person seen with the victim alive. The absence of evidence showing any improper motive on the part of the principal witness for the prosecution to falsely testify against the appellant strongly tends to buttress the conclusion that no such improper motive exists and that the testimony of said witnesses deserve full faith and credit. The essence of voluntary surrender is spontaneity and the intent of the accused to give himself up and submit himself unconditionally to the authorities either because he acknowledges his guilt or he wants to save the State the trouble of having to effect his arrest. Spontaneity and intent to give one's self up are absent where the accused went into hiding for six months after the incident and had to resort to an alias when he was involved in an accident being investigated by the police authorities. Robbery with Homicide is a special complex crime against property. Homicide is incidental to the robbery which is the main purpose of the criminal. In charging Robbery with Homicide, the onus probandi is to establish: "(a) the taking of personal property with the use of violence or intimidation against a person; (b) the property belongs to another; (c) the taking is characterized with animus lucrandi; and (d) on the occasion of the robbery or by reason thereof, the crime of homicide, which is used in the generic sense, was committed." Although there was no witness as to the actual robbing of the victim, there is testimony that the victim had more or less P2,000.00; and wore gold earrings valued at P750.00. These were never recovered.
While there is indeed no direct proof that Virginia Talens was robbed at the time she was killed, we may conclude from four circumstances that the robbery occasioned her killing: (1) Both appellant and victim gambled at the wake. (2) The appellant knew that victim was winning. (3) The victim was last seen alive with appellant. (4) The victim's purse containing her money and earrings were missing from her body when found. The decision of the regional trial court is affirmed. Costs against appellant. So ordered.

People v. Prieto G.R. No. L-399 | January 29, 1948 Plaintiff: People of the Philippines Defendant: Eduardo Prieto (Eddie Valencia) Ponente: Tuason, J. FACTS: Prieto was prosecuted in the Peoples Court for 7 counts of treason. Initially, he pleaded not guilty to every charge. Later on, he entered a plea of guilty to counts 1, 2, 3 and 7, and maintained his original plea to counts 4, 5 and 6. The prosecutor only presented evidence to count 4 as he admitted insufficiency of evidence as to counts 5 and 6. The court found him guilty to all counts except 5 and 6 of treason complexed by murder and physical injuries.

ISSUE: What is the criminal liability of Prieto? DECISION (Not guilty of count 4, guilty of treason in counts 1, 2, 3 and 7): Prieto is acquitted in count 4 as the of twowitness principle requirement was not satisfied. They failed to corroborate each other: o Juanito Albano testified that the accused and other Filipino undercovers and Japanese officers caught an American aviator and had him carried to town on a sled pulled by a carabao. That on the way, Prieto walked behind the sled and the American was taken to the Kempetai headquarters. o Valentin Cuison testified that he saw the accused following the American whose hands were tied while walking and that he struck the flier with a rope. There was no mention of a sled and nor did he see Juanito Albano. There is no crime of treason complexed with other felonies because these were not separate offenses from treason. o When a deed is charged as an element of treason, it becomes identified with it and cannot be subject of a separate punishment, or used in

combination with treason to increase the penalty. o Murder or physical injuries are charged as overt acts of treason and cannot be regarded separately under their general denomination. o But the brutality which accompanied the killing and the physical injuries are taken as aggravating circumstances since it augmented the sufferings of the offended parties unnecessarily to the attainment of the criminal objectives. o But there is a mitigating circumstance of plea of guilty, hence, the punishment should be reclusion perpetua. Other issue: o There is a presumption in favour of legality and regularity of the proceedings and the presumption that the accused was not denied of his rights. o The fact that the attorney appointed to defend Prieto is reluctant to accept the designation is not sufficient to overcome the presumption. The present counsel sincerely believes that the said Attorney Carin did his best, although it was not the best of a willing worker.

Gimenez v Nazareno Facts: 1 Respondent de la Vega, along with five others, was charged with murder. He was arraigned and pleaded not guilty. Before the scheduled date of the first hearing, he escaped from detention. 2 Prosecutors file a motion to proceed with the hearing in absentia, invoking Sec 19, Art IV of the 1973 Constitution. SEC. 19. In all criminal prosecutions, the accused shall be presumed innocent until the contrary is proved, and shall enjoy the right to be heard by himself and counsel, to be informed of the nature and cause of the accusation against him, to have a speedy, impartial, and public trial, to meet the witnesses face to face, and to have compulsory process to secure the attendance of witnesses and the production of evidence in his behalf. However, after arraignment trial may proceed notwithstanding the absence of the accused

provided that he has been duly notified and his failure to appear is unjustified. 3 The lower court proceeded with the trial. The case was dismissed against the five accused, while proceedings against de la Vega were held in abeyance. 4 The lower court denied recon of the abeyance portion of the decision. Hence, this petition. Issue #1: WON court loses jurisdiction over an accused who escapes from detention after arraignment Held #1: NO 1 In criminal cases, jurisdiction over the person of the accused is acquired either by his arrest for voluntary appearance in court. Such voluntary appearance is accomplished by appearing for arraignment as what accused-private respondent did in this case. 2 Where the accused appears at the arraignment and pleads not guilty to the crime charged, jurisdiction is acquired by the court over his person and this continues until the termination of the case, notwithstanding his escape from the custody of the law. Issue #2: WON an accused who has been tried in absentia retains his right to present evidence and cross examine witnesses Held #2: NO 1 REQUISITES FOR TRIAL IN ABSENTIA: 1. that there has been an arraignment; 2. that the accused has been notified; and 3. that he fails to appear and his failure to do so is unjustified. 2 The right to present evidence and cross-examine witnesses is a personal right and can be waived. Failure to appear during trial, after due notice, constitutes a waiver of these rights. Sec 1(c), R 115, 1985 ROC: . . . The absence of the accused without any justifiable cause at the trial on a particular date of which he bad notice shall be considered a waiver of his right to be present during that trial. When an accused under custody had been notified of the date of the trial and escapes, he shall be deemed to have waived his right to be present on said date and on all subsequent trial dates until custody is regained. . . . 3 Rendering judgment for a trial in absentia is not a violation of the right to be presumed innocent. The accused is still presumed to be innocent, and his conviction must be based on evidence showing guilt beyond reasonable doubt.

US v Tan Teng Facts: The defendant herein raped Oliva Pacomio, a seven-year-old girl. Tan Teng was gambling near the house of the victim and it was alleged that he entered her home and threw the victim on the floor and place his private parts over hers. Several days later, Pacomio was suffering from a disease called gonorrhea. Pacomio told her sister about what had happened and reported it to the police. Tan Teng was called to appear in a police line-up and the victim identified him. He was then stripped of his clothing and was examined by a policeman. He was found to have the same symptoms of gonorrhea. The policeman took a portion of the substance emitting from the body of the defendant and turned it over to the Bureau of Science. The results showed that the defendant was suffering from gonorrhea. The lower court held that the results show that the disease that the victim had acquired came from the defendant herein. Such disease was transferred by the unlawful act of carnal knowledge by the latter. The defendant alleged that the said evidence should be inadmissible because it was taken in violation of his right against self-incrimination. Issue: Whether or Not the physical examination conducted was a violation of the defendants rights against selfincrimination. Held: The court held that the taking of a substance from his body was not a violation of the said right. He was neither compelled to make any admissions or to answer any questions. The substance was taken from his body without his objection and was examined by competent medical authority. The prohibition of self-incrimination in the Bill of Rights is a prohibition of the use of physical or moral compulsion to extort communications from him, and not an exclusion of his body as evidence, when it may bematerial. It would be the same as if the offender apprehended was a thief and the object stolen by him may be used as evidence against him.

You might also like